Download as pdf or txt
Download as pdf or txt
You are on page 1of 115

English & Reading

(Answer Keys & explana ons)

Compiled by Mr. Yann

2022-2023
PRACTICE TEST 1 ANSWER KEY 61

ANSWER KEY

English Test
1. B 21. A 41. B 61. D
2. G 22. F 42. H 62. G
3. C 23. D 43. A 63. A
4. J 24. F 44. J 64. J
5. A 25. B 45. C 65. C
6. G 26. F 46. J 66. J
7. B 27. D 47. A 67. B
8. J 28. G 48. G 68. J
9. B 29. A 49. A 69. B
10. F 30. F 50. H 70. F
11. D 31. D 51. A 71. D
12. J 32. F 52. H 72. G
13. A 33. A 53. D 73. D
14. G 34. H 54. H 74. H
15. C 35. A 55. A 75. A
16. G 36. F 56. J
17. A 37. A 57. B
18. J 38. J 58. F
19. B 39. D 59. A
20. F 40. F 60. J
PRACTICE TEST 1 ANSWER KEY 63

Reading Test Science Reasoning Test


1. B 21. B 1. B 21. C
2. F 22. H 2. H 22. H
3. C 23. A 3. D 23. C
4. F 24. H 4. F 24. J
5. D 25. D 5. A 25. C
6. H 26. H 6. J 26. F
7. A 27. A 7. B 27. A
8. G 28. J 8. F 28. G
9. A 29. D 9. B 29. B
10. J 30. G 10. J 30. G
11. B 31. A 11. A 31. A
12. G 32. J 12. H 32. J
13. D 33. D 13. C 33. A
14. F 34. G 14. F 34. G
15. C 35. C 15. C 35. C
16. F 36. H 16. J 36. G
17. C 37. B 17. D 37. C
18. G 38. F 18. H 38. J
19. D 39. C 19. B 39. C
20. J 40. J 20. J 40. J
PRACTICE TEST 1 ANSWERS AND EXPLANATIONS 69

ANSWERS AND EXPLANATIONS

English Test Explanations

PASSAGE I
1. The best answer is B. In this sentence, the word directly related information, such as a list or an
“obsessed” is most appropriate to describe the example, so answer choice C should be eliminated.
action taking place. The word “obsessive” can be
8. The best answer is J. The word “was” makes the
used as an adjective. The participle “obsessing” is
most sense in the sentence, because it clearly and
not appropriate, and “obsessioned” is not a word,
simply indicates the past tense. The phrase “could
so answer choices C and D can be eliminated.
of become” can never be correct. “Could of been” is
2. The best answer is G. Answer choice G is the not appropriate in standard written English. Some
most clear and concise, because it simply uses the people incorrectly use “could of” when they should
verb “pay.” As it is written, the sentence is wordy use “could’ve,” which is the contraction of “could
and redundant. Answer choice H, “paying money,” have.”
sounds awkward and is grammatically incorrect
9. The best answer is B. The sentence is composed
in conjunction with the phrase that precedes it,
of a main clause (“The monthly highlighting went
“is willing to.” Answer choice J is written in
well”) followed by a subordinator (“except for”),
the past tense while the rest of the sentence is
then an extended noun phrase (“those times when
written in the present tense and, therefore, must
my hair turned out a little too subdued, making
be eliminated.
me look partially gray instead of brunette.”). Espe-
3. The best answer is C. Answer choice C is gram- cially as it is preceded by a subordinator, such a
matically correct and makes sense idiomatically. lengthy component of the sentence calls to be offset
Answer choices A and B indicate that natural red- by a comma.
heads actually want to possess brunettes, which
10. The best answer is F. The sentence as it is writ-
does not make sense. Answer choice D does not
ten makes the most sense in context. It sufficiently
make sense and is not correct for standard written
indicates that, despite some setbacks, the author’s
English.
experience with highlights has been mostly posi-
4. The best answer is J. The first part of the sentence tive. Answer choice G implies that the author was
and the second part of the sentence are independent making some sort of list regarding her and her
clauses that refer to different hairstyles. Answer feelings towards her highlights. Answer choice H
choice J, “and,” makes most sense here. Answer indicates some sort of contrast that is simply not
choice G, “however,” requires punctuation to fit in present in the paragraph. Answer choice J sug-
the sentence (normally between commas). Answer gests that the hairdresser’s mistakes pleased the
choice F implies that teenagers enjoy experiment- author.
ing with their hair as a result of men and women
11. The best answer is D. Sentence 5 follows from
cringing at the sight of gray hair. Answer choices G
the information in Sentence 3. Likewise, Sen-
and H imply that the two phrases negate each
tence 4 relies on Sentence 5, and also creates a
other.
good transition into the next paragraph.
5. The best answer is A. The correct preposition to
12. The best answer is J. To maintain parallel tense
use here is “of.” The phrase “result of” is idiomatic.
in the sentence, since Donna “was” surprised at
6. The best answer is G. This answer choice cor- the author’s request, the simple past tense of the
rectly precedes the verb “mixing” with its modifier verb “gather” also needs to be used. The sentence
“rapidly.” is written in the present tense; therefore answer
choice F can be eliminated. Answer choices G
7. The best answer is B. The sentence does not
and H are not in the parallel tense.
have a natural pause at this point in the sentence,
so a comma is inappropriate. In addition, you can 13. The best answer is A. The coordinating con-
eliminate answer choices A and D. A colon should junction “and” is the most concise choice; it
be used after a complete statement to introduce joins two functionally parallel elements within the
70 PRACTICE TEST 1 ANSWERS AND EXPLANATIONS

sentence—here, two nouns: “tears” and “laugh- parallel between verbs. Answer choices G and H
ter.” The phrases “along with” and “as well as” use “begun,” the past participle of “begin,” and
serve a similar function to the coordinating con- can therefore be eliminated.
junction “and,” but they are awkward and neither
19. The best answer is B. The phrase “that being iron”
clear nor concise. Answer choice C is incorrect
does not make sense. The rest of the answer choices
because “or” does not make sense in this context
correctly use parentheses or commas in setting off
with “seesawing.”
their respective appositive phrases.
14. The best answer is G. Since the preceding sen-
20. The best answer is F. The word “lugging” is
tence does not summarize the essay or relate to
the most descriptive word. It signifies carrying or
the introduction, answer choices F and H can be
pulling something heavy. None of the other answer
eliminated. This sentence is specifically about the
choices addresses the effort it took to move the
narrator ceasing her crying, which is not the pur-
anvil from Pennsylvania to Michigan.
pose of the essay. Eliminate answer choice J. The
realization of her vanity puts her situation in per- 21. The best answer is A. The phrase “get started” in
spective; therefore, answer choice G is the best answer choice A clearly indicates that Lee was just
answer. beginning his career in blacksmithing. The other
answer choices suggest that his career had already
15. The best answer is C. The passage simply dis-
started.
cusses the narrator’s experiences in dyeing her
own hair. It is unlikely that the author’s decision 22. The best answer is F. In the context of the
to change her hair color would ease the appre- sentence, “its” is in the possessive form; there-
hension of others who were considering dyeing fore, no apostrophe is necessary. “It’s” means “it
their hair. The fact that dyeing her hair gave is.” “Its”’ (note the apostrophe at the end) does
her a bald spot would probably deepen the fears not exist in English beacuse “it” is a singular
of those who were feeling anxious about dye- pronoun.
ing their hair. Eliminate answer choice A. Answer
23. The best answer is D. Omitting the underlined
choice B can also be eliminated; the passage only
portion of the sentence is the best choice in
discusses the author and other “mature” women
this instance. In any form, the information pro-
dyeing their hair. Answer choice D is incorrect
vided in this sentence is distractive—not related
because the passage as a whole is neutral and does
to the main topic of the essay—and is therefore
not attempt to influence people regarding changing
unnecessary.
hair color.
24. The best answer is F. Answer choice F provides
a logical and relevant introduction to the topic
PASSAGE II
of the paragraph. Answer choice H is incorrect
16. The best answer is G. Answer choice G is a com- because the paragraphs preceding and following
mon phrase, and sounds the best in the context of this sentence have nothing to do with moving
this sentence. It is appropriate to use “American” equipment. Answer choices G and J can be elimi-
as an adjective to describe the noun “heritage.” The nated because neither has anything to do with the
other answer choices are grammatically incorrect. construction of Lee’s first blacksmith shop, the
Also, answer choice J suggests that the “heritage” subject of the paragraph.
belongs to only one “American,” which does not
25. The best answer is B. This answer choice
fit the context of the paragraph.
is correct because it is the only one that is
17. The best answer is A. To state that Walker grammatically proper and makes sense. Answer
Lee “still practices” is clear and concise. Answer choice A does not make sense because “but”
choices B and C are wordy and redundant in is used to introduce a contradictory element;
saying that Lee “still continues to practice” and a “crude” structure standing “only nine years”
“continues to still practice.” Answer choice D is appears to be instead a correlational relationship.
awkward. Answer choice C is ungrammatical and D violates
tense agreement with “was.”
18. The best answer is J. This passage is written in
the past tense. “Began” is the simple past form 26. The best answer is F. If two adjectives modify a
of the verb “begin,” therefore answer choice J is noun in the same way, they must either be separated
correct. Answer choice F is written in the past by a comma or joined with the word “and” with no
perfect tense, and therefore does not maintain the comma.
PRACTICE TEST 1 ANSWERS AND EXPLANATIONS 71

27. The best answer is D. The phrase “at a family thing with a broom,” he would have to have
event” is a descriptive phrase that must be set “flushed” it down the toilet.
off by commas. Answer choice C is incorrect
36. The best answer is F. This part of the sentence
because subject and verb must not be separated
requires a verb, so you can eliminate answer
by a comma. Similarly, in B, a comma makes an
choice J. The adjective “deadly” is used to describe
incorrect division of a compound.
the scorpions. If the word “deadlier” is used, the
28. The best answer is G. The act of proclaiming took word “more” cannot precede it; therefore, answer
place during the event, so it is a completed action choice G can be eliminated. The comma before
and should be in the past tense. “and” indicates the phrase preceding it must be
an independent clause, meaning one that con-
29. The best answer is A. The information given in
tains a verb phrase that could stand alone in
Sentence 2 merely describes the knife from the pre-
a sentence. The gerund (“-ing”) form violates
ceding sentence, and has nothing to do with the
this test.
creation of Lee’s first object.
37. The best answer is A. This answer choice gives a
30. The best answer is F. No specific person or
logical explanation for why Diana would not seek
point in time is mentioned, and the remainder
professional help as the condition of her arm con-
of the sentence is written in the present tense.
tinued to worsen. The remaining choices are not
Thus, the present tense of the verb “watch” must
supported by the context.
be used.
38. The best answer is J. This question tests your abil-
ity to express yourself clearly and simply. Because
PASSAGE III
the sentence states that scorpions will sting any-
31. The best answer is D. Using the past tense verb one they accidentally encounter, using the word
“began” makes the most sense because the narra- “inadvertently” would be redundant and unneces-
tor is recalling what happened when she heard the sary. The word “crawl” is the clearest and most
story. concise choice.
32. The best answer is F. Answer choice F correctly 39. The best answer is D. This question tests your
places a comma following the clause that begins ability to accurately create the possessive form of
with the subordinator “As,” which describes words. In this case, the narrator is talking about
what happened that caused the narrator’s eyes the homes of many scorpions instead of the home
to pop out of her head and her jaw to drop. of one specific scorpion. The plural possessive
Answer choice G creates a run-on sentence. form of scorpion, “scorpions’,” must be used. To
Answer choices H and J incorrectly place com- make a plural word possessive, you must place an
mas following the word “could,” which creates apostrophe after the pluralizing “s.”
awkward and ungrammatical sentences.
40. The best answer is F. Answer choice G is awkward
33. The best answer is A. An easy way to figure because its components are not written in logical
out the answer to this question is to look at the order. Answer choice H places an unnecessary
preceding sentence. In that sentence, the narra- comma after the word “species.” Answer choice J
tor says, “I could.” To maintain verb parallelism is ambiguous as to whether the “ninety species” in
in the paragraph, the phrase “I could” should be question constitute all the world’s scorpions or just
repeated. those native to the United States.
34. The best answer is H. In this sentence, the word 41. The best answer is B. In this part of the sentence,
“that” is unnecessary, because the normal clause- the word “which” introduces a clause descriptive
introducing function of the word “that” is satisfied of the noun that precedes it, “the Bark Scorpion.”
by “how.” Answer choice G is wrong because
42. The best answer is H. This question tests your
“because” does not make sense after “about.”
ability to discern which details are important to the
Answer choice J lacks a clause-introducer.
subject of an essay. In this case, the author previ-
35. The best answer is A. This question tests your ously mentioned that scorpions can be found in the
ability to maintain parallel structure in a sentence. home, and it is obvious that they still live outside.
Each verb in the sequence of events is written in the The underlined portion reinforces the notion that
past tense. Since the sister’s husband “threw back scorpions can be found both inside the home and
the bedcovers” and “began beating the dreaded outside the home.
72 PRACTICE TEST 1 ANSWERS AND EXPLANATIONS

43. The best answer is A. This sentence follows a can be eliminated because they are incorrect as
standard verb pattern for hypothetical situations. idiomatic phrases.
“If” begins a clause in simple present tense, which
50. The best answer is H. In this case, the word
precedes a clause in the imperative (command)
“promised” is used as an adjective. Thus, answer
form. For example, “If you swim today, apply
choices F and G can be eliminated. Answer choice J
sunscreen.”
is an adjective, but the definitions of “promised”
44. The best answer is J. This question requires you and “promising” differ. The reward was assured,
to put things in logical order, and to decide whether or promised, to Helga. It was not likely to develop
the underlined portion is relevant to the paragraph. nor did it show potential, both aspects of the defi-
In this case, it is best to omit the underlined portion nition of “promising,” thus answer choice J can be
because it does not add any necessary information eliminated.
to the paragraph; it is an irrelevant detail. The rest
51. The best answer is A. Answer choice B creates an
of the sentences are already in the most logical
incomplete sentence, so it can be eliminated. Since
order.
the action was continuously occurring in the past,
45. The best answer is C. Answer choices A and B you would say that she “had been living.”
can be eliminated immediately because the sim- 52. The best answer is H. Answer choice J can
ple answer to the question is no. This essay does be eliminated because the underlined portion is
not provide professional advice on the treatment of preceded by the article “a.” The article “the”
scorpion stings. The essay is merely a recollection must precede the word “greatest.” Since “reward”
of a time when the narrator’s sister was stung by a serves correctly as a noun in this sentence, answer
scorpion and the narrator is offering advice based choices F and G can be eliminated. A “great
solely on personal experience and opinion. reward,” answer choice H, is the best and most
reasonable choice.
PASSAGE IV 53. The best answer is D. Sentence 1 explains how
46. The best answer is J. Answer choice F is incorrect Helga became interested in the contest, so it
because dashes should only be used to place special must be first. This eliminates answer choice C.
emphasis on a certain word or phrase in a sen- It makes sense that the requirements would be
tence, which is unnecessary here. Answer choice G listed next and that the comments about the bicycle
is incorrect because the preceding phrase is not an skirt would follow. The last sentence, Sentence 4,
independent clause, thus must not be separated by explains why Helga decided to make the journey
a semicolon. Answer choice H is incorrect because despite the embarrassment of wearing a bicycle
it creates a run-on sentence. Answer choice J cor- skirt.
rectly identifies the fact that the word “if” begins a 54. The best answer is H. The actions of Helga and her
clause that must be separated from the rest of the daughter should be written in the simple past tense
sentence by a comma wherever the clause ends. in order to maintain verb parallelism throughout the
In this case, the second clause clearly begins with paragraph. Answer choice J is in the past tense, but
repetition of the subject “you.” it is wordy. “Have,” in any form, is not necessary
47. The best answer is A. This is the most clear and to this phrase.
concise answer choice. The others are awkward. 55. The best answer is A. This answer choice is the
Answer choice C is incorrect because “always” clearest and most concise. The other choices are
modifies “accompany” and must, in this case, pre- wordy and awkward.
cede it. Answer choice D does not include the word
“always,” which causes the sentence to lose a key 56. The best answer is J. “Nowhere to be found” is a
detail. common idiomatic phrase, making it the most clear
and concise choice. The other answer choices are
48. The best answer is G. “Non-compliance” awkward.
describes the wrong each side feels the other com-
57. The best answer is B. Answer choice A indicates
mitted, thus the lawsuits assert “non-compliance
that the following sentence is a result of what
on both sides.” This eliminates all answer choices
comes before. In this case, the phrases are not
except answer choice G.
causally connected, so answer choice A is incorrect.
49. The best answer is A. Answer choice B uses Answer choice C is awkward and answer choice D
an unnecessary comma. Answer choices C and D is wordy. The phrase “in fact” is not necessary to
PRACTICE TEST 1 ANSWERS AND EXPLANATIONS 73

this passage. Answer choice B is the clearest and 64. The best answer is J. Before a term is defined, its
most concise choice. relevance to the passage must be stated. Further-
more, among the other answer choices, circadian
58. The best answer is F. The words “leaving only” rhythms are not specifically mentioned.
begin a descriptive clause, which must be set off
by a comma. Answer choice G creates a run-on 65. The best answer is C. While the sentence as it is
sentence. Semicolons must be used to separate two written may be grammatical, its use of punctuation
independent clauses; therefore, answer choice H is excessive. Answer choice C provides a fluid, con-
is incorrect. Answer choice J uses a comma cise transition to the details of the human “sleeping
incorrectly, separating “only” from the phrase it and waking” cycle.
modifies.
66. The best answer is J. This sentence elaborates
59. The best answer is A. Answer choice A gives a spe- on the statement from the previous sentence by
cific reason as to what Helga intended to use the citing an example. It does not provide supple-
$10,000 prize for. The passage clearly states that mentary (Answer choice F: “In addition”), parallel
without this prize money, the Estby farm would (G: “Likewise”), or opposing (H: “Instead”) evi-
face foreclosure. Answer choice B simply restates dence.
the fact that Helen wanted the $10,000 she would 67. The best answer is B. It is appropriate to use the
win if she completed the cross-country walk, but plural possessive pronoun “our” when referring to
does not explain what she would use the money the inner clocks of human beings.
for. Answer choice C is incorrect because no logical
connection between the prize money and the chil- 68. The best answer is J. The sentence introduces
dren’s diphtheria is made in the passage. Answer a factor adversely affecting circadian rhythms,
choice D is outside the scope of the passage as well; which does not require additional transition words.
there is no mention of Clara gaining experience Answer choices F and H create incomplete sen-
anywhere in the passage. tences.
60. The best answer is J. Answer choices F and 69. The best answer is B. “Well-tuned” stands by
G are incorrect because the writer should not itself as a satisfactory idiomatic expression mean-
make this addition to the passage; it is irrelevant. ing “optimized.” Introducing “high” or “highly”
Answer choice H identifies an unimportant detail makes the sentence unnecessarily wordy.
of the great-great-grandson’s story as the reason
70. The best answer is F. Answer choices G and H
the sentence does not belong.
may be eliminated for their wordiness. Answer
choice J may be eliminated because it makes too
PASSAGE V strong an assertion about the link between long air
travel and headaches. Answer choice F correctly
61. The best answer is D. Appositives, like “partic-
uses “Often” to define the frequency of passengers’
ularly via airplane,” must be separated from the
headaches.
sentence by commas. They are easily identified
because they can be omitted from the sentence 71. The best answer is D. As it is written in the
without rendering the sentence ungrammatical. passage, this verb phrase is wordy, as it is writ-
ten in passive voice (“being”). This same prin-
62. The best answer is G. Because the author wishes
ciple eliminates answer choice B. Answer choice
to add more detail, the best answer choice will be
C may be eliminated for its unnecessary use of
the one that includes the most descriptive language.
commas.
Answer G is the best alternative because it includes
the explanatory detail “also known as jet lag” and 72. The best answer is G. When adjectives modify
correctly sets it apart with commas. a noun in a similar way, they are separated from
each other with commas or “and,” just as in a list.
63. The best answer is A. The sentence identifies
Therefore, there must be a comma between “dry”
jet lag as a “sleeping disorder,” but the word
and “pressurized.” It is not necessary to include
“although” indicates that what follows are mitigat-
a comma after “pressurized,” because it is fol-
ing factors. First, jet lag is a “temporary condition”
lowed directly by the noun that is being modified
and second, jet lag is “not as serious” as other
(atmosphere).
sleeping disorders. Answer choice B appears to
define correctly the lack of seriousness, but is 73. The best answer is D. Using a comma to join inde-
missing the critical first “as” to make a comparison. pendent clauses creates a comma splice. The other
74 PRACTICE TEST 1 ANSWERS AND EXPLANATIONS

choices present several acceptable ways to separate Choice H corresponds to the latter location, “before
independent clauses. Sentence 6.”
74. The best answer is H. This answer choice 75. The best answer is A. This sentence would
mentions the body’s “new environment.” Logical provide a reason why understanding ways to
places for the sentence, thus, would be after the correct jet lag—which is the focus of the essay—is
technique cited for acclimating to eastbound travel important. It follows that the sentence would be
(before Sentence 4) and after the technique cited for placed after the assertion that jet lag is considered
acclimating to westbound travel (after Sentence 5). a minor sleep disorder.
PRACTICE TEST 1 ANSWERS AND EXPLANATIONS 83

Reading Test Explanations

PASSAGE I

1. The best answer is B. The passage takes place on whole river there was nothing that looked half so
a ship, the Nellie, and the narrator is one of the nautical. He resembled a pilot, which to a seaman
crew members. He uses words like “we” and “us” is trustworthiness personified.” This best supports
when referring to the crew, implying his member- answer choice A.
ship to this group. The other answer choices are not
supported by the passage. 8. The best answer is G. There are clues in the pas-
sage to indicate that the narrator, as well as the other
2. The best answer is F. Although the passage states, crew members, were not thrilled when Marlow
“for some reason or another we did not begin that began to speak. Marlow’s very first comment was
game of dominoes,” it is reasonable to assume that “accepted in silence” and “no one took the trouble
it was because they were too tired from the use to grunt even.” In the next paragraph the narrator
of the words “lazily” and “meditative.” The other begins to realize that the crew was “fated, before
answer choices are not supported by the passage. the ebb began to run, to hear about one of Marlow’s
3. The best answer is C. The passage states that “the inconclusive experiences.” Since resigned means
Lawyer . . . had, because of his many years and “accepting that something can not be avoided” and
many virtues, the only cushion on deck,” indi- tolerance means “patience,” making G the best
cating that since he was the eldest crew member answer.
and had the other crew members’ respect, he was 9. The best answer is A. In the third paragraph the
afforded the comfort of the cushion. The other passage states that the men’s mutual interest in
answer choices are not supported by the passage. the sea created a bond between them capable of
4. The best answer is F. The definition of “placid” is “holding (their) hearts together through long peri-
“not easily excited or upset; calm.” Since the men ods of separation,” “making (them) tolerant of
on the ship were feeling “meditative” and seemed each other’s yarns,” and making them accepting
not to have an abundance of energy, it makes sense of each other’s “convictions.” Answer choice A is
that they simply wanted to sit calmly. The other not mentioned in the passage.
answer choices are not supported by the context of
10. The best answer is J. The passage states that the
the passage.
crewmen watched him “affectionately,” meaning
5. The best answer is D. In the fifth paragraph the “showing fondness or liking.” The captain is also
narrator is describing how Marlow is unlike most described as “trustworthiness personified,” indicat-
sailors: “The yarns of seamen have a direct sim- ing that the other crew members have the utmost
plicity, the whole meaning of which lies within the faith and trust in him. This best supports answer
shell of a cracked nut. But Marlow was not typi- choice J.
cal . . . and to him the meaning of an episode was
not inside like a kernel but outside . . .” This is to
PASSAGE II
say that typical sailors tell simple, uncomplicated
tales, while Marlow tends to tell stories that are 11. The best answer is B. As stated in the passage,
layered and complex. This best supports answer “in the past 30 years, known supplies have dwin-
choice D. dled from almost 300 TCF to around 150 TCF,”
or known supplies have decreased by about
6. The best answer is H. Since Marlow states that
50 percent. Answer choice A is incorrect because
the episode which he is about to recount “seemed to
the passage states that at predicted rates of con-
somehow throw a light on everything about (him),”
sumption, the United States’ natural gas supply
we can assume that this experience had a profound
would be exhausted in approximately five years.
effect on him. The other answer choices are either
Answer choice C is incorrect because natural gas
not supported by the passage or are beyond the
provides for roughly 25 percent of America’s
scope of the passage.
energy needs, which has nothing to do with the
7. The best answer is A. The passage states that, “The decrease in supply. Answer choice D is incorrect
Director of Companies was our captain and our because the passage states that it is extremely diffi-
host. We four affectionately watched his back as cult to obtain natural gas from other countries, not
he stood in the bow looking toward the sea. On the from within the US.
84 PRACTICE TEST 1 ANSWERS AND EXPLANATIONS

12. The best answer is G. At the end of the passage, source of natural gas. The other answer choices are
the author states that “consumers and business not supported by the passage.
leaders should not rely on liquid natural gas to
17. The best answer is C. The context surrounding the
solve America’s energy needs.” This can also be
inferred from the point that natural gas is currently word phenomenal discusses the surprisingly large
only supplying approximately 25 percent of the growth expected in natural gas demand and the
nation’s energy needs, and even at this level there huge impact that such growth will have on deple-
is much concern over whether supplies will run out. tion of the resource. This context clearly indicates
Answer choice H may appear to be correct, but the that the demand is increasing at “phenomenal,” or
passage merely states that countries such as Japan extraordinary, rates. The other answer choices are
and China will also be searching for fuel sources, not supported by the context of the passage.
including liquid natural gas, outside of their own 18. The best answer is G. Answer choice G is the
countries in the future. only reason that America is choosing liquid natu-
ral gas; consumers are demanding it so America
13. The best answer is D. This question can be diffi-
must provide it. Answer choices F and H express
cult if you do not read the answer choices carefully.
current problems with choosing liquid natural gas;
The third paragraph is devoted to a discussion
transportation and processing are both very costly
on the limited availability of liquid natural gas,
relative to other fuel sources. Answer choice J is a
and the expense of processing the gas, which
potential problem. Liquid natural gas is inherently
makes answer choice D the best selection. Answer
expensive due to its transportation and processing
choice A may appear to be correct; however, the
costs. If natural gas prices are low, the market
passage focuses on the supply and use of liquid
for liquid natural gas will plummet, making liquid
natural gas around the world. The passage does
natural gas an uneconomical choice for consumers.
not discuss the supply and use of any other energy
sources. Answer choice B was mentioned briefly in 19. The best answer is D. As stated in the paragraph,
the passage, but is not a main idea. Answer choice “natural gas demand is increasing at phenomenal
C is beyond the scope of the passage. rates” and its consumption is expected to grow from
22 trillion cubic feet per year to 32 trillion cubic
14. The best answer is F. As stated in the second
feet per year in less than a decade. Answer choice C
paragraph, it is predicted that “liquid natural gas
may appear to be correct; however, the passage
imports will increase by almost 500 percent in a
simply states that Canada is the largest liquid nat-
few short years.” Although America may be reluc-
ural gas supplier for the United States alone. The
tant to import liquid natural gas, it is necessary
passage does not compare Canada’s liquid natural
for the nation to do so in order to relieve and/or
gas exports to those of any other country; there-
avoid shortages. Answer choice H may appear to
fore, we do not know whether or not Canada is the
be correct; however, the author states that even
world’s largest exporter. Likewise, the other answer
though transportation costs have been substantially
choices are not supported by the passage.
decreased due to new technology, importing liquid
natural gas “is still often uneconomical.” Answer 20. The best answer is J. In the paragraph, the vessel
choices G and J are beyond the scope of the in question is described as something that the liquid
passage. natural gas is injected into for transportation. It
does not make sense that a liquid would be injected
15. The best answer is C. According to the passage,
into a “process,” “source,” or “facility” for trans-
“Currently, Canada is the largest liquid natural gas
portation. Answer choice J, “container,” is the most
supplier for the United States.” Japan and China,
logical choice.
two countries in Asia, are providing competition in
attaining liquid natural gas.
PASSAGE III
16. The best answer is F. The first sentence of the third
passage states “One of the largest misconceptions 21. The best answer is B. Throughout the passage,
about liquid natural gas is that it is an abundant the author talks about the prevalence of the word
source of natural gas.” While the passage goes on “good,” further discusses several different mean-
to discuss the expense of creating new process- ings and methods for interpreting the word, and
ing facilities and prices making liquid natural gas suggests that there is no one specific denotation for
uneconomical, the only misconception mentioned the word “good.” The other answer choices are not
is the fact that liquid natural gas is an abundant supported by the context of the passage.
PRACTICE TEST 1 ANSWERS AND EXPLANATIONS 85

22. The best answer is H. As stated by the author, The goodness of the weather is not less “compli-
babies are so young and powerless (“this elemen- cated” than that of the knife; likewise, it does not
tary life has not yet acquired positive standards make sense that the goodness of the weather would
or measurement”), that they can only be judged be less “powerful” or “drab” than that of the knife.
in negative terms, “a failure to disturb.” Answer
choice G may appear to be correct; however, the 28. The best answer is J. The author states, “We must
“anxious mother” is still able to judge whether her have some respect or end in mind in reference to
baby has been good by what the baby did or did which the goodness is compared.” In other words,
not do—in this case, cry. in order to understand what “good” means, you
must know specifically what is being referred to as
23. The best answer is A. The author states that “we “good” before interpreting the definition of “good.”
employ the word or some synonym of it dur- Answer choice G may appear to be correct, but
ing pretty much every waking hour of our lives. the passage states that “good always means good
Wishing some test of this frequency, I turned ‘for;’” thereby asserting that the two are actually
to Shakespeare.” In simpler terms, the author is identical concepts.
asserting that we use the word “good” or some
form of it constantly, and he believed that the works 29. The best answer is D. Throughout the passage
of Shakespeare would provide a good test of this the author refers to the actual application of the
notion. In other words, if Shakespeare used forms word “good,” and that the word “must be use-
of the word “good” as often as the author predicted, ful for something.” This best supports answer
the author’s theory on use of the word would be choice D.
proven true. This best supports answer choice A.
30. The best answer is G. According to the author,
24. The best answer is H. The author writes “goodness goodness in life “attends all our wishes, acts, and
always has reference to something outside itself, projects as nothing else does, so that no estimate
and is measured by its performance of an external of its influence can be excessive.” The author fur-
task.” The author goes on to write, “The knife is thers this point by adding that every action we
good for cutting and the day for business . . . To be take is in hopes of achieving something good. In
bad or good implies external reference.” This best simpler terms, the author is saying that because
supports answer choice H. our pursuit of goodness has such a huge impact
on our lives, there is no way we could overes-
25. The best answer is D. The author describes a quote
timate or over-emphasize the influence this has
by Shakespeare’s Portia as being spoken “lucidly,”
on us. Answer choice F is incorrect because it is
and goes on to analyze and apply Portia’s quota-
the opposite of what the author is trying to say.
tion. The author does so in a positive light, thus
Answer choices H and J are beyond the scope of the
eliminating answer choices A and B. It does not
passage.
make sense that Portia’s quote was “enthusiastic,”
or excited. Answer choice D makes the most sense
within the context of the passage; “coherently”
means “logically and meaningfully.” PASSAGE IV

26. The best answer is H. When discussing the knife, 31. The best answer is A. At this point in the passage,
the author states, “Its goodness always has ref- the defense mechanisms of armadillos are being
erence to something outside itself.” Although the discussed. If tanks were strengthening their posi-
passage mentions cutting wood, the author never tions, they would be improving their safety and
says that a knife is good only if it cuts wood. increasing their level of protection from enemies.
Answer choices G and J are not supported by the Therefore, if armadillos “often scurry under thorn
context of the passage. bushes, rather like tanks strengthening their posi-
tions,” they are giving themselves better protection
27. The best answer is A. The author begins by against their predators. This best supports answer
discussing the goodness of a clear, tangible choice A.
object—a knife. The author then moves on to dis-
cussing the goodness of the weather—something 32. The best answer is J. The first paragraph states
“not so palpable.” Answer choice A, “apparent,” that armadillos look “far more awkward than most
makes the most sense. The goodness of an intan- animals.” The passage then goes on to describe the
gible thing, such as the weather, is not nearly armadillo as an “alien creature,” which suggests
as “evident or clear” as that of a tangible object. that the armadillo’s awkward appearance is what
86 PRACTICE TEST 1 ANSWERS AND EXPLANATIONS

makes it an alien creature. The other answer choices 37. The best answer is B. Although the passage
are not supported by the passage. mentions both opossums and farmers, neither are
identified as predators of the armadillo. Both dogs
33. The best answer is D. According to the passage,
and cars are acknowledged as predators of the
the armadillo is “safer than most animals who wan-
armadillo; however, the passage explicitly states
der the Texas roads” because its shell protects its
that “In addition to threats of being eaten by an
from predators. Answer choices A through C are
opportunistic predator, the armadillo must also
beyond the scope of the passage and, therefore, are
endure a more severe danger: automobiles.” There-
incorrect.
fore, automobiles are more dangerous to armadillos
34. The best answer is G. The author’s statement that than are any other predator. This best supports
“if chased into their burrows, they are able to arch answer choice B.
their armor against the burrow walls making them
nearly impossible to dislodge” suggests that the 38. The best answer is F. The passage states that,
armadillo curves its back against the burrow walls, “Texans see the armadillo as a pest, since they
wedging itself into the burrow. The other answer have a tendency to ruin corn by eating the parts of
choices are not supported by the passage. the plants which are low to the ground.” This sug-
gests that armadillos are damaging crops. The other
35. The best answer is C. Information in the passage answer choices are not supported by the passage.
indicates that “The nine-banded armadillo is the
only species of animal in which this remarkable 39. The best answer is C. According to the passage,
trait occurs,” which is speaking in reference to armadillos “are born fully-formed with their eyes
their ability to have four identical offspring emerg- open” so it makes sense that they would be able to
ing from the same egg. This best supports answer see. The other answer choices contradict statements
choice C. made elsewhere in the passage, or are unsupported
by the passage.
36. The best answer is H. According to the pas-
sage, most Texans feel the armadillo is a “pest” 40. The best answer is J. The only scientific name
when it destroys crops and other plants that are mentioned, Dasypus novemcincts, is directly de-
low to the ground; however, most Texans also see fined as being the name for the nine-banded
that armadillos provide “benefits” as well, such as armadillo. The other choices are similar species of
its eating harmful insects and aiding in medical armadillo, but not mentioned in reference to that
research. This best supports answer choice H. specific scientific name.
PRACTICE TEST 2 ANSWER KEY 145

ANSWER KEY

English Test
1. C 21. D 41. D 61. D
2. F 22. J 42. F 62. F
3. A 23. D 43. D 63. C
4. J 24. J 44. G 64. F
5. D 25. A 45. C 65. B
6. H 26. F 46. J 66. G
7. B 27. B 47. C 67. D
8. J 28. H 48. H 68. G
9. D 29. B 49. C 69. B
10. F 30. H 50. F 70. J
11. A 31. B 51. B 71. B
12. G 32. H 52. H 72. J
13. D 33. B 53. B 73. A
14. F 34. F 54. H 74. J
15. B 35. B 55. A 75. D
16. H 36. F 56. G
17. A 37. C 57. D
18. G 38. J 58. G
19. C 39. D 59. C
20. G 40. G 60. H
PRACTICE TEST 2 ANSWER KEY 147

Reading Test Science Reasoning Test


1. A 21. D 1. C 21. B
2. H 22. H 2. H 22. H
3. B 23. A 3. C 23. C
4. J 24. H 4. G 24. G
5. C 25. B 5. A 25. C
6. F 26. H 6. H 26. F
7. D 27. D 7. C 27. A
8. G 28. J 8. H 28. G
9. D 29. A 9. A 29. B
10. J 30. G 10. F 30. F
11. B 31. A 11. D 31. D
12. F 32. H 12. G 32. F
13. D 33. C 13. A 33. C
14. H 34. F 14. G 34. G
15. B 35. A 15. B 35. A
16. G 36. J 16. J 36. G
17. C 37. D 17. D 37. C
18. J 38. H 18. G 38. H
19. D 39. C 19. B 39. D
20. F 40. F 20. G 40. J
PRACTICE TEST 2 ANSWERS AND EXPLANATIONS 153

ANSWERS AND EXPLANATIONS

English Test Explanations

PASSAGE I
1. The best answer is C. The narrator was chosen to travel. The couple is not physically traveling to
“last spring,” which was in the past. Answer choice the business.
A is incorrect because the moment the narrator is
8. The best answer is J. This question tests your
referring to is relative to the time the narrator wrote
ability to spot relevance. Neither the underlined
the passage, not another time in the past. Answer
portion nor the information is answer choice G and
choices B and D can be eliminated because they
H add anything relevant to the sentence. Therefore,
are not past tense.
it would be best to omit the underlined portion and
2. The best answer is F. The sentence appropriately simply end the sentence with “another.”
uses the relative pronoun who to introduce the
clause that modifies couple; this sentence is correct 9. The best answer is D. Logically, the question is
as it is written. The pronoun who also functions as asking whether the speaker would like to live with
the subject of the clause. someone who speaks English or someone who does
not speak English. The term whether only requires
3. The best answer is A. The sentence represents one side of a two-sided situation (e.g. “I don’t know
a clear, complete thought that is grammatically whether she has a roommate,” not “I don’t know
correct. It is correct to begin a new sentence whether she has a roommate or lives by herself.”)
with speaking. You can eliminate the other answer
choices because either they create incomplete 10. The best answer is F. The word whom is an object
sentences (B), or are otherwise grammatically pronoun, meaning it will occur in object, not sub-
incorrect. ject, position. In this case, it is the object of find.
Remember, whom refers to Paolo, the logical direct
4. The best answer is J. The sentence indicates that object. This becomes clear if you reorder the clause:
eating breakfast with the host family was a routine “I was surprised to find Paolo playing one of my
action in the past. Answer choice J is best because favorite CDs on the stereo!”
it includes would, which suggests repeated eating
of breakfast with the couple. You can eliminate 11. The best answer is A. The essay maintains a pos-
answer choice F because it is written in the present itive, uplifting tone with regard to the speaker’s
tense. friendship with Paolo. The other answer choices
do not match the tone of the essay.
5. The best answer is D. The two clauses are unre-
lated and, therefore, you can connect the two 12. The best answer is G. The word talk takes a
separate ideas by using the word then. Answer prepositional object starting with about; you can-
choices B and C can be eliminated because they not divide the phrase with a comma. Answer choice
express a cause-and-effect relationship that does H is incorrect because modifiers like “my Brazilian
not fit with the sentence. friend” that come before proper names do not need
a comma.
6. The best answer is H. This question asks you to
place the apostrophe correctly in the underlined 13. The best answer is D. The sentence is used to
portion of the sentence. Answer choice H is best describe a faulty or incomplete first impression,
because the extended family is that of both parents then uses but to introduce a revision to it. Such
together. This is made even clearer by the subject clauses introduced by a subordinating conjunction
they in the following sentence. are offset from the first clause of the sentence with
a comma.
7. The best answer is B. Answer choice B correctly
uses the infinitive form to tend to explain why 14. The best answer is F. The speaker is summa-
the host couple must travel. Choice C is incor- rizing his trip into one important lesson. Answer
rect because inflected forms like tended require an choices G and H do not represent how meaningful
expressed subject. Choice D is incorrect because the lesson is to the speaker and answer choice J is
the to that follows is realized as a preposition linked awkward as a modifier of lesson.
154 PRACTICE TEST 2 ANSWERS AND EXPLANATIONS

15. The best answer is B. The essay describes an cannot be placed after belies; therefore, answer
enjoyable friendship between people of two dif- choices F and H can be eliminated. Answer choice
ferent nationalities that begins in the context of G would create a sentence fragment, and can be
a foreign culture. Answer choices A and C have eliminated.
a negative tone that does not match the rest of
25. The best answer is A. The verbs must agree with
the essay. Answer choice D can be eliminated
the subject refrigerator. Answer choice D is not
because it is not relevant to the passage.
appropriate when describing a single refrigerator
case, and can be eliminated. Answer choice B does
PASSAGE II not make sense. Answer choice C is not inflected
16. The best answer is H. The clause subordinate to for tense.
apartment is I’m renting and cannot be divided by
26. The best answer is F. This sentence provides a
a comma. The sentence up to the word renting is an
logical transition between the previous sentence,
adverbial, describing the location of the restaurant,
which is about building repair, and the following
so it must be followed by a comma.
sentence, which is about how the food is enough to
17. The best answer is A. The paragraph is written sustain the charm of the restaurant.
in present tense. Answer choices B and C are past
tense, and imply that the restaurant is not located 27. The best answer is B. Many is the appropriate
there anymore. Answer choice D implies that the adjective for “A lot of people” from the previous
restaurant is not located there yet. sentence. It emphasizes the disparity between the
narrator’s knowledge of regular customers’ faces
18. The best answer is G. The prepositional phrase and the sandwiches they eat. Answer choice A can
introduced by in modifies fountain and cannot be be eliminated because while much can apply to
divided by a comma. Answer choice J can be mass nouns (e.g. much trash, much noise), it can-
eliminated because a semicolon should be used to not be used with countable nouns. For those, many
separate two independent clauses. must be used (e.g. many people, many marbles).
19. The best answer is C. Adjectives such as sinful Answer choice C is an adverb and would, there-
modify nouns, such as deliciousness, which can fore, be ungrammatical. Answer choice D does not
be modified by prepositional phrases, such as of make sense in context.
Joe’s Special Reuben. The other answer choices
28. The best answer is H. The phrase describes the
incorrectly pair adjectives, adverbs, and nouns.
classics, which by itself may be unclear. Answer
20. The best answer is G. This question tests your choice G can be eliminated because the information
ability to recognize redundancy in a sentence. New- is important to your knowledge of what the classics
comers implies people who have never seen his are. Answer choices F and J are not supported by
creations. The other answer choices are redundant. the passage.
21. The best answer is D. This question requires you 29. The best answer is B. The information is made
to determine the correct punctuation. No punctu- clear without the use of excess commas or weak
ation is necessary between nouns like sauerkraut words like makes and this place. The other answer
and gerunds like spilling, which modify them. choices are too wordy or have awkward pauses.
22. The best answer is J. Sentence 4 is about the menu
30. The best answer is H. This question requires you
in the window. It should be placed after the sen-
to determine the correct punctuation. No punc-
tence that describes the menu. Placing it anywhere
tuation is required between two noun phrases
else in the paragraph would cause the paragraph
conjoined with and.
not to make sense.
23. The best answer is D. This answer choice
PASSAGE III
describes how the unwelcoming appearance of
the restaurant does not reveal the truth about the 31. The best answer is B. The speaker is talking about
delicious food. The other answer choices can be Native Americans in the past; therefore, answer
eliminated because they are not relevant to the topic choices A, C, and D may be eliminated.
of the paragraph.
32. The best answer is H. A list of only two items does
24. The best answer is J. Belies is a transitive verb that not need punctuation to separate them; therefore,
takes what follows it as its direct object. A comma the other answer choices may be eliminated.
PRACTICE TEST 2 ANSWERS AND EXPLANATIONS 155

33. The best answer is B. The speaker is talking about the Mackinac Bridge in the context of Michigan,
the ferries in past tense; therefore, the other answer not the context of the all the world’s suspension
choices may be eliminated. bridges.

34. The best answer is F. The phrase as it is written is 45. The best answer is C. This essay focuses on
clear and concise. Answer choice G is wordy and how the bridge came to be built. Answer choices
answer choices H and J do not make sense in the A and B may be eliminated because the essay
sentence. does not describe the entire process of build-
ing the Mackinac Bridge. Answer choice D may
35. The best answer is B. The underlined sentence be eliminated because the topic of the essay
explains that a potential bridge was reasonable is not specifically the reason for building the
because the Brooklyn Bridge was a success. The bridge.
other answer choices are not supported by the
passage.
36. The best answer is F. To “take action on” some- PASSAGE IV
thing is a common idiomatic expression; therefore,
46. The best answer is J. The underlined sentence dis-
the other answer choices may be eliminated.
tracts from the intent of the paragraph, and should
37. The best answer is C. The second clause describes be omitted. The other answer choices also include
something that stands in opposition to the first information that is irrelevant to the topic of the
clause; that is, ferry service was resumed, but essay.
something caused it to stop again. The other answer
choices would not link the two clauses correctly. 47. The best answer is C. The narrator is speaking
about his past. Answer choice A is ungrammatical
38. The best answer is J. The possessive form of “it” is and wordy. Answer C differs from B in that it omits
“its,” without the apostrophe; answer choice H may whom, an unnecessary element. Answer D may be
be eliminated. Here, “its” refers to “ferry service eliminated because the sentence would not make
between peninsulas,” which is singular; therefore, sense.
answer choice G may be eliminated.
48. The best answer is H. This question requires you
39. The best answer is D. The sentence as it is writ- to determine the correct punctuation. No punctua-
ten is a fragment. Simply eliminating “that” after tion may come between a noun phrase (box in his
“however” corrects this problem. The word “how- room) and the gerund modifying it (brimming).
ever,” when used in the middle of a sentence must
be offset with commas. 49. The best answer is C. The simple plural past
tense is correct here. No piece is a singular sub-
40. The best answer is G. The preceding sentence
ject, and cannot be referred to by answer choice A.
shows the significance of the bridge. Answer
Answer choice B would not make sense in the
choices H and J can be eliminated because they are
sentence.
irrelevant to the topic of the essay. Answer choice
F can be eliminated because the sentence does not 50. The best answer is F. In Paragraph 2 the author
explain how the bridge was built. analyzes the arguments in favor of electronic enter-
41. The best answer is D. The elements in the sentence tainment and then proposes his counterarguments.
are ordered logically. The other answer choices sep- Answer choice G can be eliminated because it
arate elements from each other with use of commas, is irrelevant to the author’s argument. Answer
which make the sentence less clear. choice H is unsupported by the passage and answer
choice J does not provide a logical transition from
42. The best answer is F. The other answer choices the previous paragraph.
would not make sense with the simple present form
“is” used in the sentence. 51. The best answer is B. In the sentence, but does
not correctly conjoin dynamic and engaging, which
43. The best answer is D. This question tests your abil- are closely related. The other answer choices are
ity to recognize redundancy. Highway drivers and equivalent to the underlined sentences and are
travelers in this sentence essentially mean the same acceptable.
thing. The other answer choices are redundant.
52. The best answer is H. When placed after Sen-
44. The best answer is G. This answer choice does tence 7, the sentence provides a logical transition
not fit with the topic of the essay, which is to the subsequent paragraph. Answer choices F and
156 PRACTICE TEST 2 ANSWERS AND EXPLANATIONS

G can be eliminated because their placement would 62. The best answer is F. The clause headed by which
cause the paragraph to be illogical. The assertion provides descriptive detail about why spyware is
in answer choice J is false, and may, therefore, be a threat. The remaining answer choices are not
eliminated. grammatical.
53. The best answer is B. This question tests your abil- 63. The best answer is C. The writer sets the computer
ity to recognize redundancy in a sentence. Answer virus in opposition to spyware, to show how the
choice B is the best answer because it avoids two are unlike each other. The other answer choices
redundancy. Answer choices A, C, and D would compare spyware with a computer virus, which is
introduce redundancy. not the intent of the writer.

54. The best answer is H. The answer choice cor- 64. The best answer is F. The best way to answer
rectly modifies weekend. The narrator intends to this question is to try the answer choices in
describe card shows that occurred multiple times the sentence. Here, often coordinates well with
over multiple weekends. Answer choice F can be usually. Answer choice H can be eliminated
eliminated because it does not accurately convey because always does not make sense with but. The
what the narrator intends to describe. other answer choices would not make sense in
context.
55. The best answer is A. The sentence describes a
65. The best answer is B. However, when used in the
habitual action in the past. Answer choice C can
middle of a clause is set apart with commas. The
be eliminated because it is present tense. Answer
other answer choices can be eliminated because
choice B can be eliminated because we does not
they don’t follow this rule.
make any sense in the sentence.
66. The best answer is G. The paragraph describes
56. The best answer is G. Adverbials like too are some harmful spyware programs. This sentence
separated from the main clause by a comma. The effectively introduces the topic and links Para-
gerund encouraging modifies activity, so too must graphs 2 and 3.
be followed by a comma as well.
67. The best answer is D. This is clear, concise, and in
57. The best answer is D. This answer choice elim- active voice. The other answer choices are wordy.
inates the redundancy. The word screen does not
68. The best answer is G. Answer choice G is the
need to be repeated because the reader already
only choice that avoids redundancy. Detection util-
knows that it is the video screen that silences the
ities suggest that the utilities detect spyware. It is
spectators.
unnecessary to restate the fact that spyware is being
58. The best answer is G. This answer choice is con- detected.
sistent with the narrator’s use of the second person 69. The best answer is B. The verbs must agree with
from the preceding sentence. Answer choices F, H, the plural subject Detection utilities. Answer choice
and J can be eliminated because they do not use B is the only answer in which scan and remove
second person. correspond with the plural subject.
59. The best answer is C. The best way to answer this 70. The best answer is J. The writer is using present
question is to try the answer choices in the sentence. tense. Answer choices G may be eliminated
Answer choice C is the best because only adverbs because it is in past tense. Computer is the logi-
modify adjectives. cal object of protect, and therefore, answer choices
60. The best answer is H. The narrator believes H and F can be eliminated.
video games do not teach the skills card collecting 71. The best answer is B. This is the only answer
teaches. Showing this in the passage indicates the choice that provides a specific sound web surfing
author finds card collecting superior to electronic habit. Answer choice A is not specific enough to
entertainment. provide you with an example of a habit. Answer
choices C and D are irrelevant to the argument the
PASSAGE V writer is making.
61. The best answer is D. This question requires you to 72. The best answer is J. Negative elements appear at
determine the correct punctuation. In the sentence, the beginning of imperative clauses. Never is neg-
wait takes a prepositional object headed by until, ative, and therefore, would fit best at the beginning
so punctuation must not separate them. of the sentence.
PRACTICE TEST 2 ANSWERS AND EXPLANATIONS 157

73. The best answer is A. The sentence begins with an G, and H can be eliminated because if the sentence
“If” clause and ends with a clause describing the were placed in one of those paragraphs, it would
result. These two must be separated by a comma not support the arguments it followed.
and the subject must be repeated. The other answer
75. The best answer is D. Though the author might
choices may, therefore, be eliminated.
know how to protect computers from spyware,
74. The best answer is J. The sentence would provide he makes no reference to its programming or the
a logical conclusion to the essay. Answer choices F, ethical issues that surround it.
PRACTICE TEST 2 ANSWERS AND EXPLANATIONS 165

Reading Test Explanations answer choices are not supported by details in the
passage.
PASSAGE I
8. The best answer is G. Whenever Jake brings up
1. The best answer is A. Jake makes this state- problems with Katherine, she seems genuinely
ment in response to Katherine telling him that she concerned about Max’s well-being. However,
sometimes has to “throw something together at the because she works so much and has so many other
last minute.” This suggests that it does not hap- things to get done, her attention is often diverted
pen often and is unintentional. The other answer away from Max. The other answer choices are not
choices are not supported by the passage. supported by the context of the passage.
2. The best answer is H. During the conversation, 9. The best answer is D. When Jake pulled
Katherine says, “It’s just that I work two back-to- Katherine aside to discuss the lunch issue, he
back jobs every night,” and later, “Things are just indicated that Max’s lunch is a concern of his,
a little hard for us right now.” You can infer that eliminating answer choices A and C. Although
Katherine is working hard and barely getting by. answer choice B may appear to be correct, the
word “incident” implies a one-time occurrence.
3. The best answer is B. When Jake says,
Max was consistently bringing unhealthy lunches
“Katherine, Maxwell needs to have a healthy
to camp, therefore answer choice D, “problem,” is
lunch,” he indicates that Max is currently not eat-
correct.
ing well. This example has nothing to do with how
well Katherine takes care of her son, but rather 10. The best answer is J. The passage states that
highlights one of the problems she is having in Jake thought of the Sandwich Club because he
taking care of her son. The other answer choices needed to come up with a solution to help Max
all make reference to her trying in some way. eat healthier lunches. The last paragraph also men-
tions the fact that once Max moved away, the
4. The best answer is J. The word predicament
Sandwich Club ended. These facts best support
is used to indicate Jake’s problematic situation
answer choice J. The other answer choices are
with Max’s lunches. Based on the context of
beyond the scope of the passage.
the paragraph, his situation is somewhat difficult.
Therefore, predicament most nearly means “chal-
lenge,” which refers to a difficult task. The other
PASSAGE II
answer choices are not supported by the context
of the passage. 11. The best answer is B. The passage states that
“None but the fearless and inventive, the most
5. The best answer is C. Throughout the passage,
resourceful and curious, would dare to undertake
Jake is never referred to as either Max’s best friend
such a venture,” which clearly suggests that this
or his brother, but Jake does know Max; there-
was a challenge. The passage later goes on to say
fore, A, B, and D should be eliminated. He is
that “All of the wonders of those states in the
noted as being Max’s camp counselor, and has a
West are, in part, the result of this expedition.”
good influence on him by encouraging and allow-
This implies that western expansion was impor-
ing Max to consume healthier meals. Therefore,
tant. Answer choice A may appear to be correct;
answer choice C is correct.
however, the passages tells us that fur traders and
6. The best answer is F. At the end of the pas- trappers had traversed the wilderness to reach the
sage, Jake remarks that the club lasted for two west prior to Lewis and Clark’s journey.
summers, and then Max moved away. This best
12. The best answer is F. The passage discusses
supports answer choice F. Although answer choice
many aspects of the Lewis and Clark expedition,
J may appear to be correct because it is implied
and mentions several characteristic of the country,
that Max enjoyed the meetings of the Sandwich
describing to the reader that the expedition led to
Club, the author never defines these meeting as
“the unknown lands west of the Mississippi” and
Max’s favorite part of camp.
that “The path they were to carve out would be the
7. The best answer is D. At several points in the first of its kind.” Answer choice H may appear to
passage, Katherine refers to how hard it is to find be correct, however, the passage mentions that
enough money for everything, and how much she the men on the expedition were volunteers for
has to work to make ends meet. This suggests that “North Western Discovery,” not the “Northwest
Katherine and Max have little money. The other Passage.”
166 PRACTICE TEST 2 ANSWERS AND EXPLANATIONS

13. The best answer is D. The passage states that, with indigenous people, answer choice J. The
“The settlers who came after Lewis and Clark other answer choices are not supported by details
went forward with blind-devotion knowing then in the passage.
that it could be done.” This suggests that the Expe-
19. The best answer is D. As stated in the passage,
dition gave other people confidence that they, too,
the men set out on three boats, “all of which
could cross the United States because they knew
were moved by sails, towropes, poles, or oars.”
that it had already been done by other travelers.
These methods involve man power, wind power,
Answer choice A is incorrect because the pas-
or rowing power. Although the passage indicates
sage explicitly states that after Lewis and Clark’s
that steam power eventually replaced the boats,
historic journey, many people traveled by land.
nowhere does it mention that steam power was
Answer choices B and C are beyond the scope of
used on the Expedition.
the passage.
20. The best answer is F. According to the passage,
14. The best answer is H. The passage explicitly
“After receiving wilderness training in Washing-
states that the Lewis and Clark expedition was
ton D.C., Meriwether Lewis set out . . .” This
intended to “find the fastest water route across
indicates that Lewis was the only member of the
North America.” In addition, the passage states
Expedition to have received wilderness training.
that previous to this journey, “virtually no one
All of the other answer choices are mentioned
had attempted to cross the stretch of land between
explicitly in the passage as having been done by
the mighty Mississippi and the vast Pacific Ocean
all members of the Expedition.
using only water routes.” Although Lewis and
Clark did catalogue new species of animals on
their journey—answer choice G—this was not PASSAGE III
their primary objective in traveling across the
United States. 21. The best answer is D. Although the passage
discusses Porter’s renowned ability to candidly
15. The best answer is B. The second paragraph address artists’ works, illustrates the influence
states that, “Months later, in May, the party gath- that several famous artists had on his works, and
ered in St. Lewis. The forty some men were to assesses his unusual methods of painting and cri-
travel . . .” Answer choice B best fits the context tiquing artwork, none of these are the main focus
of the paragraph. The previous statement does not of the passage. Therefore, answer choices A, B,
support the idea that party means “a celebration,” and C can be eliminated. The passage gives a brief
“a segment of the population,” or a “meeting to summary of Porter’s life and discusses all of the
discuss business,” so answer choices A, C, and D above topics as points that shaped his career. This
can be eliminated. best supports answer choice D.
16. The best answer is G. In the third paragraph, the
22. The best answer is H. At the end of the passage,
men on the expedition are told to have written
the author states that it is sad that Porter “is still
of their “health and high spirits.” The statement
virtually unknown outside of art circles,” and that
is also made that the men were “all eager to
“This remarkably insightful, articulate, creative
explore,” in spite of the potential dangers they
individual needs to be discovered by the common
faced in their long journey. This best supports
man and revered for his continuing influence on
answer choice G.
the artists of today.” Clearly, the author thinks
17. The best answer is C. It is reasonable to infer that very highly of Porter and his works and believes
the word “forge,” as it is used in the passage – “the that he deserves to be honored (“revered”) for
travelers continued to forge west in search of an his influence on today’s artists. These details
efficient trade route using only the rivers”–refers best support answer choice H. Answer choice
to advancing on a path or journey, answer choice F is incorrect because the author is clearly not
C. The other answer choices do not make sense “detached,” or indifferent. Answer choice G is
within the context of the passage. incorrect because the author is not merely “tol-
erant,” or just able to withstand Porter. Answer
18. The best answer is J. The fourth paragraph men-
choice J is incorrect because the author obviously
tions that “the Sioux and the Blackfeet tried
does not “abhor,” or hate, Porter and his works.
to impede the group’s progress on more than
one occasion.” This suggests that the Expedition 23. The best answer is A. As stated in the passage,
would likely face danger in the form of conflict “what made Porter so famous was his knack for
PRACTICE TEST 2 ANSWERS AND EXPLANATIONS 167

responding directly to an artist’s work.” Answer 29. The best answer is A. As stated in the pas-
choices B and D are incorrect because the pas- sage, “Between the years 1931 and 1932, Fairfield
sage states that those are the things that Porter spent the majority of his time in Italy learning
did NOT do, for he found criticisms based on to appreciate and critique the works of the great
those criteria to be insignificant and meaningless. Renaissance painters. His training came from
Answer choice C is beyond the scope of the pas- both . . .” This suggests that Porter was continu-
sage; Bernard Berenson’s influence on Porter’s art ing his training as an art critic, answer choice A.
critiques is not discussed. The other answer choices are outside the scope of
the passage.
24. The best answer is H. Porter’s personal philoso-
phy regarding his paintings was that they should 30. The best answer is G. The passage preceding this
be “personal, emotional, and representative of its phrase discusses Porter’s fame as an acclaimed art
subject, while at the same time be boldly col- critic. The passage then goes on to state that, “The
orful, expressive, and generally abstract.” This other side of his fame, his uncommon approach
description of his works supports all of the answer to painting, is just as important to an understand-
choices except “trite,” answer choice H, which ing of Fairfield Porter’s contributions to the world
means “ordinary or dull.” of art.” This contrast indicates that Porter was
famous both for his criticisms of art, as well as his
25. The best answer is B. The first paragraph of the artwork itself–answer choice G. Answer choice F
passage simply tells how the author first came may appear to be correct, however, the passage
to meet Fairfield Porter, and provides a smooth does not ever define Porter’s level of fame as an
transition into the life of Fairfield Porter. Answer artist versus his level of fame as an art critic.
choice A may appear to be correct; however, the
author simply tells the reader how he came across
PASSAGE IV
Porter; there is no overview of what is to come
in the passage. Answer choices C and D are not 31. The best answer is A. The passage states that,
supported by information found in the passage. “without dark matter, there are many cosmolog-
ical phenomena that are difficult to explain.” In
26. The best answer is H. The passage states that, the context of the sentence, it would make the
“Fairfield Porter, despite being remarkably intel- most sense that without dark matter, there would
ligent, appeared to be lacking any natural artistic be many “occurrences” or incidents that would be
talents.” Therefore, his high intelligence level was difficult to explain. Answer choice B may appear
not correlated at all to his level of artistic ability. to be correct; however, the passage does not indi-
This best supports answer choice H. Porter’s intel- cate the nature of the phenomena and whether or
ligence is not discussed in terms of the criteria not they are problematic.
listed in answer choices F, G, and J.
32. The best answer is H. The passage states that,
27. The best answer is D. The author states that, “Dark matter . . . is not readily observable because
“This remarkably insightful, articulate, creative it does not refract light or energy directly. Its exis-
individual needs to be discovered by the com- tence can only be deduced because of the effect
mon man and revered for his continuing influence that it has on surrounding matter. In fact, some
on artists today.” The author is clearly praising members of the scientific community have argued
Porter and his continuing influence. It makes the that dark matter does not actually exist.” This best
most sense that the author believes that Porter supports answer choice H. Answer choice F is
should be “honored” for his continuing influence incorrect because the evidence for the existence of
on artists today. The other answer choices do not dark matter is its effect on surrounding matter; this
make sense in the context of the paragraph. discussion of dark matter’s effect on surrounding
matter also eliminates answer choice J. Answer
28. The best answer is J. It is stated in the passage choice G is not supported by details in the passage.
that Porter was primarily an art critic until “1961,
when he decided to pursue a full time painting 33. The best answer is C. As stated in the passage,
career.” This was approximately 30 years after he “It has been asserted that not only does dark mat-
returned from Italy, as he spent time between the ter exist, it may also be responsible for the Milky
years 1931 and 1932 in Italy learning to appreciate Way’s unusual shape.” The passage then goes on
and critique artworks. The other answer choices to discuss the way in which dark matter probably
are not supported by details in the passage. affects the shape of the Milky Way. Answer choice
168 PRACTICE TEST 2 ANSWERS AND EXPLANATIONS

B may appear to be correct; however, the passage for dark matter. It is not stated anywhere in the
does not indicate that the Magellanic clouds have passage how dark matter was first discovered.
enormous mass. In fact, the passage explicitly
37. The best answer is D. The passage states that
states that “The interaction referenced involves
“they apparently believe that the results of their
two smaller galaxies near the Milky Way, called
studies cast doubt on some of the conventional
Magellanic clouds, moving through an enormous
theories of galaxy formation and manipulation.”
amount of dark matter.” Answer choices A and D
Answer choices A and C do not make sense in
are not supported by details in the passage.
the context of the sentence, because if a theory
34. The best answer is F. As stated in reference was “easily understood” or “strictly interpreted,”
to Romanowsky’s theory, “They point to the there would not be a lot of room for doubt to be
existence of several elliptical shaped galaxies sur- cast upon the theory. Answer choice B is incor-
rounded by very little dark matter as evidence that rect because if a theory was “formally disputed,”
dark matter is not, in fact, the cause of the warped opposing viewpoints would already exist on that
galaxies.” By showing galaxies that are similar in theory. That the result of these studies cast doubt
shape to the Milky Way but NOT surrounded by on some “generally accepted” theories makes the
enormous amounts of dark matter, the theory illus- most sense; therefore, answer choice D is correct.
trates that dark matter may not affect the shape and 38. The best answer is H. There is nothing in the
formation of galaxies, answer choice F. Answer passage to indicate with certainty that dark matter
choice G may appear to be correct; however, the has no influence on surrounding celestial bod-
passage explicitly states that Romanowsky’s the- ies. Although Aaron Romanowsky suggests that
ory is not intended “to conclude that dark matter dark matter is not responsible for warped galax-
does not exist.” ies, there is no discussion in the passage to show
that he believes dark matter has no influence
35. The best answer is A. The last paragraph states
whatsoever on galaxy shape or other cosmolog-
that “the movement of [the Magellanic] clouds
ical phenomena. The other answer choices are
through the dark matter seems to create a wake
all mentioned in the passage as scientific theories
that enhances their gravitational influence on the
regarding dark matter.
Milky Way.” Earlier in the paragraph, when dis-
cussing the Magellanic clouds, the passage states 39. The best answer is C. As stated in the passage,
that when a cloud moves through dark matter, dark matter is “a substance that is not readily
it “enhances the gravitational pull that the two observable because it does not refract light or
Magellanic clouds could have on the Milky Way energy directly.” This best supports answer choice
and other surrounding bodies.” This best supports C. The other answer choices are not supported by
answer choice A. The other answer choices are details found in the passage.
not supported by details found in the passage.
40. The best answer is F. The passage describes
36. The best answer is J. The third paragraph dis- dark matter as surrounding and impacting galaxies
cusses the substance of Newton’s hypothesis and composed of common matter, such as the Milky
the consequences that his hypothesis had on Way. This best supports answer choice F. Answer
the existence of dark matter. When applying choice G is incorrect because whether dark matter
this hypothesis, it seems that dark matter must truly exists is still a topic of debate among scien-
exist: “something that is not easily observed must tists. Answer choice H is incorrect because the
be exerting the necessary force to create the passage does not provide the magnitude of the
warped shape of the galaxy.” This best supports amount of dark matter in the universe. Answer
answer choice J. Answer choice F may appear choice J is incorrect because dark matter is not
to be correct; however, it is simply stated that directly observable at all; its effect on galaxies is
Newtonian physics provide the strongest evidence the only proof of its existence.
230 PRACTICE TEST 3 ANSWER KEY

ANSWER KEY

English Test
1. D 21. B 41. A 61. A
2. H 22. J 42. H 62. J
3. B 23. B 43. B 63. C
4. J 24. H 44. G 64. F
5. A 25. D 45. C 65. B
6. J 26. H 46. F 66. H
7. A 27. A 47. C 67. A
8. J 28. F 48. J 68. H
9. B 29. C 49. A 69. C
10. J 30. G 50. H 70. F
11. D 31. A 51. B 71. B
12. F 32. G 52. F 72. J
13. B 33. B 53. A 73. C
14. G 34. F 54. H 74. G
15. A 35. A 55. C 75. D
16. F 36. H 56. J
17. D 37. D 57. B
18. H 38. F 58. G
19. D 39. C 59. C
20. H 40. J 60. H
232 PRACTICE TEST 3 ANSWER KEY

Reading Test Science Reasoning Test


1. B 21. B 1. D 21. D
2. H 22. H 2. F 22. F
3. A 23. C 3. C 23. A
4. H 24. G 4. H 24. F
5. B 25. B 5. A 25. B
6. F 26. H 6. J 26. F
7. B 27. D 7. B 27. D
8. H 28. F 8. F 28. G
9. D 29. C 9. C 29. D
10. J 30. J 10. H 30. H
11. A 31. B 11. D 31. B
12. H 32. H 12. H 32. H
13. B 33. C 13. A 33. B
14. J 34. H 14. G 34. F
15. A 35. D 15. D 35. A
16. H 36. F 16. J 36. H
17. B 37. B 17. C 37. D
18. F 38. H 18. J 38. F
19. D 39. B 19. D 39. A
20. H 40. G 20. F 40. J
PRACTICE TEST 3 ANSWERS AND EXPLANATIONS 237

ANSWERS AND EXPLANATIONS

English Test Explanations

PASSAGE I
1. The best answer is D. The contraction he’d begins 11. The best answer is D. No punctuation is required
a new, independent clause, which is clear and con- before the clause that begins with when; therefore,
cise as a separate sentence. Answer choices A and C the other answer choices may be eliminated.
would make the sentence ungrammatical. Answer
choice B would create a run-on sentence. 12. The best answer is F. It is correct to use the
infinitive to followed by the simple verb program.
2. The best answer is H. The verb insists takes a
clause headed by that as its object, which may not 13. The best answer is B. It maintains the lighthearted,
be separated from the verb with a comma. The other slightly tongue-in-cheek tone of the passage and
answer choices incorrectly use commas. makes sense in the paragraph. The other choices
may be eliminated for being unrelated and straying
3. The best answer is B. The sentence should be in
from the general tone of the passage.
past tense, so answer choices A and C may be elim-
inated. Answer choice D would be ungrammatical 14. The best answer is G. Placing the sentence
in the sentence. between Sentence 1 and Sentence 2 provides a
4. The best answer is J. There is no cause–effect logical transition from the former to the latter. The
relationship described between this sentence and other answer choices would not accomplish this;
the preceding one, so the other answer choices may therefore, they may be eliminated.
be eliminated.
15. The best answer is A. The Sentence supports the
5. The best answer is A. Unlike the other answer assertion that Mike is profiting from cable ser-
choices, answer choice A describes the primary rea- vice. Answer choices B and D are not supported
son Mike would have wanted cable at the particular by the passage. Answer choice C, while possibly
moment in the narrative described in the previous true, is not very specific. Moreover, Sentence 2 of
sentence. Paragraph 3 is not obviously humorous, as answer
choice C claims. For these two reasons, answer
6. The best answer is J. For the sake of clarity,
choice C may be eliminated.
paragraphs are best introduced without the use of
pronouns. Furthermore, answer choice G does not
agree with the singular noun cable service, to which
it refers.
PASSAGE II
7. The best answer is A. The underlined portion
16. The best answer is F. As it is written, the sentence
requires no changes. The word so introduces the
leads logically to the next sentence, which identifies
effect of a cause stated in the first clause. The other
a single example of a “form of music.”
answer choices reflect the inverse relationship and
would not make sense in context. 17. The best answer is D. The word nation refers to
8. The best answer is J. Nouns conjoined with and a single nation and must be in possessive form
may not be separated with a comma, as in answer in order to modify rich ethnic diversity. Answer
choices F and G, nor may they be separated from the choice B would create an awkwardly long chain of
noun with which they form a compound (channels), adjectives before the noun diversity.
as in answer choices G and H.
18. The best answer is H. This question tests your
9. The best answer is B. This list sets the events of ability to express the idea clearly and concisely.
the conversation in correct chronological order. The The sentence in answer choice H is the most clear
other answer choices are not as clear and concise. and concise among the answer choices.
10. The best answer is J. The sentence describes a 19. The best answer is D. The phrase in its heyday
completed event in the past; therefore, the other from 1900 to 1918 is an appositive, which must be
answer choices may be eliminated. separated from the clause with commas.
238 PRACTICE TEST 3 ANSWERS AND EXPLANATIONS

20. The best answer is H. Clauses beginning with 30. The best answer is G. The essay provides a con-
but must be separated from the main clause with cise introduction to the birth of ragtime and its most
a comma. Only answer choice H uses the correct famous composer, which constitute a good intro-
punctuation. Answer choice F incorrectly uses the duction to the “history and development” of the
semicolon. Answer choice J omits but, a critical musical genre.
element in the sentence.
21. The best answer is B. The “detail that is elaborated PASSAGE III
on later in the paragraph” is the fame of a particular
31. The best answer is A. The underlined portion
ragtime player, Scott Joplin.
requires no change. It is in simple present tense
22. The best answer is J. The other answer choices and third-person plural form to agree with the
provide stronger reasons why Joplin’s mother subject, “few facts.” Answer choice B would incor-
would have “worked long hours” to afford a piano. rectly insert “that.” Answer choice C has the wrong
They indicate Joplin was an exceptional talent. aspect, which makes it very awkward. Answer
choice D is in third-person singular form.
23. The best answer is B. The subject pronoun who
is used to represent human beings in subordinate 32. The best answer is G. Any form of the word
clauses. Answer choice C is an object pronoun. “articulate” is awkward and unnecessary. Answer
Answer choice D is a relative pronoun used for choice G is in the correct form for the simple
non-human things. subject “information” and is clear and concise.

24. The best answer is H. This detail offers an expla- 33. The best answer is B. Answer choices C and D
nation of why Joplin, though raised in Texas, found may be eliminated for their wordiness and awk-
fame in Missouri. wardness. Answer choice B reflects the use of “of”
that is like “from” in how it introduces one’s native
25. The best answer is D. The sentence needs a main land: man of the North, child of the mountains, etc.
clause, which must begin with a subject. The other
answer choices create sentence fragments. 34. The best answer is F. It is reasonable to assume
that the mother of someone born around 1770
26. The best answer is H. When a chain of adjec- would be deceased, so the past tense is appropriate
tives modifies a noun in a related way, they are here. This eliminates answer choice J. A complete
punctuated like a list, e.g. “soft, furry, and lovable sentence is required following a semicolon as it
rabbit.” When the adjectives modify the noun each is used here, so answer choices G and H may be
in a unique way, as in this phrase, they are not eliminated.
punctuated.
35. The best answer is A. The notion of respect is
27. The best answer is A. The sentence, as it is writ- a valuable detail, so answer choice D may be
ten, ties together the subject of Scott Joplin and eliminated. Among the other answer choices, only
the broader subject of ragtime music. The other answer choice A is an appropriate adjective to
answer choices emphasize the Maple Leaf Rag, modify “Cherokee chiefs.”
which would be left dangling at the end of the
paragraph. 36. The best answer is H. The word “alleged” is simi-
lar in meaning to “supposed.” This is an example of
28. The best answer is F. This sentence describes a word commonly used as an adverb, “allegedly,”
appealing characteristics of Joplin’s music, which than as an adjective (e.g., “The defendant allegedly
supports the assertion made in Sentence 1, that robbed a bank”). Answer choice H is the only one
“Joplin is the most famous figure in ragtime his- to cast any doubt on the truth of the claims made
tory.” Sentence 6 makes a similar assertion, but about Sequoia.
the sentence concerns only one piece specifically,
37. The best answer is D. This sentence connects the
at the time of its creation. On the other hand,
preceding sentence, which simply mentions the
answer choice F, like Sentence 1, speaks to modern
belief that Sequoia invented a writing system, with
impressions of Joplin and his work.
the rest of the paragraph, which details the devel-
29. The best answer is C. It uses more descriptive opment of the writing system. It describes why
language than the other answer choices, avoiding Sequoia chose to create the writing system in the
nonspecific, common words. first place.
PRACTICE TEST 3 ANSWERS AND EXPLANATIONS 239

38. The best answer is F. The underlined portion It is consistent with the reminiscent tone of the
requires no changes. Answer choices G and H passage.
may be eliminated first because a clause intro-
49. The best answer is A. Answer choices B and C
duced by “that” would not follow a comma as it
may be eliminated because a complete sentence is
is used here. Answer choice J may be eliminated
required following the semicolon. Answer choice D
for awkwardness.
may be eliminated because “distant” should modify
39. The best answer is C. All choices with the pro- “relatives.”
noun “it” may be eliminated because they would
wrongly refer to “the English language,” the closest 50. The best answer is H. The only word that is not
expressed noun. idiomatic and does not fit the context is “about.”

40. The best answer is J. The sentence should be in 51. The best answer is B. This answer choice forms
past tense, so answer choices G and H may be two closely related complete sentences, which may
eliminated. Answer choice J correctly modifies the be joined with a semicolon. The only other accept-
verb with an adverb, not an adjective, as in answer able option would be to use a period, but that is not
choice F. offered among the answer choices.

41. The best answer is A. The underlined portion 52. The best answer is F. The next two sentences go
requires no changes. Answer choice B may be elim- into detail about the contents of the shoeboxes.
inated because it would create a sentence fragment. Answer choice G contains an awkward sequence
Answer choices C and D would make the sentence of tenses. The third paragraph describes the activi-
ungrammatical. ties the author and his grandfather shared after his
walks; therefore, answer choices H and J may be
42. The best answer is H. “Bibles and other reli- eliminated.
gious materials” constitute “sacred texts,” which
are mentioned earlier in the sentence. 53. The best answer is A. The narrator goes on to
describe simple photographs of his ancestors’ home
43. The best answer is B. The sentence should explic- lives. The other answer choices are not supported
itly address the past, so the present tense answer by the passage.
choice A and the hypothetical answer choices C
and D may be eliminated. 54. The best answer is H. A photograph is a kind of
timeless representation of something, so the gerund
44. The best answer is G. Sentence 3 elaborates on the (which bears no tense) correctly follows pictured.
claim made in Sentence 1, so it should be raised one Here is another example: “At his memorial in
position in the paragraph. No other changes need to Washington, Lincoln is captured sitting pensively
be made to the paragraph, since it flows logically. in a chair.”
45. The best answer is C. The essay is very specific in
55. The best answer is C. The definite article “The” is
its scope, so it would not satisfy the requirements
required here because the “edges” have not been
of being an “introduction to the Cherokee nation.”
previously mentioned; therefore, answer choices
Answer choice D is incorrect because the passage
A and D may be eliminated. Answer choice B
indeed describes an important part of Cherokee
does not make sense in it use of the first-
history.
person “my.”
56. The best answer is J. The relative subject pronoun
PASSAGE IV who joins a noun and a clause to form a com-
plex noun phrase, “our French ancestors who first
46. The best answer is F. This is a well-formed clause
arrived on this continent.” Answer choices G and H
that does not require internal punctuation.
incorrectly use the object pronoun whom. Answer
47. The best answer is C. Introductory phrases of time choice F separates with a comma “French” from
do not require a preposition before them, so answer the noun it modifies, “ancestors;” therefore, it may
choice C is the most concise. be eliminated.
48. The best answer is J. This detail is an example 57. The best answer is B. When listing examples, it is
of an event that is characteristic of the close rela- appropriate to use “such as.” The remaining answer
tionship between the grandfather and the narrator. choices are awkward and not grammatical.
240 PRACTICE TEST 3 ANSWERS AND EXPLANATIONS

58. The best answer is G. The first-person plural sub- through the surface of the Earth, as mentioned later
ject pronoun “we” correctly refers to the narrator in the sentence.
and his grandfather; therefore, answer choices F
66. The best answer is H. Only answer choice H par-
and H may be eliminated. Answer choice G cor-
allels the uninflected form of the verb form that
rectly uses the past tense in agreement with the
comes before the conjunction or.
passage as a whole.
67. The best answer is A. It is better to write in active
59. The best answer is C. The construction became
voice than it is to write in passive voice. As it is
+ adjective + infinitive is common. Answer
written, the sentence is clear, concise, and in active
choice D violates this and is awkward. Answer
voice.
choice B is wordy, and “increasing” does not
make sense in context. Answer choice A is wrong 68. The best answer is H. The phrase is a complex
because one adjective modifies another; an adverb noun phrase that acts as the subject of the sentence;
is required, as in answer choice C. therefore, no punctuation is needed.
60. The best answer is H. This answer choice is a 69. The best answer is C. The subordinator “so”
clear and concise conclusion. The other answer correctly indicates the cause–effect relationship
choices unnecessarily repeat elements from earlier between the slowness of geologic processes and the
in the sentence and do not capture the essence of necessity to search for ancient evidence in order to
the passage as a whole. study them.
70. The best answer is F. The author is speaking about
all volcanoes, so the introductory phrase “In gen-
eral” is most appropriate. Furthermore, the other
PASSAGE V answer choices seem awkward when such a wide
range of time is given.
61. The best answer is A. Answer choice A is best
because it leaves “can” in place, which ties together 71. The best answer is B. The phrase “that is” is
the “understanding” mentioned earlier in the sen- a means to introduce a definition or clarification,
tence and what it equips scientists to do. Answer and is necessarily set apart from the sentence with
choice B and C may be eliminated for incorrectly commas.
using the comma.
72. The best answer is J. The adverb seriously cor-
62. The best answer is J. “Impending” means some- rectly modifies the verb take. The sentence as it
thing similar to upcoming and looming, which cor- is written incorrectly uses the adjective serious to
rectly describes the kind of eruptions that demand modify the verb take, while answer choices G and
warning. H use the wrong form of the verb take.
63. The best answer is C. The sentence describes the 73. The best answer is C. It is most clear and concise.
phenomenon of volcanoes generally, so the sim- The other answer choices are awkward in context.
ple present tense is appropriate. The other answer
74. The best answer is G. The phrase appropriately
choices use incorrect verb tense.
introduces two famous examples of predictions,
64. The best answer is F. The underlined portion which none of the other answer choices do.
requires no changes. Answer choice G is incorrect
75. The best answer is D. The first clause of the sen-
because it mismatches person between the subject
tence (before the comma) is unrelated to the second
and verb. Answer choices H and J are incorrect
clause, so the conjunction “and” is most appro-
because they misuse the comma.
priate. Answer choice B would be ungrammatical.
65. The best answer is B. Only answer choice B Answer choices A and C point to a relationship
correctly describes what led the magma to break between the clauses that does not exist.
PRACTICE TEST 3 ANSWERS AND EXPLANATIONS 247

Reading Test Explanations about who told him such silly stories, and that
his father “devoted that evening and several sub-
PASSAGE I sequent evenings” to explaining to Tom the true
history and present happenings of life in Northern
1. The best answer is B. The passage states that the
states.
narrator’s (Tom’s) feelings about life in the North
were “strengthened by Aunt Chloe, who said, 7. The best answer is B. In the second para-
“there wasn’t no gentlemen in the North no way.” graph, the narrator states that he “was born
Also, when confronted by his father, who asked in Rivermouth almost fifty years ago.” Answer
him where he had heard such inaccurate things choice A may appear to be correct, but the nar-
about the North, Tom replied, “‘Aunt Chloe, sir; rator is telling a story from his adult perspective
she told me.”’ This best supports answer choice B. about his boyhood.
2. The best answer is H. The author says that 8. The best answer is H. In the sixth paragraph, the
if the name of a new pupil struck Tom favor- narrator describes kicking Sam upon finding out
ably, he would, in turn, shake the student’s hand that his father wanted to move the narrator back
cordially. This has pleasant connotations since, to Rivermouth. In the last sentence of the sixth
again, Tom only cordially shook the hands of paragraph, the narrator states “as for kicking little
those whose names he saw favorably. Answer Sam, I always did that . . . when anything went
choices F, G, and J can be eliminated since they wrong with me.” The author takes out his nega-
all have negative connotations. Answer choice H, tive feelings on Sam by kicking him and thereby
“sincerely,” makes the most sense in the context abusing him, answer choice H.
of the passage.
9. The best answer is D. The narrator states
3. The best answer is A. In the first paragraph, the in the second paragraph that he was born in
narrator describes himself as “a real human boy, New England but moved to New Orleans when
such as you may meet anywhere in New England.” just an infant. In the next paragraph he states,
This implies that he is the same as any other boy “I had no recollection of New England: my earli-
found in New England at the time; therefore, he est memories were connected with the South;” and
was neither better nor worse behaved than other that even though he was born a Yankee, “hoped
boys his age. This supports answer choice A. nobody would find it out,” indicating that he
adapted to a Southern lifestyle to the point that his
4. The best answer is H. In the second paragraph,
Northern heritage was not obvious to anyone else.
the narrator describes meeting new classmates.
He states “such names as Higgins, Wiggins, and 10. The best answer is J. In the first paragraph the
Spriggins were offensive affronts to my ear,” narrator states that he was an “amiable, impul-
while the names “Langdon, Wallace, Blake, and sive lad,” meaning that he was friendly, yet
the like, were passwords to my confidence and fickle. The second paragraph details an exam-
esteem.” Therefore, Blake and Wallace would ple of his amicability—he was eager to introduce
be acceptable to him, but Higgins would not be himself to new students on the playground—and
acceptable. also his fickleness—if the boy had the wrong last
name, Tom was not interested in being his friend
5. The best answer is B. When first told by his father
anymore.
that he was going back to school in Rivermouth,
Tom resolved that he “would not be taken away
to live among a lot of Yankees!” Also, later in
PASSAGE II
the passage he reports feeling “terror at the bare
thought of being transported to Rivermouth to 11. The best answer is A. The author uses strong
school.” However, after his father talks to him language throughout the piece; for example, say-
about how life really was in the North, Tom “was ing there is “no more important” and “no more
no longer unwilling to go North.” This best difficult” knowledge than that of the self, and that
supports answer choice B. a consequence of a nation not understanding its
own being will result in “decay and death.” Since
6. The best answer is F. Tom’s father shows
impassioned means “with strong feeling,” answer
his patience and understanding through the
choice A is best.
manner in which he handles Tom’s ridiculous
misconceptions about moving North. The pas- 12. The best answer is H. The passage states that
sage states that Tom’s father asked him “calmly” it is important for a nation “to know itself,
248 PRACTICE TEST 3 ANSWERS AND EXPLANATIONS

understand its own existence, its own powers and vindicated means, in this context, “defended or
faculties, rights and duties, constitution, instincts, proved worthy of,” “justified” is the best answer.
tendencies, and destiny.” Later in the passage the
19. The best answer is D. The author asserts that
author states that by gaining an “appreciation of
the United States has “lived the naive life of a
its own constitution tendencies,” the nation will be
child,” but the Civil War has “forced it to pass
able to “understand its own idea, and the means
from . . . adolescence to . . . reflecting manhood.”
of its actualization in space and time.” This best
The passage later states that the country will gain
supports answer choice H.
“the manliness it has heretofore lacked.” This best
13. The best answer is B. The author is com- supports answer choice D.
paring America to a child, lacking the experi-
20. The best answer is H. It is indicated in the pas-
ence and knowledge that could be gained from
sage that a nation has many aspects of itself to
“severe trial.” Because of this, answer choice C
“understand and observe,” and if it does not, it
may appear to be correct, but once placed into
will “become lethargic and infirm . . . and end
context—America has lived the difficult life of
in premature decay and death.” Since lethargic
a child—it no longer makes sense. Naive means
means “lacking energy” and infirm means “weak
“lacking experience, wisdom, or judgment,” so
or debilitated,” the best answer choice is H.
the best synonym for naive in this context is
Answer choice J may appear to be correct if you
inexperienced.
did not read the question carefully and realize that
14. The best answer is J. The passage contains the the question is asking what will happen if the
statement, “The defection of the slaveholding nation does NOT become more aware of itself
States, and the fearful struggle that has followed and its role in the global community.
for unity and integrity, have . . . forced it to pass
from . . . adolescence to . . . manhood.” Because PASSAGE III
this is referring to the Civil War, or the secession
of the Southern states and the ensuing conflict, 21. The best answer is B. The passage states that
answer choice J is best. Faulkner began writing in Lafayette County,
Mississippi, where he was “immersed” in south-
15. The best answer is A. The first paragraph states ern lore. Because the sentence is describing
that, in response to the introduction of the maxim Faulkner himself, not his writing, it makes the
of “know thyself,” “there is for an individual no most sense that he was “submerged” in traditional
more as there is no more difficult knowledge, than southern lore. Within the context of the sentence,
the knowledge of himself.” The paragraph goes it does not make logical sense that Faulkner would
on to say that a nation is just like an individual be “depicted” or “interpreted” in southern lore,
in that respect, and that failure to understand its nor does it make sense that he was “related” in
existence will lead to its ultimate undoing. This southern lore.
best supports answer choice A.
22. The best answer is H. In paragraph one, the
16. The best answer is H. The author contends that author states “Faulkner produced literary works
“the citizen who can . . . do this will render them filled with emotional turmoil and unflinching
an important service and deserve well of his coun- honesty. His unique interpretation of history is
try.” Since revered means “praised,” this is the highlighted in the symbolism and imagery of his
best answer choice. writing.” These two sentences describe symbol-
17. The best answer is B. The author states that “with ism, honesty, and turmoil. Nowhere in the passage
time, patience, and good-will, the difficulties may is it written that Faulkner’s works were considered
be overcome, the errors of the past corrected, and to be malicious.
the Government placed on the right track for the
23. The best answer is C. The passage repeatedly
future.” This indicates that though there have been
makes reference to Faulkner’s “clever satire of
troubles and errors, the nation will eventually rec-
Southern characters,” and his willingness to face
oncile its past mistakes and have a better and
the “brutality and anguish that life can bring.”
brighter future, answer choice B.
These statements show his strong convictions.
18. The best answer is F. The passage claims that Other portions of the passage talk of his promi-
the United States is “preparing to take a new start nence as an author, such as winning the Nobel
in history” with its “national unity and integrity” Prize or mastering various literary techniques.
intact after the challenge of the Civil War. Since These sentiments best support answer choice C.
PRACTICE TEST 3 ANSWERS AND EXPLANATIONS 249

While answer choice A may seem correct as the scope. The other answer choices do not contain
passage talks of how his “Deep South roots greatly anything attributed to that novel by the passage.
influenced his writing,” this does not necessarily
29. The best answer is C. The end of the third
mean that he stayed true to those roots.
paragraph speaks of the connection between
24. The best answer is G. The passage discusses how Faulkner’s “characters and their counterparts in
Faulkner abhorred “rampant racism and abuse,” the real world.” This statement implies a link-
and later that he “condemns . . . obstructions to age between the characters within Faulkner’s
human freedom.” Together these statements carry work and real individuals who exist outside
the connotation of a strong dislike for injustice, of it. Answer choice C is best supported by that
which best supports answer choice G. information.
30. The best answer is J. The passage repeat-
25. The best answer is B. The passage talks of
edly discusses the literary devices, some of
how Faulkner “abhorred” racism, which indicates
which Faulkner invented, that are central to his
a strong dislike. It further speaks of how he
craft. He clearly understands what he is doing.
“refused to avoid painful or controversial issues,”
Additionally, he is described by the author as
and it states that he “condemns, and analyzes
being “his own harshest critic”; Faulkner “consid-
obstructions to human freedom and happiness by
ered many of his books failures because they did
examining racism.” Both of these statements
not live up to his high expectations.” This speaks
support the idea that he hated racism and worked
directly to his commitment to the craft of writing.
to expose it, along with other difficult and
controversial topics concerning life in the South.

26. The best answer is H. The fourth paragraph PASSAGE IV


begins by stating that Faulkner “was awarded the 31. The best answer is B. The passage begins by con-
Nobel Prize.” Later in the paragraph it mentions necting the character Yogi Bear and his pursuit
that many of his readers, when encountering his of picnic baskets to the idea of real bears gather-
“experimental literary styles,” “were more often ing food. It continues by stating that, “remarkably
than not inspired by his insightful analysis of enough, bears have been known to seek out food
the human spirit.” These statements reflect the from some unlikely sources” and later speaks of
author’s intent to emphasize the importance of another behavior in which “bears may even vary
Faulkner’s work and mastery of various styles. their sleep cycles.” These unique feeding manner-
The other answer choices are not supported by isms are the focus of the passage, and connecting
the passage. them to a cartoon character leads to a comic por-
trayal on television. The other answer choices are
27. The best answer is D. The first sentence of not supported by the passage.
the passage calls Faulkner’s literary devices
“revolutionary.” In the third paragraph the author 32. The best answer is H. Near the end of the third
speaks specifically of Faulkner’s literary style, paragraph it states that bears “maintain vast terri-
saying they included “brilliant symbolism and tories in order to obtain food from a variety of
exquisite dialogue.” At the end of that paragraph sources.” That behavior best supports the idea
the author talks of how one technique Faulkner that they are employing several methods so as
used, stream-of-consciousness, conveyed a char- to ensure that they have enough food, answer
acter’s thoughts “in a manner roughly akin to choice H.
the way the human mind actually works.” Along 33. The best answer is C. A close examination of
with references throughout the passage to the the second paragraph will reveal that although
sharp social commentary within Faulkner’s work, bears will feast on picnic baskets if necessary, it is
these statements suggest that the author feels they “on top of their usual diet of berries, insects and
were a new take on discussing social issues, best fish.” Of those three items, insects are the only
supporting answer choice D. available answer choice.
28. The best answer is F. Within the third para- 34. The best answer is G. In the third paragraph,
graph, The Sound and the Fury is referred to especially the line following the mention of an
as “the painful demise of a distinguished south- acute sense of smell, the passage states, “these
ern family.” This most closely matches the tragic sensory talents contribute to the bears’ remarkably
downfall of a respected family in both tone and high intelligence and curiosity, giving them the
250 PRACTICE TEST 3 ANSWERS AND EXPLANATIONS

ability to open closed containers if they believe their sleep cycles to allow for “feasting on road-
food is inside.” This connection between closed side garbage during the day.” Although the pas-
containers and a sense of smell implies that the sage mentions building up “fat stores,” it does
bears’ sense of smell is very accurate, or sharp. not suggest that general stores are potential food
Words such as small, reduced, or abnormal do not sources for bears.
fit with an idea of specificity.
38. The best answer is H. The third paragraph states
35. The best answer is D. The passage refers to bears specifically that the American black bear “has
as being intelligent and curious in the third para- color vision.” The other traits, an acute sense
graph, when speaking of their sensory abilities of smell, abnormal sleep patterns, and sensitive
contributing to their “remarkably high intelligence lips are used to refer to bears in general, but not
and curiosity.” The fourth paragraph states that mentioned in conjunction with a specific species
bears are “generally solitary.” Solitary does not of bear.
mean anti-social, as the passage then goes on
39. The best answer is B. The paragraph begins
to explain their feeding relationships with other
by stating that picnic baskets are eaten by bears
bears.
“on top of their usual diet,” and concludes by
36. The best answer is F. Throughout the passage saying that the main reason bears wander into
the author refers to the feeding habits of bears as campsites looking for food is that it is “gener-
being varied and discusses a variety of methods ally scarce in the early spring, and consequently
that they use to secure food, such as keeping “vast they will gladly indulge in any foods which are
territories,” “varying sleep cycles,” and “creating high in proteins or fats.” These two ideas encap-
a hierarchical order.” These behaviors are some- sulate the main idea of bears needing to sometimes
what outside the norm of typical animal behavior, add human food to their regular diets in order
so the author would likely call them resourceful. to maintain a healthy weight when food is hard
The other answer choices are not supported by the to find.
passage.
40. The best answer is G. When describing coopera-
37. The best answer is B. In the second paragraph, tion to procure food, the paragraph mentions that
“berries, insects, and fish” are mentioned as the “black bears will create a hierarchical order in sit-
usual bear diet. The end of that paragraph also uations where paths cross in pursuit of food, so as
speaks of campsites as potential feeding grounds. to assure that all animals remain adequately fed.”
The end of the third paragraph says bears vary This statement clearly supports the idea that black
bears act as a group, answer choice G.
PRACTICE TEST 4 ANSWER KEY 309

ANSWER KEY

English Test
1. D 21. B 41. C 61. D
2. F 22. F 42. G 62. J
3. B 23. A 43. A 63. B
4. F 24. G 44. J 64. F
5. C 25. A 45. A 65. C
6. J 26. G 46. J 66. G
7. C 27. D 47. D 67. B
8. G 28. G 48. F 68. H
9. A 29. A 49. A 69. D
10. H 30. J 50. J 70. G
11. A 31. D 51. B 71. B
12. J 32. F 52. H 72. G
13. B 33. A 53. D 73. D
14. H 34. G 54. F 74. F
15. D 35. B 55. D 75. B
16. F 36. H 56. G
17. B 37. D 57. D
18. H 38. F 58. H
19. D 39. B 59. A
20. F 40. H 60. J
PRACTICE TEST 4 ANSWER KEY 311

Reading Test Science Reasoning Test


1. C 21. B 1. D 21. D
2. J 22. H 2. J 22. J
3. B 23. B 3. B 23. C
4. G 24. G 4. H 24. G
5. D 25. B 5. D 25. A
6. G 26. J 6. G 26. H
7. D 27. A 7. D 27. B
8. H 28. H 8. H 28. J
9. A 29. A 9. A 29. C
10. H 30. J 10. F 30. G
11. B 31. C 11. B 31. C
12. H 32. G 12. H 32. J
13. D 33. B 13. C 33. A
14. H 34. J 14. G 34. J
15. B 35. C 15. C 35. C
16. G 36. F 16. G 36. G
17. C 37. B 17. D 37. B
18. J 38. G 18. J 38. H
19. A 39. D 19. B 39. A
20. F 40. J 20. H 40. G
PRACTICE TEST 4 ANSWERS AND EXPLANATIONS 317

ANSWERS AND EXPLANATIONS

English Test Explanations

PASSAGE I
1. The best answer is D. The sentence is clearest and use the past tense, so they are incorrect. It is never
most concise when the word back is inserted in correct to use the helping verb has with the sim-
place of the underlined portion. Answer choices A, ple past tense verb became, so answer choice H is
B, and C are either awkward or redundant. incorrect.
2. The best answer is F. This sentence is best as writ- 7. The best answer is C. If two adjectives modify a
ten. The word fairly is correctly used as an adverb noun in the same way, they are called coordinate
to further modify the adjective regular, which mod- adjectives. It is necessary to separate coordinate
ifies the noun trips. Answer choice G is not correct adjectives, such as warm and sunny with a comma.
because the word fair is being used as an adjec- You could also insert the coordinate conjunction
tive to describe the trips, when it should be used and, but you should not use any commas if you do
to modify the adjective regular. Answer choice H so; therefore, answer choice B is incorrect. A semi-
is incorrect because it reverses the words, thereby colon should be used to separate two related main
changing the meaning of the sentence. It is not clauses, so answer choice D is incorrect.
best to use regular because the context of the sen-
8. The best answer is G. The subordinating conjunc-
tence indicates that the trips were mostly between
tion because clearly indicates that the dogs are the
Las Vegas and New York, so answer choice J is
reason that the author cannot fly to Las Vegas. The
incorrect.
subordinating conjunction so is used to indicate a
3. The best answer is B. The context of the sentence consequence, and if it is used here, it creates an
indicates that first the author goes to Las Vegas, incomplete sentence; eliminate answer choice F.
“then” the author returns home to New York. It Answer choice H is a preposition phrase that does
is not appropriate to use “than” to indicate a pro- not correctly express the idea that having the dogs
gression in time, so answer choices A and C are prevents the author from flying to Las Vegas, so it is
incorrect. Answer choice D is wordy and is not as incorrect. While answer choice J expresses essen-
clear and effective as answer choice B. tially the same idea as answer choice G, it is wordy
and awkward and should be eliminated.
4. The best answer is F. The sentence is best as
it is written. It clearly indicates that because the 9. The best answer is A. It is necessary to use com-
author and her husband are retired, they have suf- mas to set off parenthetical expressions within a
ficient time to travel. Answer choices G and H are sentence. Because the phrase “both of which are
wordy and are not as clear and effective as answer shelter mutts” is not essential to the meaning of
choice F. Omitting the underlined portion would the sentence, it is considered a parenthetical, or
result in some ambiguity within the sentence: you non-restrictive phrase. A nonrestrictive phrase can
do not know for what the author and her husband be omitted from the sentence without changing the
have plenty of time. meaning of the sentence.
5. The best answer is C. According to the passage, 10. The best answer is H. The sentence is written
the fact that the author’s grandchildren live in in the present tense, so it is necessary to use the
New York combined with the fact that she has two present tense verb lies. In addition, the sentence
dogs makes it difficult for her to stay for any longer indicates that only one dog lies on a blanket on the
than three weeks in Las Vegas. These responsibili- floor.
ties require her to be at home in New York most of
11. The best answer is A. It is best to leave Sen-
the time. Therefore, it would not be viable for her to
tence 5 where it is in the paragraph. The paragraph
stay longer than a few weeks at a time in Las Vegas.
describes the behavior of the dogs as they travel
6. The best answer is J. The paragraph is written with the author, and the sequence of events is
in the present tense, so it is necessary to use the clearly indicated without changing the order of the
present tense verb has. Answer choices F and G sentences.
318 PRACTICE TEST 4 ANSWERS AND EXPLANATIONS

12. The best answer is J. The passage is written pri- 18. The best answer is H. The sentence is written
marily in the present perfect tense so it is necessary in the present tense, so it is necessary to use the
to use the present perfect form of the verb have present tense verb is. As it is written, the sentence
passed. The word past indicates “the time before suggests that the problem will be become evident
the present,” and is not a verb that suggests move- in the future, so eliminate answer choice F. Answer
ment. Therefore, eliminate answer choices F and choice G is incorrect because it uses the participle
H. Answer choice G is incorrect because it uses the becoming; answer choice J is awkward.
present participle passing, which suggests that the
19. The best answer is D. The sentence discusses the
action is currently taking place.
“movements and facial expressions” of “a dog,”
13. The best answer is B. The context of the sentence which means that you should use the singular
indicates that the author paid multiple tolls, so the possessive dog’s.
plural form of the word is correct. The tolls do
20. The best answer is F. The sentence is best as it is
not possess anything, so it is not necessary to use
written. Answer choice G is incorrect because he
an apostrophe; eliminate answer choices A and D.
participle confusing suggests that the dog’s actions
Answer choice C includes an extraneous comma, so
are not clear to the owner, which does not fit the
it is incorrect. Because the phrase “in Oklahoma” is
context of the sentence. Answer choice H is incor-
necessary to the meaning of the sentence, it should
rect because use of the preposition by suggests that
not be set off by commas.
the dog finds “waiting for a signal” to be confus-
14. The best answer is H. The preceding sentence ing, which is not supported by the context. Answer
reads, “I’m mentally preparing myself for a sun- choice J is incorrect for the same reason; in addi-
less New York sky during our week at home.” tion, it is not idiomatic to say “confused with”
This sentence and the final sentence of the passage unless you are making a comparison.
suggest that the author enjoys her visits to “sunny
21. The best answer is B. Since the sentence refers
Las Vegas” and is glad for the chance to escape the
to a single dog—the dog—it is correct to use the
“sunless New York sky.” The sentence reinforces
singular possessive its. As it is written, the sentence
the author’s decision to travel back and forth from
is unclear as to whom or what the lips belong. It
New York to Nevada.
is never correct to use the plural possessive its,’
15. The best answer is D. The tone and context of the because it is a singular pronoun. Therefore, answer
passage clearly indicate that it is a personal account choice C is incorrect. Answer choice D is incorrect
of the author’s travels to and from Nevada. There- because the sentence refers to a single dog.
fore, it would not fulfill an assignment to write a
22. The best answer is F. The paragraph describes sev-
travel article about Las Vegas, Nevada.
eral ways that a dog will communicate non-verbally
with its owner. The description of a relaxed dog
letting “its tongue loll out of its mouth” is another
example of this type of communication.
PASSAGE II
23. The best answer is A. This question tests your
16. The best answer is F. The indefinite pronoun one
ability to convey the intended idea clearly and
is used to indicate any of the possible people who
effectively. The simplest way to show that other
acquire a new puppy or dog. Because one is a col-
forms of body language exist is to say “can also
lective pronoun, it is considered a singular pronoun,
indicate.” The other answer choices are wordy
so the correct verb to use is “acquires.” You do not
and awkward and are not appropriate for standard
need the pronoun who in the sentence, so eliminate
written English.
answer choice H. It is not correct to use the pronoun
it refer to people, so eliminate answer choice J. 24. The best answer is G. The preceding sentence
describes how a dog owner might recognize when
17. The best answer is B. By using the article the,
his or her dog is being aggressive or fearful. Answer
the sentence does not clearly indicate to whom the
choice G offers a reason for this behavior.
pets belong. The context of the sentence shows that
the pets belong to the dog owners so it is correct 25. The best answer is A. The main focus of the para-
to use the plural pronoun their. The word “there” graph is additional forms of body language that a
indicates a place, so eliminate answer choice C. The dog might exhibit when attempting to communi-
word they’re is the conjunction of they and are, so cate. Therefore, the best concluding sentence will
eliminate answer choice D. refer to and generalize the main idea. Only answer
PRACTICE TEST 4 ANSWERS AND EXPLANATIONS 319

choice A does this. The other answer choices are Answer choice C is incorrect because you must use
too specific and offer additional forms of commu- a comma to separate an introductory clause from
nication. the rest of the sentence.
26. The best answer is G. You should use a comma 32. The best answer is F. The word where correctly
to separate elements that introduce and modify a indicates that most of the piñatas were made in
sentence. The phrase “While most dogs are capable China. Because China is a place, it is better to
of learning” is an introductory clause that should be say where than in, so answer choice G is incorrect.
followed by a comma. Answer choice F is incorrect Answer choice H is incorrect because it includes an
because a semicolon should be used to separate two extraneous comma. The coordinate conjunction so
independent clauses. Answer choices H and J are indicates that China is the reason that most piñatas
incorrect because it is necessary to use a comma were made to resemble animals, which does not
to separate an introductory clause from the rest of make sense.
the sentence. In addition, answer choice J creates
an incomplete sentence. 33. The best answer is A. The underlined portion
clearly and effectively indicates that the current
27. The best answer is D. The main focus of the custom of filling piñatas with candy or toys dif-
paragraph is on learning to understand a dog’s fers from the original custom of filling the piñatas
body language and using that knowledge to more with seeds. Answer choice B is incorrect because
effectively train the dog. The sentence that men- it includes the ambiguous pronoun they. Answer
tions offering a dog treats is least relevant—it choice C is redundant—it is not necessary to
does not add any information that is necessary use both standard and customary in the sentence
to the flow or structure of the paragraph. While because the words are synonyms. If you omit
Sentence 6 is not entirely unrelated to the topic, of the underlined portion, you lose the compari-
the answer choices, it is the least relevant to the son between the original custom and the current
paragraph. custom.
28. The best answer is G. According to the con- 34. The best answer is G. The sentence is written in
ventions of standard written English, you should the past tense so you must use the past tense verb
never separate a subject from its verb with any sort were kept. Answer choice H is incorrect because,
of punctuation. Therefore, it is incorrect to place while it uses the past tense verb kept, it creates an
a comma, a semicolon, or a colon between The incomplete sentence.
bottom line—the subject—and is—the verb.
35. The best answer is B. No commas are necessary
29. The best answer is A. The ACT rewards active
to set off a person’s name from a clause when the
voice. Therefore, it is better to use the verb com-
title of occupation precedes the name: for example,
municating as opposed to the noun communication
President Abraham Lincoln.
in this sentence; eliminate answer choices B and D.
It is idiomatic to say involved in, whereas involved 36. The best answer is H. The paragraph focuses
to is not correct in standard written English. specifically on the piñata and its use in European
30. The best answer is J. The ACT rewards active celebrations. The sentence that the author is con-
voice. Therefore, it is best to say to remember; the sidering adding does not offer anything relevant to
other answer choices are not as good because they the main idea of the paragraph.
are written in the passive voice. 37. The best answer is D. The phrase “that broke eas-
ily,” while not grammatically incorrect, is redun-
dant. Something that is fragile, by definition,
PASSAGE III breaks easily. Therefore, answer choices B and C
are also redundant, and it would be best to omit the
31. The best answer is D. You should use a comma to underlined portion.
separate elements that introduce and modify a sen-
tence. The phrase “While the history of the piñata 38. The best answer is F. The sentence is describing
is somewhat murky” is an introductory clause that an event as it might have taken place in the past.
should be followed by a comma. Answer choice A Therefore, it is appropriate to use the auxiliary verb
is incorrect because it creates an incomplete sen- would be with the past tense verb hung. It is not
tence. Answer choice B is incorrect because a semi- necessary to use any punctuation in the underlined
colon should separate two independent clauses. portion.
320 PRACTICE TEST 4 ANSWERS AND EXPLANATIONS

39. The best answer is B. The past tense verb dis- 47. The best answer is D. This sentence requires the
pensed effectively indicates that the tiny treasures use of the participle spending in order to accu-
came from the pot after it was broken. The phrase rately show that the spending takes place after the
dispensed with refers to getting rid of or doing watching, reading, and hearing of the advertise-
away with something, which does not fit the ments. The helping verb having suggests that the
context. spending occurs first, which does not make sense.
Answer choice D is the most simple and clear
40. The best answer is H. Only answer choice H
selection.
uses the active voice to clearly indicate that
either the pots could be unadorned, or they could 48. The best answer is F. The underlined portion is
be decorated with ribbons and paper. The other best as it is written. It is not appropriate to use
answer choices are in the passive voice and are a comma to separate a subject from its verb, so
awkward. answer choice G is incorrect. Both answer choices
H and J create an incomplete sentence that lacks a
41. The best answer is C. The sentence is discussing
main verb.
more than one piñata, so the plural form piñatas
is correct. The “s” apostrophe means plural pos- 49. The best answer is A. The underlined portion cor-
session, and the apostrophe “s” means singular rectly uses the contraction of you are, whereas
possession; possession is not indicated by the con- answer choice B incorrectly uses the possessive
text of the sentence, so answer choices A and B are pronoun your. The other answer choices are awk-
incorrect. ward and contain extraneous commas.
42. The best answer is G. The singular subject every 50. The best answer is J. The underlined portion is
needs the singular pronoun its. The word it’s is the redundant, because the sentence already indicates
conjunction of it is. that the person is special. Therefore, the best thing
to do is to omit the underlined portion. Because
43. The best answer is A. The phrase “In America” is
all of the answer choices are grammatically correct
an introductory clause that must be followed by a
and express the same idea, you should be able to
comma.
quickly recognize the redundancy.
44. The best answer is J. The context of the sentence
51. The best answer is B. This question tests your abil-
indicates that the piñatas are filled with candy and
ity to recognize idiom. In standard written English
toys; in fact, the sentence says pounds, which can
it is appropriate to use the phrase filled with.
be interpreted as a large quantity. Because bar-
ren means empty, it would not be an appropriate 52. The best answer is H. Because the sentence uses
alternative. the pronoun you earlier in the sentence, it is not nec-
essary to restate the pronoun later in the sentence.
45. The best answer is A. As it is written, the under-
It is clear that you are performing both actions—
lined portion clearly and effectively shows what the
decorating a picture frame and using nuts and
audience most enjoys doing: “watching the blind-
bolts.
folded person swing mightily at nothing but thin
air.” It is in the active voice and is free from ambi- 53. The best answer is D. The coordinate conjunction
guity. The other answer choices are either in the and is not necessary at this point in the sentence.
passive voice or do not make sense. The sentence presents a list of items that could be
used in a shadow box, and a dried flower is an item
that falls in the middle of the list. Therefore, no
additional language is necessary.
PASSAGE IV
54. The best answer is F. In order to avoid redundancy
46. The best answer is J. If a list contains three or
and wordiness, it is best to simply say gather from.
more items, it is necessary to separate those items
This clearly and effectively expresses the idea.
with commas. It is not appropriate to include a
comma between the verb read and the preposi- 55. The best answer is D. Sentence 1 is an appropri-
tional phrase in magazines, so answer choice F is ate introductory sentence because it includes the
incorrect. Answer choice G does not include any of transitional phrase For example, and then goes
the necessary separating commas, so it is incorrect. on to give examples of how to create a hand-
A semicolon should be used to separate items in a made gift from someone’s trinkets or other personal
list that follows a colon, so the use of a semicolon items. Sentence 2 directly follows with yet another
in answer choice H is incorrect. example. Sentence 4 would best follow sentence 2,
PRACTICE TEST 4 ANSWERS AND EXPLANATIONS 321

because it provides details on how to make a 63. The best answer is B. The plural subject These
shadow box. Finally, sentence 3 is best as the last statistics requires the plural verb take. The other
sentence because it effectively concludes the topic answer choices use the wrong verb form or tense.
discussed in the paragraph.
64. The best answer is F. The sentence in answer
56. The best answer is G. The sentence as it is writ- choice F introduces the idea of ingenious designs
ten is an incomplete sentence that lacks a main which are then discussed in the paragraph. Answer
verb. Answer choice G is correct because it is choice G does not provide an effective transition,
necessary to use a comma to separate an introduc- and answer choice H is irrelevant to the topic of the
tory clause from the rest of the sentence. Answer paragraph. Answer choice J is incorrect because the
choice H has the comma after the verb, which is focus of the passage is safety.
incorrect.
65. The best answer is C. In order to maintain par-
57. The best answer is D. It is best to use the adverb allelism within the sentence, it is necessary to use
nearly to modify the adjective endless. Answer illuminating to match the verb allowing.
choice D is best because it does not include the
extraneous verb to be. 66. The best answer is G. The plural noun headlights
requires the plural verb are.
58. The best answer is H. To identify more effec-
tively exactly what is happening, it is better to use 67. The best answer is B. You should use a semicolon
the specific pronoun this. The pronoun it is rather to separate two independent but related clauses.
ambiguous, and the coordinate conjunction so is A comma is insufficient and creates a comma splice,
not appropriate. so answer choice A is incorrect. Answer choices C
in incorrect because it does not include any punc-
59. The best answer is A. The student benefits by
tuation. Answer choice D is incorrect because it is
learning about scrapbooking and the store benefits
not correctly punctuated.
be receiving customer patronage. The other answer
choices offer support for why a patron would take 68. The best answer is H. By using the verb phrase
a class, but not for why a store owner would offer doing so, answer choice H clearly and effectively
such a class. indicates that using the high-beams would provide
more light. Answer choice F is incorrect because
60. The best answer is J. The notion that original,
the pronoun they is ambiguous—it is unclear
hand-made pictures or books will be enjoyed is
whether the antecedent is high-beams or drivers.
stated previously in the paragraph. Therefore, it
Answer choice G is incorrect for a similar reason—
is not necessary or relevant to mention it again.
the pronoun it is ambiguous. Answer choice J is
The other answer choices are not supported by the
awkward and contains an ambiguous pronoun.
essay.
69. The best answer is D. It is best to say switch-
ing proves because switching is singular. The other
PASSAGE V answer choices include incorrect verb tense and
aspect.
61. The best answer is D. No punctuation is neces-
sary in this sentence. A good rule of thumb is 70. The best answer is G. It is appropriate to use a
to use a comma wherever you would naturally colon to introduce a list of items. Answer choice F
pause in a sentence. Because there are no nat- is incorrect because it creates an incomplete sen-
ural pauses in this sentence, you don’t need a tence. Answer choice H is incorrect because it
comma. Answer choice B is incorrect because a includes a comma splice. Answer choice J is
semicolon should be used to separate two indepen- incorrect because it is awkward and wordy.
dent clauses. Answer choice C is incorrect because
a colon should generally be used to introduce 71. The best answer is B. To clearly indicate that the
a list. images are displayed on a screen after the images
are detected, it is best to use the phrase are then
62. The best answer is J. Answer choice J is best displayed. The images themselves do not display;
because it indicates not only that accidents occur rather, they are displayed, so answer choices A and
more frequently, but how many times more fre- C are incorrect.
quently they occur. This is more detailed and
relevant than the information contained in the other 72. The best answer is G. Sentence 5 should be the
answer choices. last sentence of the paragraph because it effectively
322 PRACTICE TEST 4 ANSWERS AND EXPLANATIONS

concludes the topic of the paragraph, and most a sentence. A coordinating conjunction connects
logically follows sentence 6. words, phrases, or clauses that are of equal impor-
tance in the sentence.
73. The best answer is D. The phrase “particularly
during dangerous nighttime driving” is a non- 75. The best answer is B. Paragraph 1 states that more
restrictive clause, which should be set off with accidents occur at night than in the daytime. The
commas. Only answer choice D correctly places new sentence uses the possessive determiner these,
a comma between road and particularly. which takes the mention of night accidents as its
antecedent from earlier in the paragraph.
74. The best answer is F. Use a comma with a coordi-
nating conjunction to separate main clauses within
394 PRACTICE TEST 5 ANSWER KEY

ANSWER KEY

English Test
1. C 21. A 41. A 61. B
2. H 22. J 42. J 62. F
3. B 23. C 43. B 63. D
4. H 24. F 44. G 64. F
5. A 25. A 45. C 65. D
6. H 26. J 46. F 66. F
7. D 27. B 47. B 67. C
8. G 28. G 48. H 68. J
9. A 29. A 49. A 69. B
10. H 30. H 50. F 70. J
11. A 31. C 51. D 71. D
12. F 32. F 52. J 72. J
13. B 33. A 53. D 73. C
14. H 34. G 54. H 74. G
15. D 35. D 55. C 75. D
16. F 36. F 56. H
17. C 37. D 57. A
18. H 38. J 58. G
19. D 39. A 59. D
20. F 40. H 60. G
396 PRACTICE TEST 5 ANSWER KEY

Reading Test Science Reasoning Test


1. D 21. C 1. C 21. A
2. J 22. F 2. H 22. G
3. C 23. D 3. D 23. C
4. H 24. F 4. G 24. H
5. B 25. D 5. D 25. A
6. F 26. G 6. G 26. H
7. C 27. C 7. B 27. D
8. H 28. H 8. H 28. H
9. B 29. D 9. A 29. B
10. H 30. G 10. J 30. F
11. B 31. B 11. D 31. D
12. G 32. J 12. H 32. H
13. A 33. A 13. A 33. B
14. F 34. H 14. F 34. H
15. A 35. B 15. C 35. A
16. H 36. H 16. F 36. J
17. C 37. B 17. B 37. B
18. G 38. J 18. H 38. H
19. B 39. B 19. A 39. A
20. J 40. J 20. H 40. H
PRACTICE TEST 4 ANSWERS AND EXPLANATIONS 329

Reading Test Explanations 7. The best answer is D. Mark’s quotation from the
last paragraph shows that the other team was more
PASSAGE I fatigued at the end of the game than was his team.
The sentence “‘We should have won the game’”
1. The best answer is C. The passage provides
shows that Mark believed the team was prepared
several interpersonal and physical examples of
to win but failed.
teammates working together for their goal of win-
ning the game. Mark’s quotation is an example of 8. The best answer is H. The last sentence of
encouragement. The narrator describes the role of paragraph 3 describes the hand clasping, which
linemen in pushing back the defense for the running immediately follows Mark’s statement of encour-
back. The other answer choices are not supported agement to his teammates. These two events come
by the passage. at the end of the huddle before the play. The
purpose of the huddle is to establish the play
2. The best answer is J. The passage gives details
and, hence, the upcoming roles of the individual
about the almost meditative quality of Joe’s prepa-
players. The hand-clasping ritual seems to tie the
ration for the snap of the football. According to
team back together momentarily before they must
the passage, “instinct told him what he needed to
break apart to perform their individual tasks during
know.” Engrossed is the quality of being deeply
the play.
involved in something. The narrator uses rich
descriptions of the rituals of the football play to 9. The best answer is A. The passage describes Joe
emphasize the players’ dedication to the game. being pleased by the block and “assuming” it had
sent his man back “more than enough to free the
3. The best answer is B. The passage discusses Joe’s
rusher to enter the endzone.” This best supports
surprise at the final outcome of the game. He first
answer choice A.
saw what had happened, before it truly sank in that
the game had been lost. This correlates with answer 10. The best answer is H. Mark’s quotation in para-
choice A. The passage then goes on to state that graph 3 is an example of the magnitude of the game
“‘We had them beat, you know,’ Mark hissed to (and so, the final play) in the context of a season
Joe as they walked slowly off the field.” This illus- that includes “practicing for four months.” Mark
trates both commiseration with Mark, and that he even alludes to the success of the final play reflect-
may have been “trudging” off the field in walking ing the quality of the team developed over four
slowly with his teammates. The only answer choice years.
not supported by details in the passage is answer
choice B. Nowhere in the passage does it state that
PASSAGE II
Joe attacked his opponents following the game.
11. The best answer is B. The first paragraph states
4. The best answer is G. The first paragraph provides
that the tourist was “looking for a souvenir that
details on the setting that reinforce the intensity
represents the local culture.” However, when he
of the game so far. “A cold wind soothed the
stopped to browse at a small street vendor, he was
faces” indicates the faces are hurt or uncomfort-
shocked to see a plethora of American items. This
able. The third sentence describes breathing dirty
best supports answer choice B. The other answer
air. The word “muster” implies considerable effort
choices are not supported by the context of the
on the part of the lungs. The phrases “game-
passage.
ending” and “season-defining” indicate the team is
at a crucial moment in the game. 12. The best answer is H. As written in the pas-
sage, “American logos appear in quite unex-
5. The best answer is D. The last sentence of the
pected places, embodying the ubiquitous American
passage describes how the loss is particularly dev-
symbols worldwide.” Because American symbols
astating because of how close Joe’s team came to
appear in unexpected places worldwide, it makes
winning on the final play. The other choices are not
the most sense that these symbols would be
supported by the passage.
described as “far-reaching.” Answer choice F is
6. The best answer is G. Details in paragraphs 4 incorrect; the price of these symbols is not men-
and 5 describe Joe’s careful analysis of the defense tioned anywhere in the passage. Answer choice G
made prior to the start of the play. The sentence does not make sense in the context of the passage.
“Joe glared at his opponents, pleased . . .” is the Answer choice J does not make sense because if
best example to support answer choice G. The next American symbols were localized, they would be
sentence provides further evidence. found solely in America.
330 PRACTICE TEST 4 ANSWERS AND EXPLANATIONS

13. The best answer is D. It is stated in the pas- something that is obviously occurring. To “regret”
sage that those people who reject Americana do is to express grief over or be unhappy with. The
so because they “lament the steady decline of passage illustrates that the people who are rejecting
distinct national, ethnic, and cultural identities American cultural imperialism are unhappy with its
as omnipresent American influences overpower consequences.
ancient traditions and beliefs.”
19. The best answer is A. As stated in the passage,
14. The best answer is H. The passage explicitly states “What he sees shocks him: T-shirts and posters
that jobs have been created due to the spread of promoting American football, basketball, and base-
American influence, and that with American com- ball teams.” This clearly illustrates the fact that the
merce comes the modernization of infrastructure. street vendor was selling American sports memo-
The passage does not state that declines in tourism rabilia. The other answer choices are not supported
result from the spread of American influences, by details in the passage.
but rather that American businesses are quickly
spreading to tourist areas abroad. 20. The best answer is F. The last paragraph of
the passage states that “Many benefits and prob-
15. The best answer is B. When the passage states that lems are created by American commercial and
“The local cantinas and native boutiques are losing cultural expansion abroad.” While there are some
the battle against large American corporations,” it drawbacks to American expansion, there are still
is clear that these places are straining to even stay in gains. This best supports answer choice F. Answer
business and retain their cultural uniqueness versus choice H may appear to be correct; however, the
the American businesses that are invading the resort last paragraph does not clarify whether problems
towns. The other answer choices are not supported from American expansion in the future will be
by details in the passage. greater than benefits.
16. The best answer is G. The passage states
that Japan is a “prime example of where PASSAGE III
American involvement has been beneficial,” and
that “America left a pervasive cultural footprint 21. The best answer is B. The passage asks “how a
on the country” while maintaining “a distinct cul- person is able to paint or draw something so lifelike
ture that is rich in the traditions of the past.” This and emotive,” and marvels at “what stirred these
clearly indicates that American influence was ben- men to put the brush to the canvas so delicately.”
eficial to Japan and that a well-balanced mix of “Emotive” means “causing strong feelings,” which
American and Japanese culture ensued. This best is also a synonym for “stirring.” This is the
supports answer choice G. The other answer choices most logical choice based on the context of the
are beyond the scope of the passage. passage.
17. The best answer is C. The passage illustrates both 22. The best answer is H. Realism is noted in
a country that has been helped by American influ- the passage as one of the first formal styles of
ence (Japan) and a country whose native businesses art, preceding Impressionism. The passage later
are suffering due to American influence (Mexico). states that among the styles of painting evolving
It is unclear what will happen if American cultural from Impressionism were “Cubism, Abstract Art,
influences continue to penetrate foreign markets. Expressionism, Abstract Expressionism, Mod-
Answer choice B may appear to be correct; how- ernism, and a host of other styles . . .” Realism is
ever, the example of America’s influence on Japan the only style of art not mentioned in this list.
negates this option. 23. The best answer is B. As stated in the pas-
18. The best answer is J. Paragraph 3 discusses those sage, Realist painters “focused on depicting real
people who are rejecting “American ‘cultural impe- life unembellished with fanciful notions or feel-
rialism,” ’ and states that these people “lament the ings.” This correlates with answer choice B.
steady decline of distinct national, ethnic, and cul- Answer choice C may appear to be correct because
tural identities as omnipresent American influences the passage makes note of unattractive physical
overpower ancient traditions and beliefs.” Because attributes. However, the passage simply states
these people are rejecting this spread of American that artists simply did not “gloss over” unattrac-
influence, it does not make sense that they would tive physical attributes of their subjects; artists
embrace or enjoy the loss of other cultures. It also portrayed their subjects as they actually looked.
does not make sense that they would deny the Unattractive features were not the focus of Realist
steady decline of distinct national cultures, as it is artists.
PRACTICE TEST 4 ANSWERS AND EXPLANATIONS 331

24. The best answer is G. The passage clearly states did its predecessor.” This correlates best with
that though Realism fell out of favor for a while, answer choice A. The other answer choices are
it was “revived during the Renaissance,” and not mentioned in the passage as describing Post-
remained popular for several centuries thereafter. Impressionism in comparison to Impressionism.
This best supports answer choice G. The other
30. The best answer is J. As stated in the passage,
answer choices are not supported by details found
“a rebellion against Realism arose in response
in the second paragraph.
to the rigidity and staleness some saw in the
25. The best answer is B. The passage begins with style.” The author goes on to say that many artists
one of the first popular formal styles of art. The began to paint in the Impressionist style because
passage then goes on to describe different styles it “allowed for more creativity.” These facts best
of art that have evolved over time. This best sup- support answer choice J. Although Realism could
ports answer choice B. Answer choice A is incorrect trace its roots to ancient Rome—answer choice G—
because the passage does not discuss the chang- this was not the reason that artists rebelled against
ing world and how painters related to it. Answer Realism.
choice C is incorrect because the passage only men-
tions the popularity of Realism falling and rising PASSAGE IV
again; the passage does not discuss the popular-
ity of other styles of painting. Answer choice D 31. The best answer is C. The primary focus of the
is incorrect because the passage does not discuss passage is on gene linkage; what it is and how it
artists adapting to popular demand. works. The other answer choices are too specific
and do not adequately express the overall main idea
26. The best answer is J. The passage states that while
of the passage.
some styles of art that grew out of Impression-
ism became their own schools of thought, others 32. The best answer is G. The passage states that
were simply an “artist’s trademark way of paint- “Each hereditary unit, the gene, contains spe-
ing.” In this context, speaking of styles of art, it cific . . .” This clearly shows us that in this passage,
makes the most sense that an “artist’s trademark” a hereditary unit is referred to as a gene.
would be his or her own “unique style.” Although
answer choice F, “prime example,” may appear to 33. The best answer is B. The passage lists hair color,
be correct, the author is discussing different styles height, and susceptibility to disease all as inherited
of art, not just one specific piece of art. Answer traits. Therefore, answer choice B is correct. The
choices G and H do not make sense in the context passage tells us that genes are found on segments of
of the passage. the DNA molecule, not that molecules are examples
of inherited traits.
27. The best answer is A. As stated in the passage, the
Impressionist movement was a “rebellion against 34. The best answer is J. The passage describes the
Realism,” which came as a result of “the rigidity fruit fly as a “prolific breeder, producing hundreds
and staleness some saw in the style.” The rigidity of offspring in a single mating.” The word “pro-
and staleness are referring to the Realist style of lific” means “productive and fertile,” which best
painting, not the Impressionist style of painting. correlates with answer choice J. This makes the
Therefore, answer choice A is correct. most sense in the context of the passage; if fruit
flies produce so many offspring in a single mating
28. The best answer is H. The author lists many they are clearly highly productive breeders. The
styles that evolved from Impressionism; these other answer choices do not fit the context of the
details correlate best with answer choice H. Answer sentence as well.
choice F is incorrect because Impressionism greatly
expanded the range of artistic expression through 35. The best answer is C. According to the first
the many styles of art that developed as a result paragraph, asexual reproduction involves a single
of the movement. Answer choice G is incorrect parent.
because Realism was a precursor to Impression- 36. The best answer is F. The last paragraph of the
ism, not vice versa. Likewise, answer choice J is passage indicates that current research is exploring
incorrect because Abstract Art and Cubism were the fact that “many factors affect the transmission
predecessors of Impressionism, not vice versa. of certain traits from parents to offspring.” The
29. The best answer is A. The passage states that passage goes on to state the location of genes is
“Postimpressionism uses form and color to reflect “but one of a multitude of determinants involved in
art in a more personal and subjective way than whether or not a characteristic will be inherited.”
332 PRACTICE TEST 4 ANSWERS AND EXPLANATIONS

These facts suggest that research is ongoing, and traits, as well as the “multitude of determinants”
that there are still many questions regarding the that determine whether a characteristic will be
transmission of genes from one generation to the inherited. These statements indicate that there are
next. This best correlates with answer choice F. many components that affect genetic transmission,
37. The best answer is B. After stating the Morgan which is most consistent with answer choice D.
chose to experiment on Drosophila melanogaster, The other answer choices are not supported by the
the passage cites a reason for this choice: “The information found in the last paragraph.
fruit fly is a prolific breeder, producing hundreds 40. The best answer is J. To answer this question,
of offspring in a single mating.” This best cor- a parallel must be drawn between the logic used
responds with answer choice B. Although answer in the passage and the question at hand. As writ-
choice A may appear to be correct because fruit flies ten in the passage “the genes for body color and
have easily distinguishable chromosomes, fruit wing size are transmitted together from parents to
flies have only four pairs of chromosomes, not offspring because they are located on the same
“many.” chromosome and must be somehow linked.” There-
38. The best answer is G. As stated in paragraph 4, fore, if the genes for blue eyes and brown hair
Morgan bred “female flies that appeared normal, are found on the same chromosome, these two
but carried the mutant genes.” This tells us that traits would be inherited together. Because the
the female flies held the mutant genes, yet did passage also states that combinations from both
not appear to be mutated. This statement is best parents were present, it can be concluded that a
supported by answer choice G. The other answer certain number of offspring would inherit both blue
choices are not supported by the context of the eyes and brown hair, but a certain number of off-
passage. spring would receive the eye and hair color of
39. The best answer is D. The last paragraph discusses the other parent. This best correlates with answer
the “many factors” that affect the transmission of choice J.
PRACTICE TEST 5 ANSWERS AND EXPLANATIONS 401

ANSWERS AND EXPLANATIONS

English Test Explanations

PASSAGE I
1. The best answer is C. The words surprised and 9. The best answer is A. The semicolon is the
shocked both have the same meaning. To avoid appropriate punctuation mark between two closely
redundancy, you should use only one of them related sentences.
in the sentence. Eliminate answer choice A. One
10. The best answer is H. The correct word is
would also not be saddened or disappointed to learn
the conjunctive adverb otherwise, which properly
about Michigan’s location 350 million years ago.
introduces what will occur if some action is not
Eliminate answer choices B and D.
taken.
2. The best answer is H. Only answer choice H 11. The best answer is A. Sentence 2 describes the
clearly indicates in a clear and concise manner that case opposite to that in Sentence 1; therefore,
Michigan was covered by water. The other answer Sentence 2 should follow Sentence 1. The word
choices are awkward. though signals that the sentence presents some
3. The best answer is B. The word because creates information that counters a preceding statement.
an incomplete sentence. The word from implies that 12. The best answer is F. The phrase is correct as writ-
the stone and its pattern are two separate entities. ten, with the noun in plural non-possessive form
By is the only answer choice that makes it clear that and no punctuation between subject and verb.
the stone is easily identified through recognition of
its pattern. 13. The best answer is B. The verb must agree with the
simple subject advantage, which is singular; this
4. The best answer is H. The passage tells us the cells eliminates answer choice A. The clausal predicate
turned into rock over time. The phrase as slow as that comes after is requires that to be grammat-
suggests a comparison, which there is not. Slow to ical; therefore, answer choices C and D may be
is in the present tense, but the passage is written eliminated.
in the past tense. Slowly is the only answer choice
14. The best answer is H. The sentence is evi-
that properly modifies the verb becoming.
dence of the claim made in the first sentence
5. The best answer is A. This question asks you to of the paragraph, which asserts that Petoskey
decide which phrase indicates that the Petoskey stone availability fluctuates over the course of the
stones can differ from each other. The phrase but year.
some also contain highlights the fact that while 15. The best answer is D. The essay is clearly about a
the stones are high in calcite, the elements that particular kind of rock. No details are given about
they contain can vary. All other answer choices living near the Great Lakes.
imply that the stones always contain these varied
elements.

6. The best answer is H. This sentence is written in PASSAGE II


the present tense; eliminate answer choice J. The 16. The best answer is F. The underlined portion is
question requires you to correctly place the mod- clear and precise. The remaining answer choices
ifier high or highly. Answer choice H, requires a are not idiomatic.
high level, is the most concise and correct answer.
17. The best answer is C. The simple subject of the
7. The best answer is D. The items in a list must be sentence is each, which is singular; this eliminates
separated by commas, so there should be a comma answer choices A and D. Answer choice B, “settled
after wind and a comma after sand. into,” is not idiomatic in this sentence.
8. The best answer is G. This sentence elaborates on 18. The best answer is H. The simple present-tense
the hard labor the preceding sentences mention. form of this answer choice parallels the verb form
To say that the stones are conducive to polish- of the previous sentence. Answer choice F may be
ing means that they are easy to polish, making grammatical, but should be eliminated for being in
Fortunately the appropriate introductory word. passive voice.
402 PRACTICE TEST 5 ANSWERS AND EXPLANATIONS

19. The best answer is D. The word us ends the first where the simple past tense form “landed” is
clause and we begins the next one; therefore, some also used.
conjunction or punctuation is necessary to separate
32. The best answer is F. The sentence is best as it is
them.
written. The other answer choices have clear redun-
20. The best answer is F. The phrase followed by is a dancies; the notion of a beginning implies it is the
common introductory phrase, therefore, the other first of a course of actions.
answer choices may be eliminated.
33. The best answer is A. The verb continue, as it
21. The best answer is A. The phrasal verb sound off is used here, takes an infinitive verb (“to be”) as
means “express one’s opinion vigorously.” This is its complement; this eliminates the other answer
the most humanlike of the answer choices. choices.
22. The best answer is J. This sentences follows 34. The best answer is G. The word is used to mean all
logically from the previous one, which describes of the Australian people, so it should be in plural,
daytime-flying animals. Additionally, a new sub- non-possessive form.
topic is being introduced, so it is appropriate to
begin a new paragraph. 35. The best answer is D. The word principal has
nearly the same meaning as main, so it is redundant
23. The best answer is C. The simple present-tense and may be eliminated.
form coincides with the same form in the previous
clause: dart. 36. The best answer is F. The word majority means a
group more than half of the whole. This indicates
24. The best answer is F. This sentence develops the that the dialect is “the most prevalent form,” as the
idea of “the massive blanket of stars” presented in question asks.
the previous sentence.
37. The best answer is D. The subject of the clause
25. The best answer is A. The passage maintains a in General Australian English, which is singular;
pattern of vivid descriptions of natural sounds and this eliminates answer choices A and B. Answer
surroundings. This sentence follows that pattern choice C may be eliminated because the clause
better than the other choices. on diction (word choice) does not detract from the
26. The best answer is J. A subordinate clause intro- claim that the dialect has different vowel sounds.
duced by but must be preceded by a comma.
38. The best answer is J. The paragraph defines
27. The best answer is B. Sentence 2 strays from dialects of Australian English in order of preva-
the pattern of vivid descriptions of sights, sounds, lence; therefore, the cultivated form of Australian
and emotions that is maintained throughout the English is the least spoken. In fact, the passage
passage. cites a figure of 10% of the population.
28. The best answer is G. Two opposing adjec- 39. The best answer is A. The word too is an adverb;
tives divided by but do not require punctuation the other answer choices are not. Adjectives like
to separate them. This rule also applies with yet. haughty and snobbish must be modified by adverbs.
For example: “a fun but exhausting afternoon Answer choice C is an adverb, but the word too
on the lake,” and “The experience was silly yet more clearly proves why many Australians object
rewarding.” to the cultivated dialect.
29. The best answer is A. The skin in question is still 40. The best answer is H. The vast majority is a
cool, so the present perfect form is appropriate here, common expression. An article such as the or a
and no changes are required. may never be omitted; therefore, the other answer
choices may be eliminated.
30. The best answer is H. This verb should be in sim-
ple present-tense form to match the same form of 41. The best answer is A. As it is written, the sentence
jerk later in the sentence. is clear and concise. The other answer choices are
awkward.
42. The best answer is J. This detail identifies the phe-
PASSAGE III
nomenon for which examples are given in the next
31. The best answer is C. The best clue is in the sentence. The other choices are not supported by
Australian version at the beginning of the passage, the passage.
PRACTICE TEST 5 ANSWERS AND EXPLANATIONS 403

43. The best answer is B. The act of publication simple past is the most appropriate of the answer
occurred in the past and is finished; therefore, the choices. Answer choice H is actually an incorrect
simple past-tense form is required. The simple sub- pair of the auxiliary verb had with the simple past
ject Dictionary is singular, so answer choice C may form of sink, which is sank.
be eliminated.
53. The best answer is D. Sentence 1 introduces the
44. The best answer is G. Flipping Sentences 2 and 3 paragraph by mentioning the author’s first expe-
allows the paragraph to flow more logically. Sen- rience with eyeglasses. Sentence 4 comes next
tence 3 describes the implementation of and sub- because it is a logical exclamation to include about
sequent problems with the Macquarie Dictionary. never having had clear vision. Sentences 2, 3,
Sentence 2 describes the publisher’s reaction to the and 5, in that order, describe the first sensations of
problems. going outside with new glasses. Sentence 6 tells the
45. The best answer is C. The essay is about ultimate effect that the glasses had on the author,
Australian language exclusively. No other topics so it naturally concludes the paragraph.
are addressed that would be important information
54. The best answer is H. The word later is part of
for a tourist, so the passage would not serve well
the phrase six years later and must be included.
as the article the question describes.
The than in answer choice G is incorrect because
no comparison of time is being made within the
PASSAGE IV sentence.
46. The best answer is F. As it is written, the sen- 55. The best answer is C. The best introductory word
tence correctly matches singular dime with each is unfortunately because the sentence defines the
eye. Answer choice G could only be correct if the undesirable feature of the lenses—their thickness—
sentence described instead putting dimes (plural) in that caused all the troubles the author describes ear-
“your eyes.” Obviously, one dime could not be set lier in the paragraph. The other answer choices do
into two eyes at once. not reflect the relationship between lens thickness
47. The best answer is B. The sentence is clear that and the author’s problems.
the act of putting in contacts occurred regularly
and in the past. The past progressive (e.g. “I was 56. The best answer is H. The author did not intend to
studying”), sometimes called the past continuous, have the lenses pop out, so answer choices F and G
is appropriate here. may be eliminated. Answer choice J is not the best
answer because a pronoun could take the place of
48. The best answer is H. There should not be punc- the lenses and also does not clearly link to anything
tuation between a verb (was), and its modifier in context.
(truly), and its predicate (a novelty). Adding com-
mas breaks up the natural rhythm of the sentence 57. The best answer is A. The correct conjunction in
and makes it awkward to read. the case is or because the author is proposing two
alternative illustrations of the same annoyance of
49. The best answer is A. As it is written, the sen- retrieving a lost lens in the bathroom. The other
tence is clearest and most concise. The other answer answer choices would indicate the two events are
choices incorrectly associate in the third grade with more closely linked to each other.
only one of the two diagnoses.
58. The best answer is G. The prepositional phrase
50. The best answer is F. The learning described in
introduced by certainly is further detail added to the
the sentence is in the past and completed, so the
lens’ falling, so it must be separated from preceding
simple past form is appropriate.
prepositional phrase with a comma.
51. The best answer is D. The other answer choices
suit the tone of the passage, which describes early, 59. The best answer is D. This answer choice is the
hard-to-manage lens technology. Commenting on strongest detail in support of how helpless the
the simplicity or primitiveness of a slide presen- author is to see when missing a contact lens.
tation helps reinforce the notion of the difference
60. The best answer is G. The sentence provides a
between modern technology and that of the 1960s,
response to the rhetorical question posed previ-
the time of the author’s childhood.
ously. The passage shows how the author recovered
52. The best answer is J. The sentence describes a from numerous vision-correction setbacks, so this
simple habitual action in the past; therefore, the sentence maintains that pattern.
404 PRACTICE TEST 5 ANSWERS AND EXPLANATIONS

PASSAGE V Answer choice D creates an incomplete sentence,


so it should be eliminated.
61. The best answer is B. Quotations are set apart
from the short clause that defines the speaker with 68. The best answer is J. The context of the paragraph
a comma (e.g. “Joe says, ‘I like cheese,’” or “ ‘Go indicates that certain events happen in order: first
home,’ cried Martha.”) An exclamation point may the drawings are applied to cels, and eventually
be used when the sentence warrants it; this sen- those cels are photographed in succession to cre-
tence does not, since it is not an unusually excited ate a movie. Only answer choice J expresses this
utterance. idea. Answer choices F, G, and H can be eliminated
because they mean essentially the same thing and
62. The best answer is F. The preceding sentence is
cannot all be correct.
a quotation from a corporate publicist. This sen-
tence should be relevant to her concerns. The other 69. The best answer is B. The word animated means
answer choices too broadly address the field of moving, so to avoid redundancy, one should be
graphic arts. eliminated. Only answer choice B is free from
63. The best answer is D. The sentence should mean redundancy.
that Mauro’s comment is relevant to animation in 70. The best answer is J. Only answer choice J clearly
general, not limited to some time period in the and concisely expresses the intended idea. The
past. This eliminates answer choice C. The also in other answer choices are awkward.
answer choice D is important for tying the previous
comment on business with the upcoming mention 71. The best answer is D. It is not necessary to use
of animated films. both amount and time in the same sentence. To
avoid redundancy and wordiness, select one or the
64. The best answer is F. This sentence essentially other. Answer choice D is clear and concise.
defines the earliest animation as a kind of flip-book,
which is the subject of the next sentence. 72. The best answer is J. The phrase animators from
all over the world is a complex noun phrase that
65. The best answer is D. This adverbial phrase (When must not be broken up with commas, as in answer
the pages are flipped in rapid succession) begins the choices F and H, nor separated from the sentence
sentence and so should be separated from the main with a comma, as in answer choice G.
clause (an action scene unfolds . . .) with a comma.
73. The best answer is C. A comma cannot be the
66. The best answer is F. The conjunctive adverb
only element separating two independent clauses.
although is the best choice because it is used to
A dash, semicolon, or period may be used.
lessen the degree of a preceding claim. Here, the
complication and time requirements of traditional 74. The best answer is G. Describing data entry on a
animated films takes away from the assertion that graphic’s tablet corresponds to the mention of scan-
the production is based on the same principle as ning hand-drawn images into a computer describes
flip-books. in Sentence 2. Sentence 3 introduces the description
of the computer-side editing process, so Sentence 5
67. The best answer is C. The phrase or sheets of
could come before it.
clear plastic describes the cels. It is correct to
use a comma to separate a modifying clause from 75. The best answer is D. This sentence is a
the noun that it modfies. A period or a semi- good introductory statement for Paragraph 4, in
colon should be used to separate two independent which artists’ adaptation to technological change
clauses, so answer choices A and B are incorrect. is addressed.
412 PRACTICE TEST 5 ANSWERS AND EXPLANATIONS

Reading Test Explanations reach of civilization. Answer choice D is beyond


the scope of the passage; nowhere in the passage
PASSAGE I is a wilderness skills training facility mentioned.
1. The best answer is D. The first paragraph discusses 6. The best answer is F. In the passage, the narrator
the fact that people today are “fortunate to enjoy asks the boss what the rationale is behind con-
many conveniences once unheard of or reserved for structing his lodge so far out in the wilderness, and
the elite,” and then goes on to declare that people the narrator continues on to describe the boss as a
take water and electricity for granted. These details “peculiar man.” These statements indicate that the
best support answer choice D. Answer choice A narrator does not fully comprehend the extreme iso-
may appear to be correct; however, the passages lation of the lodge. The narrator goes on to describe
states the it is “in the future” when the prices of the boss as having impressive “creative solutions to
these commodities will be so high that people can the lack of most kinds of outside assistance.” This
no longer take their quantity and use for granted. implies that the narrator respects the resourceful-
ness, or ingenuity, of the boss. These facts best
2. The best answer is J. The passage indicates that
correlate with answer choice F.
water from the blue tap is “drinkable hot and cold
water,” while water from the red tap is “unpurified” 7. The best answer is C. Modern society’s consump-
and not meant for consumption by humans. If the tion of water and power is described as being taken
water from the red tap is unpurified and not drink- for granted and presumed. This implies that society
able, it makes the most sense that those who drank does not value water and power as much as they
it would run the risk of falling ill from waterborne should, nor do they take conservation efforts that
pathogens – answer choice J. Answer choice G may they could. “Exorbitance” means “excessiveness
appear to be correct; however, water with added and wastefulness,” which makes the most sense in
chlorine and fluoride is water that has been puri- the context of the passage.
fied. As indicated by the passage, water from the
8. The best answer is H. The author follows his state-
red tap is not purified.
ment that he was “off the grid” with the fact that
3. The best answer is C. In the passage, the author he was fully dependent on the lodge to give him
states that “The boss is a peculiar man,” and that “heat, light, water, and sanitation.” All of these pro-
“His creative solutions to the lack of most kinds of visions are components of utility systems—answer
outside assistance are impressive.” This indicates choice H. Therefore, though the word “grid” could
that the boss is both eccentric and enterprising. refer to any of the answer choices, answer choice H
The other answer choices are beyond the scope of makes the most sense in the context of the passage.
the passage and not supported by descriptions of the
9. The best answer is B. Throughout the passage,
boss of the lodge.
the lodge is described as being extremely rustic.
4. The best answer is H. The passage states that However, it is mentioned that a computer is present
though the hunters often return with “migratory in the back office, and that at night “a limited set
birds and caribou,” the narrator has seen “plenty of of high-efficiency fluorescent bulbs illuminates the
other unique game come back to the lodge kitchen corridors and public spaces.” This best corresponds
for preparation.” This indicates that an unusual with answer choice B; even though the lodge is
variety of game exists in the woods surrounding rustic and has a wilderness charm, some modern
the lodge, answer choice H. Answer choices F, G, conveniences help it remain habitable.
and J are all statements made without supporting
10. The best answer is H. The passage discusses the
details to back them up present in the passage.
fact that the lodge is situated “deep in the wilder-
5. The best answer is B. The second paragraph dis- ness,” “far beyond the reach of civilization.” The
cusses the many hunters who voyage to the lodge fact that the lodge is far from civilization correlates
every summer, and the fact that the lodge is almost best with answer choice H, which states that civi-
totally self-sufficient. The third paragraph discusses lization is not present to alter or pollute the rivers.
the boss’ desire to situate himself so deeply into Answer choice F is incorrect because the passage
the wilderness and his hardiness. These facts all indicates neither the quantity of fish in the creeks,
support answer choice B. Answer choices A and C nor the effect that would have on the cleanliness of
are incorrect because they indicate that the boss the creeks. Likewise, answer choice J is incorrect
would like a more modern, less remote hunting because the types of fish caught in the creek are not
lodge, when he clearly enjoys being beyond the mentioned. Although answer choice G is mentioned
PRACTICE TEST 5 ANSWERS AND EXPLANATIONS 413

in the passage, its relevance to the cleanliness of the 15. The best answer is A. The fifth paragraph gives
creeks is not drawn and therefore is incorrect. two treatments that were used in handling polio
patients and says that the “latter approach soon
became typical protocol because it seemed to
relieve some pain and discomfort,” referring to the
PASSAGE II
exercising of the paralyzed limbs of polio patients.
11. The best answer is B. As stated in the pas- This suggests that exercising soon became the most
sage, polio was a “particularly devastating disease commonly practiced treatment for polio patients,
because of its effect on children. Many children which best correlates with answer choice A.
stricken with polio became permanently confined to Answer choices B, C, and D do not make sense
wheelchairs or died at a very early age.” The effects within the context of the passage.
of this disease on children were clearly devastat-
ing and far-reaching. Although there were effects 16. The best answer is H. Throughout the passage,
on the medical community because its members the author discusses the difficulties that medical
were desperately searching for a cure, the effects researchers had in finding a cure for polio, and
on children were much more extensive and severe. many of the different attempts made at treating
Answer choices A and D do not make sense within the inexplicable disease. This best correlates with
the context of the passage. answer choice H. Answer choice G may appear to
be correct; however, nowhere in the passage does
12. The best answer is G. The passage speaks it mention that doctors and researchers were com-
of “managing the disease’s debilitating effects,” peting to find a cure; all those who were studying
which, according to the passage, included difficulty polio were working towards the same goal. Like-
breathing, paralysis, and potential death. These wise, answer choice J may appear to be correct
effects all can best be described as “crippling,” because the medical community did have trouble
answer choice G. Answer choices F and J do not understanding how to cure polio. However, answer
make sense; they both imply that the effects of choice J is too broad in stating that the medical
the disease brought strength and vitality to patients community as a whole lacks understanding about
with polio. Answer choice H is incorrect because curing diseases in general.
“coercing” means “forcing to think or act in some
way.” This does not make sense in the context of 17. The best answer is C. The passage discusses
the passage. Salk’s breakthrough vaccine and its effectiveness
and then goes on to state that “Later, another med-
13. The best answer is A. According to the passage, ical researcher named Albert Sabin developed an
the iron lung assisted patients in breathing, and even easier method of distributing the vaccine.”
some patients “would have to continue this treat- This suggests that Sabin had developed an easier
ment their entire lives.” This suggests that if the method of distributing Salk’s vaccine. This best
iron lung was not in existence, or if the patients supports answer choice C. Answer choice A may
did not continue iron lung therapy, that they may appear to be correct because Sabin’s vaccine was
have not have survived for nearly as long with their easier to distribute than Salk’s. However, Salk’s
disease. This best correlates with answer choice A. vaccine was indeed successful in curing polio, it
Answer choices B and C are outside the context merely was not as easy to distribute as Sabin’s.
of the passage; neither is associated with the iron
lung in the passage. Answer choice D refers to 18. The best answer is G. Cancer is very similar today
the exercising of paralyzed arms and legs, not the to what polio was in the early 20th century. Like
iron lung. polio, cancer has many crippling effects and for
most types of cancer, there is still no known cure
14. The best answer is F. The passage states that
despite the many attempts of medical researchers.
“conditions such as overheating, chilling, and even
Therefore, G is the best answer choice. Answer
teething were thought to cause polio’s symptoms.
choices F and H are epidemics like polio, but nei-
Some scientists and doctors even believed that dis-
ther is a strictly medical condition and both can be
eases such as whooping cough and pneumonia were
cured. Answer choice J is outside the context of the
the cause of polio.” The passage goes on to say
passage.
that “For many decades, polio research centered
on treating symptoms …” Clearly, scientists and 19. The best answer is B. As stated in the passage,
doctors did not have a clear picture as to the cause “Some scientists and doctors even believed that dis-
of polio. This best supports answer choice F. eases such as whooping cough and pneumonia were
414 PRACTICE TEST 5 ANSWERS AND EXPLANATIONS

the cause of polio.” Therefore, whooping cough creativity to make it to the finish line before other
and pneumonia were thought to trigger polio, not to animals; the rat climbed atop the ox to cross the
be the result of polio. Answer choices A, C, and D river and jumped ahead of him to reach the finish
are all supported by details found in the passage. line and the snake coiled itself around the horses
leg and jumped ahead at the last moment to beat the
20. The best answer is J. The author states in the last
horse. This best suggests that “cunning” refers to
sentence of the passage that “500,000 Americans
the “ingenuity,” or the “cleverness and creativity,”
continue to live with the effects of childhood polio
of the rat and the snake – answer choice F. These
infections that occurred decades ago.” These peo-
animals were certainly not “dependable,” in refer-
ple still are feeling the consequences of a disease
ence to the cat and rat incidence. They also did
that was prevalent years and years ago, answer
not appear to be “apprehensive” or “nervous and
choice J. Answer choice F directly contradicts the
worried” about making it to the finish line on time.
author’s final statement in the passage. Answer
Likewise, it does not make sense within the con-
choice G is incorrect because an effective cure has
text of the passage that the rat and the snake were
been found for polio; therefore, children are not at
“tolerant” or “understanding and patient.”
risk for infection of the disease. Answer choice H
is outside the context of the passage. 25. The best answer is D. As stated in the passage,
“Even numbered years are considered Yang, and
PASSAGE III odd numbered years are considered Yin.” The only
answer choice that does not correctly pair even
21. The best answer is C. All four answer choices numbered year with Yang and odd numbered year
are mentioned directly or indirectly throughout with Yin is answer choice D, stating that 3030
the passage. However, you must decide between would be a year of Yin.
which answer choices are supporting details and
which answer choice is the main idea of the pas- 26. The best answer is G. The passage states that the
sage. Answer choice C best encompasses the whole cat “was a victim of the rat’s cunning.” This once
of the passage by including both past myths and again implies the craftiness of the rat in assuring
present times, both of which are discussed through- himself and himself alone a place among those ani-
out the passage. Answer choices A, B, and D are mals that received the Emperor’s great distinction.
all true, but are not the main focus of the passage. This best corresponds with answer choice G. The
passage does not indicate what or how many other
22. The best answer is F. Following this statement, animals in all strove to earn the Emperor’s great
the passage reads “This is simplified to such a distinction; therefore, answer choice F is incorrect.
great degree by following the Gregorian calendar.” Answer choice H is incorrect because although the
Therefore, the phrase mentioning ease of finding cat slept through the race, there is no evidence
out the date is referring to the Gregorian calendar. that he was “indolent,” or lazy. Answer choice J
The passage goes on to state that “This has not is beyond the scope of the passage; no reference
always been the case, however,” referring to the is made to which animal’s companionship the ox
ancient Chinese Zodiac calendar. These facts best preferred.
support answer choice F.
27. The best answer is C. The dragon was delayed
23. The best answer is D. “Vanity” is an “extreme
in crossing the finish line because he needed
interest in one’s own appearance and achieve-
to make rain for the people and “His kindness
ments.” Thus, the dog was more interested in how
earned him the fifth place in the cycle.” The
he looked than in his performance in the race, caus-
sheep, monkey, and rooster all worked together
ing him to stop and bathe in the river. Answer
to cross the river and “For this show of team-
choice D best summarizes this, in saying that the
work they were rewarded in the order that they
dog prioritized his selfish whims over his wish
stepped off the raft.” These are the only animals
to reach the Emperor’s Palace in time. Although
who are described as being “rewarded” with a
answer choice B may appear to be correct, vanity
place in the cycle, thus it can be concluded that
refers to self-interest, not the interests of others.
the emperor most values “generosity,” or “kind-
Therefore, the dog was taking a bath for his own
ness,” and “unanimity,” or “unity and teamwork.”
benefit and not that of the Emperor.
Answer choice B may appear to be correct because
24. The best answer is F. Both the rat and the snake “goodwill” is a synonym for “kindness”; how-
are described as “cunning” when participating in ever, “narcissism” is a symbol for vanity; the vain
the race. Both animals used their cleverness and dog was not rewarded with his place among the
PRACTICE TEST 5 ANSWERS AND EXPLANATIONS 415

12 animals but merely earned it by reaching the 32. The best answer is J. As stated in the passage,
finish line in time. “chimpanzees do most of their hunting in the dry
season.” The passage also notes that chimpanzees
28. The best answer is H. As stated in the passage,
supplement their diet with meat when food short-
“Suddenly, hidden coiled around the leg of the
ages occur in the forest, particularly during dry
horse, appeared the snake.” The snake was coiled
season. Although answer choices G and H may
around the leg or the horse, not the leg of the
appear to be correct, they are not inclusive of all
dragon. Therefore, answer choice H is correct. All
of the environmental factors that play a role in
of the other answer choices can be found within the
the hunting habits of chimpanzees. Thus, answer
passage.
choice J is correct. Answer choice F is incorrect
29. The best answer is D. The passage states that the because although the red colobus monkey is noted
“origin of the 12 animals is mythological, with the as “one of the most important animals in the chim-
story being passed down from generation to gen- panzees’ diet,” the relative ease of hunting the red
eration.” The story of the 12 animals is clearly not colobus monkey is not specified.
based on actual fact, so answer choices A and B
33. The best answer is A. It is noted in the passage that
can immediately be eliminated. “Legitimate” and
“chimpanzees are actually very proficient hunters,”
“bona fide” are both synonyms for “genuine” and
and that “Individual chimpanzees have been known
“true.” Answer choice C is incorrect because the
to hunt and eat more than 150 small animals each
word has “ludicrous” has negative connotations,
year.” Answer choice A–“skilled”–makes the most
meaning “ridiculously absurd.” Answer choice D,
logical sense within the context of the passage.
“legendary,” makes the most sense within the con-
The sheer number of animals that chimpanzees can
text of the passage. If something is “legendary,”
successfully hunt each year tells you that they are
such as the tale of the 12 animals, it is “an ancient
competent at hunting and can find plenty of prey to
or traditional story lacking actual historical basis.”
supplement their diet each year.
30. The best answer is G. The last sentence of the
34. The best answer is H. The passage indicates that
paragraph is referring to the second-to-last sen-
chimpanzees prey on many different kinds of small
tence of the paragraph, which states “Primary
animals and that the amount of meat that they
among the great astrological purposes to the Zodiac
consume varies each year with environmental fac-
is the common belief that the animal that governs
tors. Not only this, but chimpanzees’ diets can
the time of a person’s birth will influence that per-
vary due to social and reproductive factors. This
son’s personality for life.” The author is indecisive
information best supports answer choice H; the
as to whether this statement is true or not, lead-
exact diet of chimpanzees cannot be determined.
ing the reader to believe that the author has not
Answer choice A is incorrect because the study in
taken a side in the debate over the accuracy of
the passage was done in the wild. Answer choice G
the Chinese Zodiac and its influence on person-
is incorrect because the passage clearly states
ality. This best corresponds with answer choice G.
“Although it is clear that the hunting habits of chim-
Answer choice J may appear to be correct; how-
panzees are guided mostly by nutritional needs …
ever, the last sentence is referring to the Zodiac’s
chimpanzees might hunt for social gain.” It is a
influence over people’s personalities, not the
possibility that chimpanzees’ eating habits could be
calendar year.
influenced by social factors, but not a given fact.
35. The best answer is B. The passage states that
PASSAGE IV “It is not difficult to determine why studying
chimpanzees might be beneficial. Modern humans
31. The best answer is B. The passage discusses first
the diet and hunting behaviors of chimpanzees, and and chimpanzees are actually very closely related.
then moves on to briefly discuss similar human Experts believe that chimpanzees share about
98.5% of our DNA sequence,” which best supports
activity in diet and early human hunting and gather-
answer choice B.
ing. This is the overall focus of the passage, which
most closely corresponds with answer choice B. 36. The best answer is H. As stated in the passage,
Answer choice A may appear to be correct; how- “This is comparable to the quantity of meat eaten
ever, the passage solely discusses diet and hunting by modern human societies whose members forage
habits; the passage does not go into depth about when other food sources are scarce.” This sen-
biological and physiological similarities between tence indicates that these humans looked for other
humans and chimpanzees. sources of food, such as meat, when their normal
416 PRACTICE TEST 5 ANSWERS AND EXPLANATIONS

food sources were in short supply. This corresponds for social gain.” You can infer then, that ancient
with answer choice H, “search for food.” humans also hunted for social gain and prestige in
their communities. The other answer choices are
37. The best answer is B. The passage states that
not supported by information in the passage.
“It was Goodall who first reported that meat was a
natural part of the chimpanzee diet. In fact, Goodall 39. The best answer is B. As stated in the passage,
discovered that chimpanzees are actually very pro- “During the dry season, food shortages in the forest
ficient hunters.” This tells us that Goodall was the cause the chimpanzees’ body weight to drop. Con-
first scientist to discover previously undocumented sequently, chimpanzees supplement their diets with
chimpanzee behavior, and that chimpanzees were meat.” This best corresponds with answer choice B.
actually not vegetarians. Answer choice D might Answer choices A, C, and D are not supported by
appear to be correct; however, the passage refers details found within the passage.
to the colobus monkey information as being found
40. The best answer is J. The lines in the question
in “one notable study,” not the study of Dr. Goodall.
stem refer to the sixth paragraph, which focuses on
38. The best answer is J. According to the pas- a discussion of food choices among chimpanzees.
sage, humans and chimpanzees share many The paragraph is strictly informational, and does
biological and social qualities. The passage states not express any emotion.
“Researchers suggest that chimpanzees might hunt
PRACTICE TEST 6 ANSWER KEY 477

ANSWER KEY

English Test
1. A 21. D 41. B 61. C
2. H 22. F 42. H 62. G
3. B 23. D 43. D 63. C
4. H 24. G 44. H 64. F
5. D 25. B 45. A 65. B
6. F 26. H 46. G 66. J
7. B 27. C 47. A 67. B
8. J 28. F 48. H 68. J
9. B 29. A 49. C 69. C
10. J 30. G 50. J 70. G
11. C 31. D 51. D 71. D
12. F 32. H 52. J 72. F
13. C 33. B 53. B 73. C
14. G 34. G 54. F 74. F
15. D 35. A 55. C 75. D
16. G 36. H 56. F
17. D 37. D 57. D
18. F 38. J 58. G
19. C 39. C 59. A
20. H 40. F 60. H
PRACTICE TEST 6 ANSWER KEY 479

Reading Test Science Reasoning Test


1. A 21. A 1. C 21. B
2. J 22. G 2. H 22. J
3. C 23. B 3. A 23. A
4. J 24. H 4. G 24. F
5. A 25. C 5. A 25. D
6. H 26. J 6. F 26. G
7. B 27. C 7. C 27. D
8. J 28. F 8. J 28. G
9. D 29. C 9. B 29. D
10. G 30. F 10. G 30. F
11. B 31. B 11. A 31. C
12. H 32. F 12. G 32. F
13. A 33. D 13. A 33. A
14. J 34. J 14. H 34. H
15. C 35. C 15. D 35. D
16. H 36. F 16. H 36. H
17. A 37. B 17. D 37. A
18. G 38. F 18. G 38. G
19. A 39. C 19. C 39. B
20. H 40. G 20. H 40. J
PRACTICE TEST 6 ANSWERS AND EXPLANATIONS 485

ANSWERS AND EXPLANATIONS 7. The best answer is B. Because “species” ends in an


“s,” an apostrophe can simply be added to the end
of the word without adding another “s” to create
English Test Explanations possession. Answer choice C is too wordy. Answer
choice D is wrong for two reasons: first, “species”
PASSAGE I
has to modify “survival,” not the other way around,
1. The best answer is A. The comma is needed to sep- and second, in this example “species” should not
arate the introductory clause from the main clause. be possessive.
The date does not have to be separated by two com-
mas, and a semicolon would indicate two complete 8. The best answer is J. Answer choice H is incorrect
sentences. because the passage already states that the most
recent sighting was in 1999. The rest of the sen-
2. The best answer is H. “Far more rarer” is redun- tence is too vague to answer the question. Answer
dant. Remember that the ACT rewards conciseness. choice G does not make clear if the month in 2002
In addition, to form the comparative, either add the was when the biologists searched the woods or
word “more” or add the “-er,” not both. Always when the bird was spotted again. While answer
choose the grammatically correct response with choice F is grammatically correct and offers new
the fewest words—in this case, answer choice H. information, choice J adds the most specific details
Answer choice G is awkward and wordy. to the passage.
3. The best answer is B. The gerund “leading” does 9. The best answer is B. A comma is used before
not bear tense and so cannot be the main verb of a coordinating conjunction joining independent
the sentence. In this case, the sentence needs a form clauses in a sentence. Therefore, answer choice B is
of the future tense because the search would occur correct, and answer choice C is incorrect. Answer
after the student’s report. Answer choice C, “was choice A adds an extra “it” to the sentence and uses
leading,” is in the past. Answer choice D, “was a semicolon where a comma is needed. The clause
lead,” could never be correct because the past form “later determined,” in answer choice D, does not
of “lead” is “led” and the passive voice does not make sense.
make sense here.
4. The best answer is H. The sentence would be 10. The best answer is J. The topic of the paragraph
improved with a definite pronoun to identify the is the new search for the ivory-billed woodpecker.
habitat. Answer choices F and J are indefinite, The search is instigated by the 1999 sighting, so
and therefore should be eliminated. “Its” is cor- Sentence 1 should remain first. Sentence 2 intro-
rect because “woodpecker” is singular. Answer duces the search and should continue to follow
choice G is correspondingly incorrect because Sentence 1. The conjunction “though” in Sen-
“their” is plural. tence 3, however, indicates a contrast that only
makes sense if it comes after the lack of evi-
5. The best answer is D. The subject of the pas- dence cited in Sentence 4. The fifth sentence
sage and of the paragraph is the ivory-billed starts with the phrase, “In the end,” and should
woodpecker. It’s the only “species” designated remain last.
as such. Therefore, it’s redundant to combine
“species” with “ivory-billed woodpecker” in this 11. The best answer is C. Answer choices A and D
sentence. The ACT values conciseness. Always are too wordy. Answer choice B is more concise,
choose the least-wordy option that is grammatically but would need a semicolon to separate the clauses
correct. because they form two complete sentences. Answer
choice C uses a period to separate the sentences and
6. The best answer is F. In this sentence, it is is the best answer choice.
necessary to use commas to set off the intro-
ductory phrase and the two independent clauses. 12. The best answer is F. Commas should be used
Answer choices H and J use commas to sep- to separate the elements in a series, but it is not
arate short phrases that are not full clauses. necessary to put a comma between the last ele-
Semicolons are used between independent clauses ment in the series and the following verb. So,
not linked by a conjunction, so answer choice answer choices H and J are incorrect. A comma
G would be correct if there was no conjunc- should come between the second to last ele-
tion, “and.” Therefore, the sentence is best as it ment and the conjunction, so answer choice G is
stands. incorrect.
486 PRACTICE TEST 6 ANSWERS AND EXPLANATIONS

13. The best answer is C. The idea here is that choice H) is also unnecessary because there’s no
researchers feared a sudden influx of birdwatch- contrast.
ers drawn by the new sightings. The only selection
19. The best answer is C. The sentence needs a rel-
that matches that meaning is answer choice C.
ative clause to explain what is done with the call
14. The best answer is G. The word “accepts” makes for papers. In this case, the clause needs a subject
the most sense in the sentence because it clearly and a verb in the present tense. Answer choice D
and simply indicates present tense. The phrase “is lacks a subject. Answer choice B is not in stan-
accepting of” may appear to be correct, but it is dard English word order (i.e. subject, verb, object).
rather awkward and would be more appropriate if And finally, the phrase “nationwide research insti-
it referred to evidence of the birds rather than their tutions” in answer choices A and C implies that
actual existence. each institution exists nationwide (as opposed to,
say, a state university). Only answer choice C is
15. The best answer is D. Sentence 3 discusses “sight- complete and grammatically correct.
ings” by researchers in Arkansas. This would lead
easily to information about those sightings, like 20. The best answer is H. “Researchers and authors”
the “15 sightings” mentioned in the extra sentence. forms a compound subject. A compound subject
Sentences 1 and 2 could refer to multiple video requires a plural verb, which this sentence has. It
tapings, but the tapings are from motion-sensing does not need any commas, however, so the correct
cameras and are different from personal sightings. answer is answer choice H.
Therefore, the best place for the new sentence is
21. The best answer is D. This sentence needs a main
after Sentence 3, answer choice D.
clause that contains a subject, verb, and object.
Answer choices B and C would leave an incom-
PASSAGE II plete sentence. Answer choice A has an unclear
antecedent—that is, the sentence does not make
16. The best answer is G. This sentence is made up
clear who is meant by “they.” Only answer choice
of two complete clauses that could both stand
D is clear and grammatically correct.
independently. Therefore, answer choices H and J
would be acceptable ways to write the sentence. 22. The best answer is F. This sentence has two
Answer choice H makes two complete sentences parallel subordinate clauses. Each clause should
separated by a period, while J uses a semicolon have the same structure, so the editors should
to separate them. Answer choice F, “no change,” “read” and “submit” each work. Answer choices G
is also acceptable because the conjunction “and” and J change the form of the second verb. Answer
follows the comma. Answer choice G, however, is choice H is incorrect because, as it stands, the sec-
not acceptable because it uses a comma without a ond clause is not a complete sentence and cannot
conjunction to join two complete thoughts. This is be set apart by a semicolon.
called a “comma splice,” and it is grammatically
incorrect. 23. The best answer is D. The sentence as it stands
(answer choice F) is grammatically correct. The
17. The best answer is D. The sentence is written in next sentence begins with the pronoun “those” that
the present tense, and therefore, the correct form refers back to the subject of the previous sen-
of the verb is “spend.” Answer choice B is also tence. In answer choices A and C, the subject is
wrong because the sentence has a compound sub- the editors. In answer choice D, the subject is the
ject (“editors and I”) which requires a plural verb paper submissions, which is a better match for what
form (“are” instead of “am”). Answer choice C comes next. Answer choice B is too wordy and the
is incorrect because it leaves the clause without a word “either” is unnecessary.
main verb.
24. The best answer is G. The added sentence would
18. The best answer is F. The passage is acceptable be helpful because it would clarify how a paper
as it stands because it doesn’t need a causal con- might be indecipherable. This is why answer
nection between the end of the first paragraph and choice G is correct and answer choice J is incor-
the beginning of the second. In other words, the rect. The form of the papers is relevant (answer
editorial board doesn’t reconvene in the summer choice H) because papers can be returned to the
as a result of the editors’ long hours, so it’s author if the formatting is poor. Answer choice F
unnecessary to say “consequently” or “as a result,” is incorrect because the new sentence does not
as in answer choices G and J. “However” (answer change or establish the tone—the first sentence has
PRACTICE TEST 6 ANSWERS AND EXPLANATIONS 487

already done that by introducing the possibility of PASSAGE III


bad papers.
31. The best answer is D. “Italy,” as it is used here,
25. The best answer is B. The clause needs a finite needs to be put in the possessive. Because the coun-
verb, not a participle. This eliminates answer try is singular and does not end in an “s,” the correct
choices A and C. Answer choice D is incorrect form is “Italy’s.”
because a verb should be modified by an adverb 32. The best answer is H. The passage states that
(“immediately”), not an adjective (“immediate”). Montessori was Italy’s first modern woman physi-
cian. That implies she was trying to advance within
26. The best answer is H. Answer choices F and G are
the profession (answer choice H). Answer choice F
essentially interchangeable. They both imply that
would imply that the male physicians were help-
the most beneficial information for the journal’s
ing her, which is contrary to the statement that she
readers is an example of the professional standards
struggled. Answer choice G would imply that she
of the journal. Therefore, it is best to eliminate
was trying to promote the profession and not her
both choices. Answer choice J implies a contrast
own career, which is not supported by the rest of
between the beneficial information and the profes-
the passage. In answer choice J, the pronoun does
sional standards. This does not make sense within
not appear to refer to anything.
the passage. Answer choice H, however, suggests
that the professional standards refer, at least in part, 33. The best answer is B. A colon is used to introduce
to the format and presentation mentioned earlier, an explanatory phrase if the first part of the sentence
while the beneficial information is an additional creates a sense of anticipation about what follows.
requirement for publication. This makes the best This sentence does not do that, so answer choice A
sense in context. is incorrect. However, the addition does provide
important information about why Montessori was
27. The best answer is C. The simplest option is sent there, so it would be wrong to eliminate it
often the best. In this case, answer choice C has as in answer choice D. The additional information
the fewest clauses and the simplest structure. It is is in a subordinate clause and so needs a comma
grammatically correct and not redundant. to separate it from the rest of the sentence. Answer
choice B is correct.
28. The best answer is F. Adverbial clauses, like the
one that starts this sentence, need to be separated 34. The best answer is G. The passage makes a
from the main clause by a comma. This eliminates distinction between Montessori’s training and the
answer choices H and J. The phrase, “once we processes of the mind. Only answer choice G,
finalize,” indicates a single action that should be “anatomy and physiology,” makes this distinction
followed by another single action to keep paral- clear. Answer choices F, H, and especially J are
lel structure. The phrase, “we’re sending,” implies broad and could include study of the mind. They
repeated or continuing action. Therefore, answer do not make a clear distinction and are incorrect.
choice G is incorrect. Answer choice F has a
comma after “layout,” and “send” is parallel in con- 35. The best answer is A. This clause is specifying
struction with “finalize,” so this is the best answer Montessori’s studies in the processes of the mind.
choice. It needs to be an example of a process. Answer
choices C and D put the emphasis on the children,
29. The best answer is A. The verb “meets” indicates not on the way they learn. Answer choice B has the
that the action is taking place in the present, while right emphasis, but it is unnecessarily wordy.
the journals are being prepared to mail. Answer
choice B is incorrect because the noun phrase 36. The best answer is H. The main clause in this
“editorial board” is singular, and so requires a sin- sentence begins with “Montessori established a
gular verb. Both answer choices C and D create ‘children’s house.’” What comes before is an
incomplete sentences. adverbial clause that needs to be separated from the
main clause by a comma. This eliminates answer
30. The best answer is G. In standard English, adjec- choices G and J, which were also wordy and awk-
tives typically go before the nouns they modify. In ward. Answer choice F is also wordy and difficult to
this case, “well-earned” modifies “vacation,” and follow. Answer choice H makes clear that Montes-
so answer choice G is correct. The other choices sori had sixty children from the working class when
are incorrect or simply awkward. she established her house.
488 PRACTICE TEST 6 ANSWERS AND EXPLANATIONS

37. The best answer is D. The word “ready” actually introduced by relative pronouns. The only choice
refers to how the children learn, and so it modifies of a relative pronoun is “which,” answer choice D.
the verb “learn” not the noun “children.” Therefore,
44. The best answer is H. In simple sentences,
it should be in the form of an adverb, “readily.”
That eliminates answer choices A and C. There is no standard English usually puts the adjective in front
natural pause between “children” and “readily” and of its noun. While answer choice F is not gram-
no grammatical need to put a comma, so answer matically incorrect, it does not match the straight-
choice B is incorrect. forward style of the rest of the passage. Answer
choice G has unnecessary commas, while J is
38. The best answer is J. Sentence 2 establishes awkward and too wordy.
the topic of the paragraph: Montessori’s experi-
ences in her “children’s house.” Sentences 3 and 45. The best answer is A. The fourth paragraph shifts
4 give examples of what she learned. Sentence 1 the passage from a historical discussion of how
describes how these specific experiences influenced Montessori developed her method to a description
her subsequent work. Therefore, sentence 1 should of how Montessori’s method is used by schools
conclude the paragraph and be put after Sentence 4. today. This eliminates answer choice B, which
talks about past development, and answer choice D,
39. The best answer is C. By mentioning both devel- which ignores the shift in topic. Answer choice C
opment and learning, answer choice C combines is wrong because a discussion of Montessori’s
the information in answer choices A and B. There- current significance shows her importance to her
fore, it is more complete and specific than either field, which helps justify a biography of her in the
of them. Answer choice D introduces the idea first place. That leaves answer choice A, which is
of school districts, which does not belong in this correct.
passage.
40. The best answer is F. The topic of the sen-
tence is development, not children. This means the
PASSAGE IV
child’s development is supposed to be compared
to standards and norms, not the child in general. 46. The best answer is G. The verb “immerse” needs
This eliminates answer choices H and J. Answer to agree with the preceding verb “want” in order to
choice G places the apostrophe in the wrong maintain parallel construction. The verb “want” is
place. The correct placement is used in answer actually “will . . . want” and is in the future tense.
choice F. Therefore, answer choice G, which creates “you
41. The best answer is B. This sentence has a com- will immerse,” is the correct answer.
pound verb: the subject “children” is shared by 47. The best answer is A. The underlined portion
“develop” and “acquire.” Commas are not used forms a clause that contrasts (“but”) Paris’ awe-
to separate compound verbs. (Consider the related inspiring cultural treasures with the ease of travel
rule about compound subjects: Bill, and I went on the Metro. Answer choice A works grammat-
to the store is clearly incorrect.) This eliminates ically because it creates two independent clauses
answer choices A and C. Meanwhile, semicolons separated by a comma. To be correct, answer
are used to separate two complete sentences. The choice B would need a period or a semicolon after
final clause is not a complete sentence (it lacks a “treasures” because “France’s capital has cultural
subject), so answer choice D is incorrect. treasures” is a complete sentence. Both answer
42. The best answer is H. To maintain parallel con- choices C and D are noun phrases in need of a
struction with the verb form “provides,” use the verb to make sense.
present tense “observes.”
48. The best answer is H. The underlined section
43. The best answer is D. The second clause in this introduces a nonrestrictive relative clause, which
sentence is a relative clause. That means it is a means it contributes information about what is
subordinate clause that behaves like an adjective referenced (the subway) without restricting the ref-
in that it helps describe a noun phrase. In this erence. In this case, the clause tells the reader the
case, the noun phrase is the fundamental processes name of the subway system but does not distinguish
of the Montessori Method. The subsequent clause it, say from subways in other cities. Nonrestrictive
provides more information about that noun phrase relative clauses are introduced by “wh”-relative
and so acts like an adjective. Relative clauses are pronouns (who, which, etc.) and are frequently
PRACTICE TEST 6 ANSWERS AND EXPLANATIONS 489

separated from the rest of the sentence by a comma. 55. The best answer is C. The verb “to swipe” should
Answer choice G is awkward and ungrammatical. be in the imperative form because the author is giv-
Answer choice F uses the wrong pronoun, while ing a directive. The imperative form of “to swipe”
answer choice J would only be correct if the final is “swipe.”
clause were a complete sentence.
56. The best answer is F. Answer choice F empha-
49. The best answer is C. The gerund “making” marks sizes how a rider can utilize the full extent of the
the beginning of a descriptive phrase that needs to Metro at a very reasonable price. This definitely
be set apart by some form of punctuation. A dash illustrates a tremendous advantage to using the sub-
indicates a sharp break in the continuity of the sen- way system. Answer choice H does this, too, but
tence. A semicolon indicates the start of a new to a lesser extent. Answer choice G is even more
complete sentence. Neither of these occur in this vague, and therefore, less acceptable as an answer
sentence, so a comma is the best choice. choice. Answer choice J points out a disadvantage
50. The best answer is J. The sentence needs a main to using the Metro and should be eliminated.
verb, and “has” fits the sense well. While it is pos-
sible to contract the subject/verb “Metro has” to 57. The best answer is D. Commas are used to sepa-
“Metro’s,” the apostrophe is in the wrong place in rate words or phrases in a series of three or more
answer choice G and missing in answer choice F. items. ACT style always uses a comma between
Answer choice H is incorrect because the Metro is the second-to-last item and the conjunction (usu-
more than just stations; it is a system of travel. ally “and”). Answer choices B and C do not have
enough commas, while answer choice A adds an
51. The best answer is D. Answer choice D illustrates unnecessary “and” before “parking.”
how easy it is to access the Metro from any-
where in Paris. This would provide a good example 58. The best answer is G. In standard written English,
of the Metro system’s expansiveness and cover- the modifier usually goes before whatever it is mod-
age. Answer choice B would contradict the desired ifying. Here, “generally” modifies the adjective
emphasis. While answer choice A would indicate “accustomed” and so should go before it. The main
the Metro is popular, it doesn’t provide proof that verb “to be” should also agree in tense with the
the stations are numerous. Answer choice C is other main verbs. The other verbs are in the present
irrelevant. tense, so this sentence should use the form “are.”
Both of these points support answer choice G.
52. The best answer is J. The phrase “ideal for tourist Because “accustomed” is an adjective, there is no
travel” is all-encompassing. It could be modified need to change its spelling, answer choice J.
by a phrase that restricts the ideal (for example,
“unless you don’t speak French”), but none of the 59. The best answer is A. The author tells the reader
choices does that. Therefore, it is best to omit the that the Metro provides incredible freedom of
underlined portion altogether. movement. Because it’s clear that the author is
53. The best answer is B. Answer choice B correctly talking about the Metro, it is redundant to use
states that the stations took their names from the the phrase “Paris Metro” again, as in answer
adjacent attractions. Answer choice A implies that choices B and C. Answer choice B could also
the attractions named the stations, which would only be correct if the relative pronoun “that” were
be impossible because the attractions are inani- removed. Answer choice D would leave a clause
mate. Answer choice C implies the opposite, which that reads “that providing it.” This doesn’t make
is also impossible. Answer choice D makes no sense because it needs something to come next.
sense. Therefore, answer choice D should be eliminated.

54. The best answer is F. Here, we have a com- 60. The best answer is H. Paragraph 3 talks about the
pound adjective. The author loves the feeling of ease of traveling around the city and the joys of
going underground and coming back to the sur- finding new places. The new sentence continues the
face. The going and coming should not be separated idea of easy traveling but adds the delight of return-
by a comma. “Coming” should also remain the ing to a familiar place. This completes the circuit
gerund form to maintain parallel construction with of a day and leads smoothly to the last sentence:
“going.” “Metro travel couldn’t be easier!”
490 PRACTICE TEST 6 ANSWERS AND EXPLANATIONS

PASSAGE V verb form must agree with a singular subject. That


eliminates answer choice A. Also, the verb needs to
61. The best answer is C. Plentiful and abundant are
be in the past tense to be consistent with “plowed”
synonyms. Therefore, it is redundant to use both
in the preceding clause.
of them in this sentence. Answer choice D is also
grammatically incorrect. 68. The best answer is J. The underlined section
introduces a nonrestrictive relative clause, which
62. The best answer is G. This sentence contains only
means it contributes information about what is ref-
one clause. Therefore, it is unnecessary to divide
erenced (Bord Na Móna) without restricting the
it with punctuation of any kind. The remaining
reference. In this case, the clause tells the reader
answer choices imply strong divisions that are not
the output of the company but does not distin-
supported by the structure of the sentence.
guish it, say, from the output of other companies.
63. The best answer is C. Here it is necessary to Nonrestrictive relative clauses are introduced by
choose the correct preposition to introduce the “wh”-relative pronouns (who, which, etc.) and
rest of the prepositional phrase, “the acidic envi- are separated from the rest of the sentence by a
ronment of these marshlands.” The preposition comma. The relative clause is not a complete sen-
“in” suggests location—that the stuff that makes tence, so answer choices F and G could never be
up peat is kept from decomposing because it is correct.
in the acidic environment. “With” would suggest
accompaniment—that the stuff is sitting in the bog 69. The best answer is C. Answer choice C has what
alongside the acidic environment, which is not cor- is known as a “comma splice.” This means that the
rect. “For” is incorrect because it suggests that sentence is composed of two complete sentences
some other agent is preventing decomposition to joined together by a comma with no separating
maintain an acidic environment. Finally, “among” preposition or conjunction. Answer choice A has
would only be used if there was a plural object in the adverb conjunction “while” after the comma,
and answer choice D has the word “and.” Both
the prepositional phrase.
choices are grammatically correct and acceptable.
64. The best answer is F. “When peat is harvested” Answer choice B simply separates the two sen-
is an introductory clause. It should be separated tences, and so it, too, is acceptable.
from the main clause by a comma. A period or
semicolon, as in answer choices G and H, would 70. The best answer is G. Answer choice G is the most
mean that it is a complete sentence. concise option. It is also consistent with “ridging
machine” used later in the paragraph. In fact, none
65. The best answer is B. Choosing the proper verb of the equipment is defined or described in any
form depends on understanding the sense of the detail. Therefore, “harrow machine” fits best with
sentence. Something is happening to the peat in the language of the paragraph.
the Irish villages. Peat is dug out of wet marsh-
lands and used as fuel. It makes sense that at some 71. The best answer is D. The correct form of the
point freshly dug peat has to dry out. This pro- verb is “passes” to stay consistent with the rest of
cess would have to occur over time, so the correct the paragraph. The word “passes” is used several
verb form would use the participle “drying” to indi- times in the paragraph, and this sentence gives no
cate action over time. This is called the progressive reason to change tenses.
aspect. 72. The best answer is F. Answer choices G and H
66. The best answer is J. The passage’s main idea is would create a comma splice by combining two
the use of peat as a fuel source. The most appro- complete sentences with only a comma. Answer
priate and relevant contrast would be in direct choice F uses the passive form of the verb “to draw”
opposition to that main idea. Answer choice F to correctly describe the harvester as putting the
talks about the amount of peat used, while Answer peat into a collection bin. Answer choice J is incor-
choices G and H state where it is used. These are rect because it uses the active form of the verb. This
details that add to the main idea, not contrast with implies that the peat is putting something into the
it. Only answer choice J offers alternatives to peat bin, which does not make any sense in the context
as a fuel source. Therefore, answer choice J is the of the paragraph.
correct answer.
73. The best answer is C. The phrase “the history” is
67. The best answer is B. First, the verb in this clause too vague. The sentence would have better clues
refers to the peat, which is singular. Therefore, the for the reader with a pronoun referring to the
PRACTICE TEST 6 ANSWERS AND EXPLANATIONS 491

sentence’s subject, “peat.” The correct pronoun is provides the best link between the opening and
“its.” Answer choice B, “it’s,” is a contraction of closing sentences.
“it is.” The verb is not necessary here, and answer
75. The best answer is D. An essay on the evolution of
choice B is incorrect.
power sources suggests that the topic would have
74. The best answer is F. The passage puts peat in some breadth. In other words, it would cover more
the context of Ireland from the very beginning. than one type of power source and more than one
Peat is not just a resource, it is an Irish resource. country. This passage only describes one power
Answer choice F is the only choice that acknowl- source and locates that source in only one country.
edges peat’s role in Irish culture. Therefore, it Therefore, answer choice D is correct.
500 PRACTICE TEST 6 ANSWERS AND EXPLANATIONS

Reading Test Explanations I’m sure I will ever do in my lifetime. My father’s


extreme career and energetic lifestyle made him the
PASSAGE I coolest dad in town, and I had to live up to him,”
which best supports answer choice J.
1. The best answer is A. In the second paragraph,
the narrator states, “I didn’t want to start a trend of 9. The best answer is D. The word “rigging” is a
hair-raising stunts,” but his father continues with well-known sailing term, referring to the ropes and
his plan for a rappelling platform anyway. pulleys used with the sails. By extension, any sup-
2. The best answer is J. The passage focuses on the port system of ropes or similar lines—such as the
fun that the narrator, his father, and his friends one required for rappelling—could be called “rig-
had while building a rappelling platform, even ging.” The other answer choices are mentioned in
though there was some hesitation on the part of the passage, but do not have the same meaning as
the narrator. Only the first paragraph details the “rigging.”
narrator’s father’s popularity, and nothing in the 10. The best answer is G. The passage states that
passage suggests that the narrator is “struggling” “At first, I resisted, since I didn’t want to start a
with it. trend of hair-raising stunts on my neighborhood
3. The best answer is C. The first and second para- street. Who knows what the old lady across the
graph demonstrate that the narrator is more timid street would think?” Answer choice G is the best
than his father when it comes to exhilarating stunts. choice because while the passage focuses on a fun
The narrator is not sure whether he wants to bring experience that the narrator had with his friends, it
his friends to his father for ideas. also describes his apprehension to do anything that
could be unreasonably dangerous or disruptive to
4. The best answer is J. The first sentence of the the neighborhood.
passage establishes a timeframe that reveals the
narrator is no longer a child: “As I was growing
up, . . .” Therefore eliminate answer choices G and
H. The tone of the passage is positive so answer PASSAGE II
choice J is best. 11. The best answer is B. The passage states that
5. The best answer is A. The last sentence of the Robert Catesby, Guy Fawkes, and several other
first paragraph describes the narrator’s father’s Roman Catholics “were thought to be denounc-
“extreme career and energetic lifestyle.” Answer ers of the king’s own Church of England,” and
choice D might seem correct, although the pas- they “decided that unless something was done from
sage notes that the narrator’s father was once a within, nothing would likely change.” These state-
Hollywood stuntman, indicating that he would be ments suggest that some English Roman Catholics
trained in safety procedures, some of which arise did not agree with the king and rebelled against
during his “impromptu lesson on the critical safety him, seeking their own religious freedom. The
rules.” other answer choices are not supported by the
passage.
6. The best answer is H. In the paragraph, the
author describes the pleasure of following his father 12. The best answer is H. The passage asserts that
around the lumberyard, as well as the anticipation the plan was carefully conceived and well exe-
of what all the “shiny, new parts” would serve to cuted, except just before the planned detonation,
build, which best supports answer choice H. when a conflicted conscience led one conspirator
to reveal the plan to a government official, which
7. The best answer is B. After this sentence, the best supports answer choice H.
rest of the passage describes how the narrator’s
friends convinced him to consult his father for 13. The best answer is A. The last paragraph states that
advice on a bigger, bolder stunt. This information “Guy Fawkes Day serves as a chilling reminder”
suggests that this uncommon adventure, at least, for British people. Answer choice D seems plau-
was the result of the narrator’s father’s unusual sible, but the passage does not indicate that mod-
occupation. ern British citizens find fault with (i.e., consider
depraved) Fawkes’ reaction to the king’s policy
8. The best answer is J. According to the passage,
governing religion.
“My father is recently retired from his career as a
Hollywood stunt performer. When I was a child, he 14. The best answer is J. The phrase “tongue-in-
would do more death-defying tricks in a week than cheek” implies humor at the expense of members
PRACTICE TEST 6 ANSWERS AND EXPLANATIONS 501

of parliament. Skepticism of the true motives of 22. The best answer is G. The passage maintains an
legislators is evident in the passage. even, analytical tone, meaning the author does not
attempt to persuade the reader to feel a certain way
15. The best answer is C. The final two sentences about the facts of Williams’ life; the author is inter-
of the passage state that “an individual will press ested in William’s life, but remains detached, or
back . . . in defense of his beliefs.” This best apart, from it. The other answer choices suggest
supports answer choice C. emotions that are not indicated by the tone of the
passage.
16. The best answer is H. The execution was meant
to teach a lesson to the public. The king wanted 23. The best answer is B. The year of publication of
to make a powerful statement against treason, so Williams’ plays mentioned in the passage are (in
he chose an “unusual” punishment to shock the order of appearance) 1948, 1955, 1944, and 1958.
masses. The word “exotic” can refer to something The passage states that the play from 1948, A Street-
that is “unusual” or “unexpected,” so it is the best car Named Desire, won a Pulitzer Prize, perhaps
choice here. the most prestigious literary award in the United
States.
17. The best answer is A. As Fawkes and his Catholic
24. The best answer is H. The prefrontal lobotomy
cohorts saw it, the king was unreasonably tread-
is described as a “now-defunct” (“outdated”) pro-
ing on their right to practice their own religion. In
cedure that was “ruinous” (“destructive”) to Rose,
the second paragraph, the author asserts that the
rendering her a vegetable. The other choices hint
Church of England was “the king’s own,” insinua-
at some benefit to the procedure, but the passage
ting that it held political sway. Hence, answer
asserts that no good to Rose nor to the Williams
choice A is correct because it represents an action
family came from the treatment.
of government taken to preserve its power. The
other choices reflect government actions taken for 25. The best answer is C. The third paragraph
the common good. describes how Williams used family members as
models for his characters. The final paragraph
18. The best answer is G. The passage implies that details how emotionally damaged he was for the
although the threat of punishment may deter most rest of his life. This best supports answer choice C.
people from action, “no brutal threat can stop the Answer choice D may be distracting, but the fact
most committed believer,” which best supports that Williams is “among the most celebrated” of
answer choice G. Southern Gothic playwrights proves that his work
did indeed appeal to his audience.
19. The best answer is A. The passage states that
the conspirators were lucky to find a cellar space 26. The best answer is J. The phrase “blatant accu-
beneath Westminster Palace. A physical sense of sations” refers to transparent rhetoric that might
“stumbled upon,” as in answer choices B and C come from a character taking a stand on the “moral
does not make sense in context. Answer choice D issues” of the past. The richness of Southern Gothic
is incorrect because the decision to seek something literature is due in part to the absence of overt com-
seems unrelated to luck. mentary on such subjects. The reader is left to infer
the morality in the prose from the actions of the
20. The best answer is H. According to the passage, characters. The word “overtly” refers to something
the conspirators were preparing an explosion under- that is open and obvious.
neath the floor of Parliament in order to maximize
destruction. The “mineshaft” refers to the actual 27. The best answer is C. The passage states that
space to be filled with explosives. Williams’ “mother . . . pushed him toward creative
arts during his period of illness.” He “heartily
accepted” her gift of a typewriter. These statements
PASSAGE III best support answer choice C.
21. The best answer is A. The first two paragraphs 28. The best answer is F. According to the pas-
describe the qualities of Southern Gothic literature. sage, Southern Gothic literature “reflects life in
The rest of the passage relates Tennessee Williams’ the American South. It maintains some of the
tumultuous life story, explaining in the process characteristics of Gothic writing, such as use of
how the author’s family members inspired several the supernatural or the ironic; however, Southern
of the his characters, which best supports answer Gothic does not focus on creating tension and sus-
choice A. pense as do other Gothic genres.” In addition, the
502 PRACTICE TEST 6 ANSWERS AND EXPLANATIONS

passage states that “that the Southern Gothic writer jack pines. The principal reason that the bird is
is able to present and explore moral issues of the endangered is the widespread decline of suitable
American South, such as slavery and bigotry . . .” habitat.
These statements best support answer choice F.
34. The best answer is J. The passage states that
29. The best answer is C. A person “depressed and the Kirtland’s warbler “migrates to the Bahamas
fighting alcoholism” can be said to be “embroiled” to winter.” This suggests a temporary stay, much
in personal problems. As for Williams’ characters, like how people in the United States may winter in
which are characteristic of Southern Gothic style, Florida, but still call their northern residence home.
they are brooding and often uninviting, and it The warbler engages in its most complex behavior
is through them that “moral quandaries” (ethical while in Michigan (i.e., building nests and mating),
dilemmas) are analyzed. which best supports answer choice J.
30. The correct answer is F. The word “postbellum” 35. The best answer is C. According to the passage,
means, in general, “post-war,” but it is usually forest policy at the turn of the century dictated that
restricted to mean “post-Civil-War.” With the end all fires should be prevented; as a result, Jack pines,
of the Civil War came the end of slavery, but which require fire to reproduce, suffered, which
antebellum, or pre-war attitudes of “racism and best supports answer choice C.
inequality” persisted, which are what Williams’
characters struggle to reconcile. This best supports 36. The best answer is F. The word “brood” describes
answer choice F. the group of eggs a bird raises at one time. Thus a
“brood parasite” is an organism that preys on the
PASSAGE IV nestling of another bird.

31. The best answer is B. According to the passage, 37. The best answer is B. The fact that the pas-
jack pine “depends on regular fires to expose the sage claims that Kirtland’s warblers are “fas-
seeds within its tough cones.” It is likely that tidious about their habitat” indicates that their
any management of the forests would include peri- habitat requirements are uncommonly rigid as com-
odic, supervised burning in order to encourage new pared to other birds, which best supports answer
growth. choice B.
32. The best answer is F. The passage states that, 38. The best answer is F. The passage states “the
“The early 1900s, though, saw natural fires sup- tree depends on regular fires to expose the seed
pressed by new forest management policies and, within its tough cones.” This indicates the cones are
consequently, jack pine forests quickly dimin- ready prior to the fire, eliminating answer choice G.
ished.” These fire prevention policies stopped all Answer choices H and J are not supported by the
fires, most likely because of the destructive threat passage.
they pose to land, buildings, and infrastructure.
An unforeseen consequence (“externality”) was 39. The best answer is C. The Jack pine problem is
the depletion of fire-dependent jack pines, which cited as the major contributing factor to Kirtland’s
constitute the exclusive habitat of the Kirtland’s warbler decline. These trees are the bird’s habitat.
warbler. Answer choice D might appear to be correct, but the
logging of jack pines is not cited as a reason for the
33. The best answer is D. The passage states that the widespread depletion of the tree; fire prevention is.
warblers are “extremely fastidious [picky] about
their habitat.” Subsequent paragraphs describe his- 40. The best answer is G. The word “percolates”
torical strain on Jack pine forests which host means “drains” in the context of soil, indicating it is
the birds. Answer choices B and C are never “permeable.” This is important to prevent flooding
said to be detrimental to the success of the bird. of nests. The passage continues by indicating that
Answer choice A, while detrimental, is coinciden- Grayling sand supports the plants that the Kirtland’s
tal to the birds’ renewed proliferation among the warbler requires.
PRACTICE TEST 7 ANSWER KEY 563

ANSWER KEY

English Test
1. C 21. D 41. A 61. B
2. F 22. H 42. J 62. J
3. B 23. D 43. C 63. A
4. H 24. F 44. H 64. J
5. D 25. B 45. A 65. A
6. H 26. F 46. J 66. G
7. D 27. D 47. B 67. B
8. F 28. H 48. G 68. J
9. B 29. A 49. D 69. C
10. G 30. G 50. F 70. H
11. D 31. D 51. B 71. B
12. H 32. G 52. H 72. F
13. C 33. A 53. B 73. B
14. F 34. H 54. H 74. J
15. B 35. A 55. C 75. C
16. G 36. J 56. J
17. D 37. B 57. A
18. F 38. H 58. F
19. B 39. B 59. B
20. H 40. J 60. H
PRACTICE TEST 7 ANSWER KEY 565

Reading Test Science Reasoning Test


1. C 21. B 1. A 21. C
2. G 22. H 2. H 22. G
3. D 23. C 3. A 23. A
4. G 24. H 4. J 24. F
5. C 25. A 5. B 25. C
6. H 26. J 6. J 26. J
7. B 27. D 7. D 27. A
8. J 28. G 8. H 28. F
9. A 29. C 9. C 29. C
10. J 30. G 10. J 30. H
11. A 31. C 11. B 31. A
12. G 32. G 12. G 32. J
13. B 33. A 13. A 33. D
14. J 34. H 14. G 34. H
15. C 35. D 15. C 35. B
16. J 36. J 16. H 36. G
17. A 37. A 17. B 37. C
18. H 38. H 18. G 38. H
19. C 39. B 19. D 39. B
20. F 40. G 20. H 40. H
PRACTICE TEST 7 ANSWERS AND EXPLANATIONS 571

ANSWERS AND EXPLANATIONS is more appropriate; therefore, answer choices


A and D may be eliminated. Answer choice C
does not have correct word order, so it may be
English Test Explanations eliminated.
PASSAGE I 10. The best answer is G. Nouns conjoined with “or,”
1. The best answer is C. It is necessary to include as in the underlined portion, stand on equal footing.
a comma between “tedious” and “tiring” because They should not have any intervening commas.
they are coordinate adjectives—they modify the 11. The best answer is D. The word “notepad”
same noun. Answer choice A is incorrect because it implies writing, which is the subject of the next
includes two additional and unnecessary commas. paragraph. As a description of how moments of
2. The best answer is F. The sentence is clear as it literary creativity arise for the author, this answer
is written. The other answer choices would be very choice would provide a good transition between
awkward to read. paragraphs.

3. The best answer is B. This question relies on 12. The best answer is H. The phrase “other times”
your ability to recognize wordiness. The phrase means “on other occasions.” Among the answer
“needless to say” adds little to the meaning to the choices, only H is acceptable because it is plural
sentence and interrupts the natural rhythm of the and lacks the possessive apostrophe.
clause.
13. The best answer is C. Answer choice A may be
4. The best answer is H. This question relies on eliminated for wordiness. Answer choice B is an
your ability to recognize redundancy. “Asking” is adjective, so it cannot modify a verb; eliminate
equivalent to “inquiring,” so answer choices F and it. Answer choices C and D would both be gram-
G may be eliminated. Between answer choices H matical, but answer choice C would form a less
and J, answer choice H is better because it omits awkward chain of modifiers.
the unnecessary phrase “of me,” further eliminating
wordiness. 14. The best answer is F. The word “others” is a noun
meaning “other people;” eliminate answer choices
5. The best answer is D. The future tense is neces- H and J. As the passage in written in present tense,
sary because the feeling of passion the hiker asserts the past tense answer choice G may be eliminated.
will come only after reading the book.
15. The best answer is B. This sentence is simply a
6. The best answer is H. The verb “brings” requires
response to the preceding sentence, which identifies
a direct object (what is being brought) and a
some people’s preferred activity. The introductory
prepositional element (where the direct object is
phrase “For me” is a natural fit for a sentence about
being brought). The pronoun “you” and the phrase
the author’s opposing view.
“closer to peace” are the constituents of the verb
form “bring”; no commas are necessary within the
clause.
7. The best answer is D. The preceding sentence PASSAGE II
identifies what is “no longer” the case. It would 16. The best answer is G. The word “ever” is best
make sense that the next sentence describes what is placed between auxiliary and participle; eliminate
true currently. Answer choices B and D may appear answer choice J. The word “saw” is the simple past
to be equally appropriate, except answer choice B form of “see,” not the participle; therefore, answer
must be eliminated because “now” appears again choice H may be eliminated.
later in the sentence.
17. The best answer is D. The word “urban” is an
8. The best answer is F. Placement before a noun
adjective and should modify the noun “streets.”
indicates that the word must be an adjective,
Answer choices A and B contain an incorrect pos-
eliminating answer choices G and J.
sessive form. Answer choice C may be eliminated
9. The best answer is B. The word “time” should because adjectives do not take the pluralizing “s”
refer to “sunrises and sunsets,” so the plural “times” before nouns.
572 PRACTICE TEST 7 ANSWERS AND EXPLANATIONS

18. The best answer is F. The next-best answer 28. The best answer is H. The “but” heads infor-
choice would be J because both answer choices F mation that is part of the clause. Answer choice
and J have appropriate meaning and word order; F may be eliminated because a comma is nec-
however, answer choice J may be eliminated essary before “but.” Answer choice G may be
because the clause refers to vacation in general, eliminated because the semicolon divides only full
so it must be written in simple present tense. independent clauses. Answer choice J may be elim-
inated because the dash is used to give special
19. The best answer is B. The next two paragraphs emphasis to something, which is not appropriate
explore the differences between dogs in America here.
and dogs in Africa. Answer choice B provides
a general introduction to what is coming up in 29. The best answer is A. Answer choices B and
the essay. C may be eliminated first because they are very
awkward. Answer choice D, while less awkward,
20. The best answer is H. The dog is panting, not the should be eliminated, too, because the placement
owner, so answer choices F and G may be elimi- of “never” after the verb is less acceptable than
nated. The sentence is in simple present tense, so the placement of “never” at the beginning of the
answer choice H is best. sentence, as in answer choice A.
21. The best answer is D. “Dog owners” is a com- 30. The best answer is G. A single comma preceding
pound noun and the subject of the sentence. The “so” is all that is necessary. No comma is necessary
other answer choices incorrectly contain possessive after “so” when it introduces an action resulting
markers on one or both parts of the compound. from some pre-existing condition (e.g., “It rained,
so I stayed home.”); therefore, answer choices H
22. The best answer is H. The preceding sentence and J may be eliminated.
asserts people who have never owned a dog think
dogs are not smart. Only answer choice H reveals
naïveté on the part of the animal.
PASSAGE III
23. The best answer is D. To get a dog to improve
behavior, it is logical that more training is needed, 31. The best answer is D. This sentence states
not less. something that seems at odds with the preceding
sentence. Only “though,” answer choice D, reflects
24. The best answer is F. The sentence addresses a this relationship.
general situation and makes sense in the simple
32. The best answer is G. “Carnaval season” indicates
present tense. Answer choice F is the third-person
that the event comes around at least once a year.
singular simple present form of “happen,” and
The verb “flock” should be in simple present tense
agrees with the subject “a dog.”
to reflect how the clause is a general statement.
25. The best answer is B. This clause should par-
33. The best answer is A. The sentence is clearest
allel the previous clause’s logical subject (“a dog”
as written. Answer choices B and C are comma-
becomes “it”) and tense; therefore, the other answer
spliced run-on sentences. The subject-predicate
choices may be eliminated.
pair in answer choice D does not make sense.
26. The best answer is F. “Stark” is an adjective
34. The best answer is H. Reminding the reader
commonly used with “differences” in order to
of the dangers the author likes to face empha-
emphasize the degree of difference. Here, “starkly”
sizes his “adventurous nature,” as the question
(adv.) modifies “different” (adj.), so answer choice
puts it. The other answer choices do not seem
F is best. The other answer choices have awkward
well-matched to the “extreme” theme the author
word order.
maintains throughout the passage.
27. The best answer is D. “Mutualism” is a kind of
35. The best answer is A. The major clue here is
symbiotic relationship that exists in nature. To say
the initial “If” clause in the sentence. It must be
that “mutualism” between person and dog is tan-
followed by a comma, then a second clause; the
gible means that the exchange of benefits between
other answer choices may be eliminated.
the two is outwardly apparent. Inserting “tangible”
before “mutualism” is supported by the rest of the 36. The best answer is J. Answer choice F has cor-
paragraph, which details exactly how the human rect word order, but a comma would be necessary
and dog profit from each other’s presence. before the gerund “shooting.” The best alternative
PRACTICE TEST 7 ANSWERS AND EXPLANATIONS 573

is answer choice J because it uses the conjunction PASSAGE IV


“and,” which does not require a preceding comma,
46. The best answer is J. What follows a semi-
and maintains past tense from the preceding
colon dividing a sentence must be a well-formed
clause.
clause, containing minimally a subject and a verb.
37. The best answer is B. The “sarcastic grin” from Answer choice J correctly uses “the water” as
the preceding sentence surely indicates that the a subject and “shines” as the appropriate simple
race organizer has his doubts about the author’s present tense verb. Answer choice G would make a
fitness for the activity, but is willing to let him try well-formed clause, but the past tense is not appro-
it nonetheless. Only answer choice B reflects this priate for giving a description of a place that is
attitude. unlikely to be affected by time (like the color of the
water).
38. The best answer is H. The verb “went” may only
47. The best answer is B. The possessive form of the
be modified by an adverb, so the other answer
pronoun “it” is “its.” Note the lack of apostrophe,
choices should be eliminated.
which contradicts the normal rule of adding apos-
39. The best answer is B. Answer choice C may trophe + “s” to singular nouns to create possessive
be eliminated for being a sentence fragment. The forms.
other answer choices are grammatical, but answer 48. The best answer is G. The sentence means to say
choice B is clearest because it is in conventional that the “euphoric swell” is responsible for pushing
word order and has active voice. up “the corners of the mouth” of some hypotheti-
40. The best answer is J. The phrase “This is the cal visitor to the Caribbean. Answer choice G uses
time” is wordy; eliminate answer choice F. The pre- conventional word order and is clearest and most
ceding sentence does a satisfactory job of defining concise.
the time frame. No words are required before the 49. The best answer is D. The verb phrase from the
main clause “I learned … legal”; therefore, answer previous sentence, “sailed blissfully from one cay
choices G and H may be eliminated. to another,” means exactly what this sentence does.
For the sake of brevity, it may be omitted.
41. The best answer is A. Here, “Racing” is the sub-
ject of the sentence, so answer choice C may be 50. The best answer is F. This paragraph describes the
eliminated. Since the race took place in the past, chaotic scene of a sailing crew effecting a change
answer choice A is appropriate. of direction. Answer choice F alludes to this fact
by saying that tacking is “reserved for the experts.”
42. The best answer is J. Because that the race
result was so close, the mention of a photo fin- 51. The best answer is B. A verb in infinitive or
ish seems an appropriate place for an exclamation gerund form would be appropriate here. Answer
point. Answer choice F leaves a sentence frag- choice A lacks “to” before the verb in order to
ment at the end of the paragraph. Answer choices form the infinitive. Answer choice C may be elim-
G and H may be eliminated because they incor- inated because “start” is redundant with “begins.”
rectly link the two clauses, which are not related Answer choice D is not appropriate because omit-
conceptually. ting the mention of yelling takes away from the
chaotic scene the author is trying to create.
43. The best answer is C. The noun phrase “head of
52. The best answer is H. For the sake of clarity
the pack” is a common colloquialism and should
and brevity, the simple infinitive form should be
not be rephrased.
used here.
44. The best answer is H. This answer choice ties the 53. The best answer is B. A clause introduced by
events of the author’s heat with the overarching a subordinating conjunctive adverb such as “along
concept of his penchant for thrill seeking. Surely, with” must be set apart from the sentence with com-
the author would celebrate a successful slide down mas. Eliminate answer choices A and D. Answer
the chute. choice C is incorrect because the past tense verb
45. The best answer is A. An “essay discussing “went” is not appropriate.
extreme sports” would have to include informa- 54. The best answer is H. This answer choice uses
tion on several extreme sports. This essay only conventional word order and is clearest and most
mentions one. concise.
574 PRACTICE TEST 7 ANSWERS AND EXPLANATIONS

55. The best answer is C. Introducing speech or inner 64. The best answer is J. If a clause comes after a
monologue requires a comma before the direct quo- form of the verb “decide,” either the complemen-
tation. Furthermore, if the quotation is a complete tizer “that” or a null (absent) complementizer must
sentence, the first word of it should be capitalized precede the clause.
as usual.
65. The best answer is A. Answer choice B may be
56. The best answer is J. The subject “the unpre- eliminated for wordiness. Answer choice C may be
dictable wind” introduces the main clause, so what eliminated for the lack of a conjunction between
comes before it should anticipate that. Answer any of the elements of the list, as well as incor-
choices F and H fail to do this, and answer choice rect use of the comma after “Titian.” The “Mona
G is ungrammatical; they all may be eliminated. Lisa” is a painting and “Titian” and “Raphael”
are artists, so answer choice D does not make
57. The best answer is A. This sentence is part of the
sense.
narrative of the author’s trip, so it should be in past
tense; eliminate the other answer choices. 66. The best answer is G. At the beginning of the
paragraph, the Louvre is described as “majestic.”
58. The best answer is F. The preceding two sentences
Certainly, a the French king would want a palace
identify that the boat lay adrift at sea for some
to match his extensive art collection. The other
time before the motor was started. The introduc-
answer choices do not reflect this, so they may be
tory word “Eventually,” answer choice F, is most
eliminated.
appropriate.
67. The best answer is B. Charles V is mentioned
59. The best answer is B. This sentence is part of the
earlier in the passage as a previous resident of
narrative of the author’s trip, so it should be in past
the Louvre. Answer choice B describes a durative
tense; eliminate answer choices A and C. Answer
condition—how nice the palace was—in the past.
choice D may be eliminated because it does not
make sense in context. 68. The best answer is J. It is generally inappropri-
ate to begin a paragraph with a sentence that uses
60. The best answer is H. The sentence gives the
one or more pronouns; therefore, answer choice J
author’s ultimate reaction to the absence of wind at
would create the clearest sentence.
the end of his sailing trip. It provides a conclusion
to the details of the paragraph. 69. The best answer is C. This sentence pairs the
number of works of art with the corresponding year
and represents the clearest word order among all the
PASSAGE V choices.
61. The best answer is B. In general, details intro- 70. The best answer is H. The concept of “elitism”
duced by the word “including” must be set apart matches best with “creativity” because they are
from the clause with a comma. both human qualities. The other answer choices are
62. The best answer is J. A subordinate clause headed words for certain kinds of people and would not be
by “but” or “and” must be set apart from the main a good fit in the conjoined phrase.
clause with a comma. Eliminate answer choice 71. The best answer is B. The preceding paragraph
H. Answer choice G would create a run-on sen- focuses on Francois 1 and his renovation of the
tence; eliminate it. Answer choice J is the best Louvre. Therefore, Sentence 1 is a good transition
fit for the sentence because the adverb “ordinar- between the third and fourth paragraphs. Eliminate
ily” describes a general condition that the clause answer choice D, which places Sentence 2 first.
with “but” is going to counter. Careful reading will The three remaining answer choices (A, B, and C)
reveal that “but” (answer choice J) is preferable to all put Sentence 2 in the second position, so look
“and” (answer choice F) in the sentence. at whether Sentence 3, Sentence 4, or Sentence 5
63. The best answer is A. These two sentences are should follow Sentence 2. Because Sentence 2
independent thoughts and cannot be separated with mentions that royalty lived in the sprawling palace
a colon or a comma; eliminate answer choices B (the Louvre), Sentence 3, which discusses the art
and D. The adverb “thus” normally introduces the collection, seems out of place. Eliminate answer
consequence of some pre-stated condition. The two choice A. Sentence 5 effectively joins Sentence 2
sentences addresses by the questions do not have and 6 by indicating that after Louis XIV left, many
that relationship. more people lived in the palace. Sentences 3 and
PRACTICE TEST 7 ANSWERS AND EXPLANATIONS 575

4 chronologically follow Sentence 6, which makes 74. The best answer is J. Answer choice J cre-
answer choice B the most logical selection. ates two incomplete sentences, so it would not
be acceptable. The remaining answer choices are
72. The best answer is F. The “ballooning collection” grammatically correct, and do not change the
mentioned in this paragraph supports inserting an meaning of the paragraph.
introductory sentence like answer choice F at the
head of the paragraph. None of the other answer 75. The best answer is C. The verb should match the
choices are supported by evidence in the paragraph. unconjugated form of “visit” before the conjunction
“and;” eliminate answer choice A. Answer choice B
73. The best answer is B. The correct idiomatic phrase shows “view” as a noun, so it may be eliminated.
is “moved to,” which indicates where the artwork Answer choice D may be eliminated since it would
has been moved. leave “and” without a second part.
PRACTICE TEST 7 ANSWERS AND EXPLANATIONS 583

Reading Test Explanations setting a good example to her more worldly con-
temporaries, had actually forgotten these fine res-
PASSAGE I olutions, and before she was twenty had become
the wife of Sylvester Slack, a young lawyer in a
1. The best answer is C. Although the statement
neighboring town!” This information best supports
in the question uses the word “wedded,” it does
answer choice C.
not simply mean “marriage;” wedded also means
“closely dedicated or attached to.” In this state- 6. The best answer is H. According to the pas-
ment, Clover is implying that Cecy had an attach- sage, Elsie “adored Clover,” which suggests that
ment or affection for pink roses, answer choice she enjoyed spending time with her sister. The
C. Answer choices A, B, and D can be elimi- context of the passage indicates that both girls
nated because there is no reference to an actual enjoyed the pleasure of each other’s company. The
wedding when Clover says “She was rather wed- other answer choices are not supported by the
ded to them.” Clover is simply referring to Cecy’s passage.
fondness for pink roses.
7. The best answer is B. The passage states that “her
2. The best answer is G. Cecy and Clover’s relation- arms and wrists and shoulders … were round and
ship can be implied when the passage writes that dimpled like an infant’s.” The other answer choices
the name of Cecy’s baby was “‘Katherine Clover,’ are not supported by the passage.
after her two friends.” This distinguishes Clover as
8. The best answer is J. The passage mentions
a close friend of Cecy, answer choice G. Answer
several times that Cecy was trying to convince
choice F may appear to be correct; however, when
Clover to use a spray of pink roses on her dress.
the passage identifies Clover as a friend of Cecy, it
When Clover is discussing the type of flowers to
is clear that they are not sisters.
use, she states that she wouldn’t use the spray
3. The best answer is D. This statement indicates that of pink roses because “it cost lots, and I didn’t
Elsie is pleased to be sewing ruffles onto the skirt want it at all … fresh flowers are a great deal
in which her sister Clover is about “to burst upon prettier, anyway, than the artificial ones.” This
the world.” The fact that Clover is preparing to clearly indicates that Clover did not want to fol-
“burst upon the world” tells us that Clover is attend- low her friend Cecy’s advice on what type of
ing a sort of coming-of-age or milestone event. flowers to use on her dress. The other answer
Answer choice A simply discusses Clover’s visions choices are not supported by details found in the
of her first party dress. Answer choice B is incor- passage.
rect because although it discusses girls “coming
9. The best answer is A. As stated in the passage,
out,” it does not specifically refer to Clover. Answer
“Elsie, who adored Clover, considered her as beau-
choice C is incorrect because Elsie is dreaming
tiful as girls in books.” Later in the passage, Elsie
of her future, which has little to do with Clover’s
is described as “the adoring Elsie.” This best cor-
upcoming milestone event.
relates to answer choice A, “admiring,” which is a
4. The best answer is G. This statement indicates synonym for “adoring.” The other answer choices
that Clover’s eyes were always her most notable are not supported by descriptions of Elsie’s feelings
source of beauty; therefore, answer choice F can towards Clover.
be immediately eliminated. The passage states that
10. The best answer is J. In the last passage, Elsie
Clover’s eyes, with their pathetic look, “made
expresses her approval of Clover’s flower choice
them quite irresistible to anyone with a tender or
and how “lovely” she thinks that they will be. Elsie
sympathetic heart.” This indicates that her eyes
also discusses the dress that she will wear when
were compelling to kind and compassionate peo-
she is old enough to “go to parties and things,” and
ple, which best correlates with answer choice G.
how it will be “interesting” when both Clover and
Answer choices H and J are not supported by details
herself go to pick it out. These facts both indicate
found in the passage.
that Elsie is both interested in and appreciative of
5. The best answer is C. The passage states that, Clover’s opinion, answer choice J. Answer choice
“Yes, the excellent Cecy, who at thirteen had F is incorrect because the last paragraph highlights
announced her intention to devote her whole life the fact that Clover is of age to attend parties and
to teaching Sunday school, visiting the poor, and the like, while Elsie is not.
584 PRACTICE TEST 7 ANSWERS AND EXPLANATIONS

PASSAGE II Therefore, answer choice C is correct. All of the


other answer choices are supported by details found
11. The best answer is A. In reference to Smith’s
in the passage.
Wealth of Nations, “the book is thoroughly stud-
ied; for the theory within, Smith’s magnum 16. The best answer is J. Those people who criti-
opus remains controversial.” The phrase “magnum cize the Wealth of Nations “argue that untempered
opus” means “a great work or masterpiece,” which self-interest is immoral,” or in other words, exces-
makes sense in the context of the passage since sive self-interest is corrupt or unethical, answer
Smith’s Wealth of Nations is portrayed as being a choice J. The other answer choices are beyond the
far-reaching economic treatise that is studied by the scope of the passage and are not illustrated as being
masses. reasons for criticisms of Adam Smith’s Wealth of
Nations.
12. The best answer is G. The passage states that
“Adam Smith asserts the power of the ‘invisi- 17. The best answer is A. According to the pas-
ble hand,’ the notion that a society benefits from sage, “Standing as a testament to his benevolence,
people acting in their own self-interest, without Smith bequeathed much of his wealth to char-
regard to community service.” This corresponds ity.” “Benevolence” is a “tendency to perform kind
with answer choice G. The other answer choices or charitable acts,” which best supports answer
are not supported by details found in the passage. choice A. Answer choices B and D can be elim-
inated because it is implied that Smith voluntarily
13. The best answer is B. The passage discusses
donated much of his wealth; the words “obligatory”
Smith’s theory of markets and the “natural price”
and “requisite” imply that Smith was required to
of goods and services, stating that “For example,
donate his money. Answer choice C does not make
should a product be in excess production, its mar-
sense within the context of the passage.
ket price would fall.” If a decrease in market price
is caused by having too much of a product, you 18. The best answer is H. Smith’s quotation begins
can safely assume that an increase in market price by stating that man “intends only his own gain,”
would be caused by a shortage in production of which insinuates that people are naturally selfish
a product. Therefore, answer choice B is the best and are out for their own interests. This elimi-
choice. Answer choice A is incorrect because the nates answer choice J. The quotation continues on
passage states that stock is reduced if a product to say that man is “led by an invisible hand to pro-
is in excess production, not during a shortage of mote an end which was no part of his intention.”
that product. Answer choice C may appear to be This statement tells you that people are led towards
correct; however, Smith’s theory states that “nat- unintentionally fulfilling certain objectives by the
ural prices” include a “reasonable profit” but does “invisible hand.” This suggests that the “invisi-
not indicate whether profit margins fluctuate with ble hand” directs the actions of selfish individuals
excesses/shortages of products. and creates an outcome—one that ultimately ben-
efits society—not anticipated by the individual.
14. The best answer is J. The passage tells you that
Answer choice G may appear to be correct, but
“the butcher does not cut meat because the com-
the quotation does not indicate that the “invisible
munity desires it, but rather because it is a means
hand” discourages people from following their self-
to earn money … The buyer of a steak values the
ish interests; it merely says that the hand promotes
steak more than his money, while the butcher val-
some actions that are unintentional for man.
ues the money more than the steak.” Applying this
to the baker, you can infer that the baker bakes in 19. The best answer is C. When discussing the the-
order to earn money and that the community mem- ory of natural price, the passage states that “This
bers value the bread more than their money. This idea would become central to the doctrine of the
corresponds with answer choice J. All of the other laissez-faire economists several generations later.”
answer choices contradict the example set forth in This supports answer choice C. The other answer
the butcher illustration. choices are not supported by the passage.
15. The best answer is C. The passage states that 20. The best answer is F. Answer choices G and
Smith believed that the “natural price” of goods and J can be eliminated immediately; Smith was an
services should equal “the production cost plus a advocate for “low government intervention” and
reasonable profit.” Since “profuse” means “extrav- believed that markets recover from recession “with-
agant or excessive,” Adam Smith would not see out intervention on the part of government or
a profuse amount of profits as being “reasonable.” similar regulatory bodies.” Answer choice F is
PRACTICE TEST 7 ANSWERS AND EXPLANATIONS 585

the best answer choice because in a competitive 25. The best answer is A. The passage states that
market, there are many options for comparable Nyman “layers repetitive melodies … to create an
products. This can be paralleled to Smith’s hypoth- atmosphere of soaring highs and endless sorrows.”
esis on excessive production of products—the more It was these emotional melodies that were said
available a product is, the lower its market price to put the audience into a trance, answer choice
falls. Therefore, competition would be a good thing A. Answer choice B is incorrect because Nyman
for consumers in keeping prices reasonably low. including “soaring highs,” and finished the film
“on an uplifting note.” Likewise, the other answer
choices are not supported by the passage.
PASSAGE III
26. The best answer is J. The author describes Nyman
21. The best answer is B. The closing sentence of the as “one of the great writers, conductors, and per-
passage states that “Though rarely awarded for his formers” of his type of music; the author also
many accomplishments, Nyman certainly remains describes Nyman’s film scores as “stunning.” The
an important figure in the innovation of classical author also identifies the score to The Piano as
music and represents a substantial reason why its being one of the best-selling film musical record-
popularity persists.” This conveys both the sense ings ever. Answer choices F and G can be elimi-
that Nyman has produced important music and the nated because the author has clearly distinguished
fact that Nyman is underappreciated because he is Nyman as worthy of critical acclaim, and the fact
“rarely rewarded.” Answer choice A may appear that his score to The Piano has been so success-
to be correct; however, the passage states that “in ful proves that his music does appeal to modern
spite of its waning limelight …” This indicates audiences. Answer choice J makes the most sense
that the fading popularity of classical music is still within the context of the passage.
occurring, and is not a thing of the past.
27. The best answer is D. The word “snub” means “to
22. The best answer is H. The first paragraph states rebuff or refuse to acknowledge,” and the author
that classical music has been “pushed aside by such describes the score to The Piano in a very positive
contemporary forms as jazz, rock, and rap.” This light. From this, you can infer that although the
describes the other types of music with which clas- author believed that Nyman’s work was very good
sical music has come to share an audience. The and deserving of an Academy Award nomination,
first paragraph also states that classical music is Nyman was not so well received by the Academy.
“always the subject of experimentation on the part The other answer choices are not supported by the
of composers and performers,” and the passage also context of the passage.
mentions the “various schools of composition that
emerged in the 20th century.” This provides basis 28. The best answer is G. The passage states that
for the fact that classical music has not stopped “In spite of its waning limelight, however, classi-
evolving, and these points together provide the best cal music occupies an enduring niche in Western
support for answer choice H. culture, always the subject of experimentation on
the part of composers and performers.” This asser-
23. The best answer is C. The passage states that tion highlights both the fact that classical music
“Nyman’s compositions vary greatly in mood and has never disappeared, and the fact that classi-
orchestration, but generally reflect the character- cal music is always changing and evolving due to
istic tenets of minimalist fare.” The context of the ceaseless experimenting of composers and per-
that statement suggests that Nyman’s compositions formers. Although answer choice H may appear to
typically reflect the style of the collection of min- be correct, this choice focuses only on the music of
imalist music. The other answer choices are not Michael Nyman and not classical music as a whole.
supported by the passage.
29. The best answer is C. It is stated in the pas-
24. The best answer is H. Answer choice H states that sage that Michael Nyman “emerged as” one of the
“These works, though, would not reach his largest great originators of “experimental and often min-
audience,” indicating that the works of Nyman’s imalist pieces of music.” The passage goes on to
band were not largely popular. Nyman’s popularity state that it was Nyman himself “who coined the
came later with his score for The Piano. Answer term ‘Minimalism.’” This indicates that Minimal-
choice J may appear to be correct; however, answer ism was still very new when Nyman emerged as a
choice J calls attention to the high point in Nyman’s composer, being that it had not even been formally
commercial success, not the fact that his popularity named yet. However, although Nyman gave a for-
is limited. mal name to the musical style, he did not create
586 PRACTICE TEST 7 ANSWERS AND EXPLANATIONS

this musical style. Therefore, answer choice D is uses of the mortar when constructing the Great
not the correct answer. Pyramid.
30. The best answer is G. The passage states that 35. The best answer is D. Throughout the pas-
“Nyman layers repetitive melodies played on string sage, there are several references to researchers
instruments to create an atmosphere of soaring not having conclusive facts and making the best
highs tempered with sorrow, but these melodies hypotheses they can regarding their findings. For
overcome hopeless melancholy to finish on an example, the fourth paragraph begins with the
uplifting note. The austere blues and greens of statement “Examining the mortar from the Great
the film’s cinematography blend with Nyman’s Pyramid assists scientists in making inferences
round melodies to impose a trance on the audi- about Egypt’s past,” and goes on to discuss the con-
ence, infusing a cold future reality with vivid clusions researchers drew based on the findings that
romance.” The word “austere” means “stark” or they had discovered. Answer choice A may appear
“severe.” The statements suggests that the sever- to be correct; however, the information given on
ity of the film’s cinematography blended with radiocarbon dating does not provide an exact date
Nyman’s softer, “rounder” melodies to appeal to of construction. Rather, scientists had to conjec-
the audience’s sense of romance. The other answer ture about the precise date the Pyramid was built,
choices are not supported by the passage. which supports answer choice D. Answer choice
B is incorrect because the passage states that after
PASSAGE IV analysis of the gypsum mortar, “even using modern
techniques, scientists have been unable to repro-
31. The best answer is C. “Fascinating” and “intrigu- duce it.” Answer choice C is beyond the scope of
ing” are synonyms of one another, both meaning the passage; there is no evidence that the conclu-
“interesting or exciting.” Likewise, “contentious” sions of scientific researchers and archaeologists
and “controversial” are synonyms of one another, usually vary.
both meaning “causing disagreement.” Answer
choices A, B, and D do not have pairs of words 36. The best answer is J. The passage states that
that mean the same thing as “fascinating” and Egypt “has few natural fuel sources available,”
“contentious.” and goes on to say that “The shortage of natural
fuel sources would most likely have made the cre-
32. The best answer is G. The passage states that ation of lime mortar highly uneconomical.” This
mortar used in the construction of the Great Pyra- best supports answer choice J. It is stated in the
mid was found to be formed of “particles of pollen, passage that limestone was both more durable and
charcoal, and other organic matter,” and that most more abundant than gypsum, thus answer choices
of the mortar collected was made of “processed F and G can be eliminated. The inferences drawn
gypsum with traces of sand and limestone.” Answer by the researchers are irrelevant to the choice of
choice G, processed limestone, is the only material material that the Egyptians used in constructing
not explicitly listed as composing the mortar of the the pyramids; therefore, answer choice H is also
Great Pyramid. incorrect.
33. The best answer is A. The passages tells you 37. The best answer is A. The passage states that
that gypsum needs a temperature of 265 degrees “Instead of clarifying or expunging older theories
Fahrenheit to be turned into mortar; on the other about the Great Pyramid, the results of the study left
hand, “to make lime mortar, extremely high the researchers mystified.” The word “expunge”
temperatures around 1800 degrees Fahrenheit are means “to get rid of or remove,” so within the con-
needed.” This best supports answer choice A. text of the passage, the results of the study did not
Answer choices B, C, and D are not supported by clarify or eliminate any older theories about the
details found in the passage. Great Pyramid, answer choice A.
34. The best answer is H. The passage states that 38. The best answer is H. The passage states that
the gypsum mortar used in constructing the Great from radiocarbon dating, it was found that the aver-
Pyramid “was not used to bond the heavy stone age date of construction for the Great Pyramid was
blocks together.” This fact supports answer choice 2977 BC. The passage goes on to state that this
H. The passage also goes on to state, “The gypsum discovery was disputed because this date asserted
mortar’s role was to buffer the joints and to reduce that the Pyramid was built “over 400 years ear-
friction as the enormous blocks were placed.” This lier than most archaeologists originally believed.”
tells you that answer choices F, G, and J all were If archaeologists believed the Pyramid to have been
PRACTICE TEST 7 ANSWERS AND EXPLANATIONS 587

constructed 400 years after 2977 BC, the original the researchers were looking to learn from studying
date of construction was, therefore, assumed to be the mortar.
approximately 2577 BC.
40. The best answer is G. The passage states
39. The best answer is B. As stated in the passage, that the gypsum “would have been easier to
“researchers began studying the mortar from the mine than limestone.” This best correlates
pyramid, hoping it would reveal important clues with answer choice G. Answer choice F may
about the pyramid’s age and construction.” This appear to be correct; however, the passage
supports answer choice B. The topics of the other does not discuss whether a more water-soluble
answer choices are discussed throughout the pas- material would be advantageous when making
sage; however, these topics are not the chief topics mortar.
PRACTICE TEST 8 ANSWER KEY 651

ANSWER KEY

English Test
1. C 21. B 41. A 61. C
2. F 22. F 42. J 62. J
3. D 23. C 43. C 63. B
4. F 24. H 44. F 64. H
5. D 25. C 45. D 65. D
6. H 26. F 46. G 66. G
7. A 27. D 47. B 67. D
8. J 28. F 48. F 68. F
9. B 29. D 49. C 69. D
10. F 30. G 50. F 70. F
11. C 31. C 51. D 71. A
12. J 32. G 52. G 72. G
13. B 33. D 53. C 73. C
14. G 34. J 54. G 74. G
15. A 35. B 55. A 75. B
16. J 36. J 56. F
17. B 37. B 57. A
18. F 38. J 58. H
19. D 39. D 59. C
20. G 40. F 60. H
PRACTICE TEST 8 ANSWER KEY 653

Reading Test Science Reasoning Test


1. B 21. C 1. A 21. A
2. F 22. J 2. H 22. J
3. D 23. A 3. D 23. B
4. G 24. F 4. G 24. G
5. B 25. D 5. A 25. B
6. H 26. G 6. H 26. C
7. A 27. D 7. B 27. A
8. J 28. J 8. H 28. H
9. C 29. A 9. C 29. C
10. F 30. H 10. F 30. J
11. A 31. D 11. C 31. B
12. H 32. F 12. F 32. A
13. D 33. C 13. B 33. A
14. F 34. J 14. J 34. F
15. B 35. C 15. A 35. A
16. J 36. H 16. H 36. H
17. A 37. B 17. B 37. C
18. F 38. J 18. G 38. F
19. D 39. B 19. C 39. C
20. H 40. H 20. J 40. H
PRACTICE TEST 8 ANSWERS AND EXPLANATIONS 659

ANSWERS AND EXPLANATIONS

English Test Explanations

PASSAGE I
1. The best answer is C. The modifier that should be choice J adds to that idea without being redundant,
used to reference a specific year; because the actual while also being grammatically correct.
year is not stated in the passage, the underlined por-
tion should be changed to this, which clarifies the 9. The best answer is B. The phrase needed to be
sentence and provides context. It is not appropri- remedied immediately is a nonrestrictive clause.
ate here to use the article a before the noun year, This means it could be eliminated from the sentence
so eliminate answer choice B. Omitting the under- and the sentence would still make sense. Nonre-
lined portion creates an incomplete sentence, so strictive clauses take the pronoun which. Answer
eliminate answer choice D. choice C is wrong because deterring modifies qual-
ities and therefore should not be separated from it
2. The best answer is F. The adjective careful by a comma.
correctly modifies the verb phrase monitoring
of. This choice also indicates the writer’s direct 10. The best answer is F. Answer choice F implies
involvement—answer choices G and H are not con- sophistication and detailed preparation, both of
trolled by the writer, and answer choice J does not which indicate time-consuming planning. Answer
indicate any action on the part of the writer. choices G, H, and J are vague. They add very little
to communicate the writer’s message.
3. The best answer is D. It is correct to use the
past tense verb instigated to match the tense used 11. The best answer is C. Answer choices A, B, and D
throughout the paragraph. all indicate a sequence of time where the writer first
goes to the hardware store and then arranges her
4. The best answer is F. This sentence is correct as
tools, which she logically implies were purchased
written. It is in the active voice and correctly uses
at the store. Answer choice C reverses the sequence
the infinitive to witness. Answer choice G suggests
and, therefore, doesn’t make sense.
that witnessing the transformation was the method
by which the author arrived home, which doesn’t 12. The best answer is J. Answer choices F, G, and H
make sense. Answer choices H and J are awkward convey the same idea of a difficult task. While some
and do not effectively convey the intended idea. are wordier than absolutely necessary, all three are
5. The best answer is D. The phrase upon our return grammatically correct. Answer choice J, however,
is redundant because the writer has already stated is neither grammatically correct, nor idiomatic.
that they had arrived back home. Answer choices 13. The best answer is B. Only answer choice B
B and C are also redundant for the same reason. matches the tone of the rest of the passage, while
6. The best answer is H. The phrase the local deer providing the same degree of description. Answer
population correctly identifies the subject of the choice D is too wordy and awkward, while answer
clause. The pronoun they is an unnecessary addition choices A and C do not provide the level of detail
in both answer choices F and G. In answer choice offered in the rest of the passage.
J, they would have referred logically to the plants,
14. The best answer is G. The descriptive clause swift
which doesn’t make sense.
and beautiful leapers should refer to the subject of
7. The best answer is A. The focus of the para- the sentence. Only answer choice G has the subject
graph is the condition of the vegetable garden. deer. In the other choices, the subject is the newness
The writer is painting a detailed picture of the and appearance of the enclosure, which cannot be
garden, so the level of detail is appropriate. The modified by the clause swift and beautiful leapers.
construction of the garden fence comes later and
is irrelevant at this point, which eliminates answer 15. The best answer is A. This question really asks
choices B and C. which preposition should follow we sit down. A per-
son sits down to eat something—in this case, a
8. The best answer is J. The writer’s use of the word variety of vegetables. Simplifying the sentence
comical conveys the idea of humor. Only answer allows you to hear the correct idiomatic form.
660 PRACTICE TEST 8 ANSWERS AND EXPLANATIONS

PASSAGE II 24. The best answer is H. The underlined portion


introduces a subordinate clause that explains the
16. The best answer is J. The idea of surprise is
action of the main clause (i.e. the bird has stopped
already expressed by the word unexpected. There-
singing because it’s exhausted). All of the answer
fore, surprisingly is redundant and should be
choices could be used in a subordinate clause,
omitted.
but, apart from answer choice H, they would need
17. The best answer is B. The adverb truly is used here additional words to make grammatical sense. For
as an interjection to express wonder. Answer choice example, answer choice J would have to read:
B changes the tone to something more objectively and so indicates in order to make sense. Answer
descriptive. Because of this change, the choice is choice H contains a participle clause that stands on
NOT acceptable. its own. The addition of the relative pronoun that
(answer choice F) is unnecessary.
18. The best answer is F. The clause that begins as they
enjoy is not a complete sentence. Therefore, answer 25. The best answer is C. The added sentence gives
choices G and J should be eliminated. Answer more detail about the songs that the writer begins
choice H places a comma after as, which breaks up to describe in sentences 4 through 6. Therefore,
its clause. However, the phrase Viewing these birds answer choices A and B are too early in the para-
can be difficult is a complete thought and can be graph to be correct. By sentence 7, the topic has
separated from the following clause by a comma. shifted to why the songs have ended. Only sentence
6 (answer choice C) places the additional informa-
19. The best answer is D. The beginning of this para-
tion in the correct context with other details about
graph focuses on a physical description of the
the Wood Thrush songs.
Wood Thrush and where it likes to perch. Without
the information about migration, a reader might 26. The best answer is F. In the next paragraph, the
assume these details explain when the bird is seen writer describes the bird’s continuing urge to sing,
in the United States. This would be misleading, even after each act exhausts him. Only answer
because the real explanation is the bird’s migration choice F introduces that idea to the reader. The
pattern. Therefore, D is the correct answer choice. other answer choices are either irrelevant, like
answer choice H, or mis-focused, like answer
20. The best answer is G. The singular possessive pro-
choices G and J.
noun refers back to a singular subject: the Wood
Thrush. Therefore, answer choices H and J are 27. The best answer is D. The underlined portion is
incorrect. The word it’s is a contraction of it is, in the main clause of the sentence and needs an
so answer choice F is incorrect. active verb. Because the writer is using the present
tense, only answer choice D, will start, is consis-
21. The best answer is B. This introductory clause
tent. Answer choices B and C are forms of the past
needs a preposition to indicate duration of time.
tense. Answer choice A is a participle and would
Answer choices A, C, and D all have preposi-
only be appropriate in a subordinate clause.
tions that place the action of nesting in a tem-
poral sequence. Only answer choice B lacks the 28. The best answer is F. The phrase harmonious musi-
preposition, which makes the clause awkward and cal arrangement is the most specific and precise
incomplete. because it gives the most detail about the sound
itself. The other answer choices are vague and less
22. The best answer is F. The underlined portion is a
descriptive.
complete thought. This means it does not require
commas to separate its ideas. Answer choice H 29. The best answer is D. The chosen female and
places too much emphasis on in distinct parts, subsequent offspring belong to the Wood Thrush
which is not supported by the focus of the sentence. male. Therefore, the possessive form needs to be
used. Answer choices B and C are incorrect because
23. The best answer is C. The writer wants to give the
chosen is an adjective modifying female, indicat-
idea of the listener learning new things about the
ing that the female is the Thrush’s mate; the verb
Wood Thrush’s song over time. All of the answer
phrase, are begging, comes later.
choices have a sense of duration except answer
choice C. Answer choice C implies that the listener 30. The best answer is G. The subordinate clause
learns something at the beginning of the song only; describes what the female and offspring are beg-
the learning process does not continue as the song ging the male to do. Consequently, the infinitive
continues. Therefore, answer choice C is the correct clause to keep singing is correct. Answer choice H
answer choice. implies that the female and offspring are the ones
PRACTICE TEST 8 ANSWERS AND EXPLANATIONS 661

singing. Answer choices F and J are awkward and 38. The best answer is J. The simple past tense was
grammatically incorrect. reunited is consistent with the tense used in the rest
of the passage.
PASSAGE III 39. The best answer is D. The interjections in answer
choices A, B, and C do not add necessary informa-
31. The best answer is C. The singular noun America
tion to the passage. More importantly, they are too
must be in the possessive form.
casual and do not match the academic tone of the
32. The best answer is G. The underlined portion con- rest of the passage.
tains the sentence’s main verb. Answer choices F,
H, and J all have active voice verbs that would be 40. The best answer is F. Only answer choice F is
acceptable in the passage. Answer choice G has grammatically correct. Answer choices G and H are
only an –ing participle, being, which doesn’t work wrong because they create incomplete sentences.
as a main verb, and is NOT acceptable. Answer choice J places an unnecessary comma
after in addition to, which makes it grammatically
33. The best answer is D. The writer explains that incorrect.
other members of the group wrote the journals.
Therefore, no modifier is necessary before the word 41. The best answer is A. The best answer choice
journals, and answer choice D is correct. is A because the final clause directly explains
why her successful negotiations were important.
34. The best answer is J. The addition is not neces- While the whole sentence is consistent with the
sary because it is off topic. Both the preceding first paragraph, the final clause does not provide
paragraph and the following paragraph focus on a direct link, so answer choice B is not correct.
Sacajawea’s life and personality. Clark’s role may The clause is not humorous (answer choice C) or a
be true and interesting, but it doesn’t really clarify lengthy description of Sacajawea’s family (answer
Sacajawea’s role. Instead, the addition distracts choice D).
the reader from Sacajawea’s contribution and is
unnecessary. 42. The best answer is J. Because 1812 is directly
followed by the phrase as the year she died, answer
35. The best answer is B. The original sentence con- choices F and H are redundant. Answer choice G
tains two complete thoughts with two main verbs. does not make sense.
Connecting these two clauses with a comma is
called a comma splice and is grammatically incor- 43. The best answer is C. The sentence has to specify
rect. Answer choice A creates two separate sen- what the books and instruments were falling into,
tences, which is acceptable. Answer choices C and so answer choice D is incorrect. The pronouns in
D turn the second half of the sentence into a subor- answer choices A and B are unclear referents. Only
dinate clause, which can be separated from a main answer choice C correctly explains where the books
clause by a comma. and instruments were going.
36. The best answer is J. The most logical arrange- 44. The best answer is F. Only answer choice F accu-
ment would place the two verbs close together rately summarizes the main idea of the passage.
so that the reader could easily follow the action. Answer choice G is incorrect because it focuses
Therefore, answer choices G, H, and J are all prefer- on a detail, and it also contradicts the passage,
able to F. Adjectives should also be placed close to which claims Sacajawea’s lifespan is controver-
the noun they modify. Therefore, answer choice J sial. Answer choice H also focuses on a detail:
is better than G and H because westward is kept Sacajawea’s role as mother. While the passage does
closest to trek in answer choice J. While H does mention Sacajawea’s calm manner several times
keep the two verbs the closest, its construction is (answer choice J), her demeanor is only important
too awkward to be the best answer choice. because it enabled her to be an important influence
on American history.
37. The best answer is B. Its is a possessive pronoun
modifying roster, but refers back to the singular 45. The best answer is D. Answer choice D is correct
noun expedition. It’s is a contraction of the sub- because it places the information about the end
ject/verb it is, which makes no sense here. Answer of Sacajawea’s life at the end of the passage.
choice D is incorrect because it includes an extra- This way, the passage presents its information in
neous comma and their is plural. Because it is a consistently chronological fashion. It also leads
a singular pronoun, its’ does not exist in correct nicely into the last paragraph, which refers to the
English. controversy introduced in Paragraph 4.
662 PRACTICE TEST 8 ANSWERS AND EXPLANATIONS

PASSAGE IV tives. Answer choice C suggests a contrast, which


doesn’t make sense in this context.
46. The best answer is G. To take something by storm
is an idiomatic expression meaning to overcome 54. The best answer is G. The underlined verb needs
suddenly and with great force. None of the other to be in the present tense to be consistent with the
answer choices make sense in this context. rest of the passage. Answer choice J contains a mis-
taken form of should have, which is never correct
47. The best answer is B. The underlined portion con- English.
tains the main verb of the sentence. Therefore,
answer choice D is incorrect because it contains 55. The best answer is A. The modifier which correctly
a relative pronoun, which would make the phrase a specifies that the writer is referring to a unique, but
relative clause. Answer choices A and C are vague unspecified, number. That is inappropriate because
– the reader does not know who is doing the need- it would indicate that the writer had a specific num-
ing. Answer choice B, however, contains a pronoun ber in mind, so eliminate answer choice B. The
that means “anyone.” This makes perfect sense in article a is too vague, and doesn’t give any direction
the context of the sentence. to the reader, so eliminate answer choice C. Omit-
ting the modifier creates an awkward sentence, so
48. The best answer is F. The sentence in question eliminate answer choice D.
is important because it explains how to play the
game of Sudoku, and Sudoku is the subject of the 56. The best answer is F. This choice introduces the
passage. Answer choice G is incorrect because it idea of a hypothetical mistake that the reader can
does not provide a solution to any particular game. then correct. This makes logical sense within the
Answer choices H and J are incorrect because the scope of the paragraph, which focuses on making
sentence is both relevant and to the point. mistakes while playing Sudoku. Answer choice G
does not give a reason for the correction; therefore,
49. The best answer is C. In this sentence, the solver’s it’s unclear and should be eliminated. The same is
task is the subject and is is the main verb. It is true of answer choice J. Answer choice H intro-
unnecessary to separate them with any punctuation. duces a time constraint that is not supported by the
passage.
50. The best answer is F. The subject here is a level,
which is singular. Therefore, answer choices G and 57. The best answer is A. Introductory dependent
H, which use plural verbs, are incorrect. Answer clauses must be separated from the main clause
choice F is also correct because it uses the present by a comma. Therefore, answer choice D should
indicative is. be eliminated. The preposition and participle (By
attacking) also make the conjunctions and and so
51. The best answer is D. In this sentence, become is unnecessary. Therefore, answer choices B and C
the main verb and is modified by the adverb even- should be eliminated.
tually. Answer choices A and B use the incorrect
58. The best answer is H. The underlined word some-
form of the adverb. Answer choice C turns become
times introduces the idea of irregularity. Euler was
into the gerund becoming, which is incorrect for a
not occasionally Swiss nor did he invent Sudoku
main verb in a main clause.
only on certain days, so answer choices F and J
52. The best answer is G. Here the underlined word can be eliminated. On the other hand, it makes log-
serves as a connector between this sentence and ical sense that only some people credit Euler with
the preceding sentence. This sentence gives spe- inventing the game. Therefore, answer choice H is
cific details refining the ideas of the previous one. the best answer.
Therefore, answer choice G is best. Answer choice 59. The best answer is C. The beginning of this
F suggests contrast that is inappropriate in this paragraph focuses on Euler’s game and its rela-
context. Answer choice H suggests consequence. tionship to Sudoku. However, with Sentence 4, the
Answer choice J is too vague and doesn’t make writer shifts focus to just Sudoku and the ways it
sense here. challenges players. Therefore, answer choice C is
the best answer because it recognizes this shift in
53. The best answer is C. The subordinate clause
focus.
explains why the puzzle is aptly named. Answer
choices A, B, and D all contain the idea of expla- 60. The best answer is H. The beginning sentence of
nation, and therefore would be appropriate alterna- this paragraph ties the topic of the passage back
PRACTICE TEST 8 ANSWERS AND EXPLANATIONS 663

to the introductory paragraph. Therefore, answer they can be connected by a conjunction as in answer
choice H is best. The writer does not make explicit choices A and C. Answer choice D is grammatically
comparisons between Sudoku and crossword puz- unacceptable.
zles, so answer choice G can be eliminated. The
writer also repeatedly encourages readers to try 68. The best answer is F. This sentence contrasts
Sudoku, not to avoid it, so answer choice J can the results of failure to communicate with the
be eliminated. Answer choice F should be elimi- results of success in the previous sentence. There-
nated because the writer only hints at where Sudoku fore, answer choice F is the best answer because
puzzles are located in a newspaper. it is the only answer choice that indicates con-
trast. The other answer choices indicate causation
(answer choices G and H) or comparison (answer
choice J).
PASSAGE V
69. The best answer is D. The adjective emerging tells
61. The best answer is C. In this example, baby is
the reader that the child is just learning to talk.
modifying cries and should be in the possessive.
Therefore, any additional statement of that idea
62. The best answer is J. In this sentence, the two is redundant. Answer choice D is the best answer
clauses contain two grammatically complete ideas. because the underlined passage is unnecessary.
They can be made into two separate sentences as
70. The best answer is F. This is the only answer
in answer choices F and G, or they can be con-
choice that lists specific needs. The other answer
nected by a conjunction as in answer choice H.
choices are not detailed and should be eliminated.
Answer choice J, however, connects two complete
sentences with a comma. This is called a “comma 71. The best answer is A. The phrase explains the
splice” and it is grammatically incorrect. causal relationship between sign language and ver-
bal language development. It does not provide any
63. The best answer is B. This question tests parallel
specific detail about individual signs, so answer
construction. Because anxiety is a noun, the idea
choice B can be eliminated. The phrase does not
that is linked to it should also be a noun in order to
contradict other information in the passage, so
stay consistent. Therefore, answer choice B confu-
answer choice C can be eliminated. Finally, the
sion is the best answer. The other answer choices
phrase is already at the end of the paragraph, so
are verbs.
answer choice D can be eliminated.
64. The best answer is H. This choice clearly indi-
cates what is being played back in slow motion 72. The best answer is G. The underlined word distinct
(the film). Good writing is clear and not ambigu- is an adjective; it can’t modify a verb, therefore
ous. The remaining answer choices are awkward answer choice J is incorrect. Also, in English,
and unclear. adjectives almost always come before the noun they
modify. Since distinct correctly modifies words,
65. The best answer is D. A list of only two items, in answer choice G is the best answer. Answer choices
this case happiness and comfort, does not need to F and H don’t make sense.
be separated by commas. Therefore, answer choice
D is the best answer. 73. The best answer is C. The verb phrase may be able
must be followed by an infinitive verb, so answer
66. The best answer is G. In this example, sound choice C is the best answer. The other answer
refers back to the plural noun phrase new utter- choices don’t make grammatical sense.
ances, so answer choice F can be eliminated.
Because sound is the main verb in a relative clause, 74. The best answer is G. The section links the child’s
answer choice H should be eliminated. Finally, to ability to follow commands to her own language
stay consistent with the rest of the passage, sound acquisition. Answer choice F should be eliminated
needs to be in the present tense, so answer choice because the author doesn’t say commands are the
H can be eliminated. only way children learn language—in fact, the sec-
tion on sign language would contradict this idea.
67. The best answer is D. In this sentence, the two Answer choice H is also contradicted by the pas-
clauses contain two grammatically complete ideas. sage and should be eliminated. Answer choice J
They can be made into two separate sentences should be eliminated because baby names are not
joined by a semicolon as in answer choice B, or mentioned at all.
664 PRACTICE TEST 8 ANSWERS AND EXPLANATIONS

75. The best answer is B. This sentence stands in idea of contrast but implies that the two ideas
contrast to the sentence before it by correcting are equally correct, so it should be eliminated.
the mistaken idea of most people. The under- Answer choice D is similar to answer choice C.
lined phrase should introduce that concept of con- Only answer choice B clarifies the logical relation-
trast and correction; therefore, answer choice A ship between the two sentences, and is the best
should be eliminated. Answer choice C has the answer.
PRACTICE TEST 8 ANSWERS AND EXPLANATIONS 673

Reading Test Explanations for several minutes, he could be described as shy,


but he certainly could not be described as outgoing;
PASSAGE I eliminate answer choice H.
1. The best answer is B. The narrator begins by 9. The best answer is C. The passage states that,
describing how silence was a concept unfamiliar “Within the parameters of my innocent world, I
to her. It made her uncomfortable: “I knew silence knew silence as a lack of something.” This, along
as a lack of something.” The passage concludes with other sentiments expressed by the author, sug-
by showing that her silence on the bus allowed gests that the author’s view of silence was limited
Jaime to gather the courage to speak to the narrator, or restricted. The other answer choices are not
and that this experience had changed her long held supported by the context.
opinion of silence.
10. The best answer is F. The passage states that,
2. The best answer is F. The tone of the passage “On the bus, most everyone was either asleep or
indicates that the narrator is initially uncomfort- totally oblivious.” This detail best supports answer
able with silence in general. When the narrator sat choice F.
next to Jaime, his quietness made her “feel a little
awkward.” This information best supports answer PASSAGE II
choice F.
11. The best answer is A. As stated in the first para-
3. The best answer is D. According to the passage, graph of the passage, “The Republic had been
the narrator “never understood the value of under considerable stress for several years before
silence.” She considered silence “a lack of some- Caius Julius was born, thus he did not create the
thing,” and goes on to say that, “maybe I was fissures that led to the collapse of the Roman
even a little afraid of the emptiness it created—the Republic, though he did capitalize on them bril-
aural darkness where forgiveness never happened.” liantly.” This sentence indicates that Julius Caesar
These details best support answer choice D. used conflicts of the past to enhance and promote
4. The best answer is G. The passage states that, his own power.
“I knew silence as a lack of something.” It goes on 12. The best answer is H. The passage states that,
to say about silence that the narrator was “a little “Caius Julius Caesar is popularly considered the
afraid of the emptiness it created.” In addition, once founder of the Roman Empire, though it would be
the narrator encounters Jaime, he did not speak for more accurate to consider his political rise as mark-
several minutes. These details best support answer ing the end of the Roman Republic.” From this
choice G. statement, one can assume that because Caesar’s
5. The best answer is B. The narrator says that she political rise marked the end of the Roman Empire,
didn’t believe in silence as a healer before she met Caesar exploited the political problems of the
Jaime. However, the lines referenced in the ques- Roman Empire in order to gain power. The other
tion indicate a change of heart that is best expressed answer choices do not reflect the main idea of the
in answer choice B. paragraph.
13. The best answer is D. The passage states, “Roman
6. The best answer is H. According to the passage,
armies were traditionally made up of small land-
“Buses have their own unique demographic: each
holders.” This statement, along with the phrase in
crowd is unlike any other.” This best supports
the question stem, indicates that unless a farmer
answer choice H.
owned a certain amount of land, he could not join
7. The best answer is A. According to the passage, the military, thereby establishing a close traditional
“My university required a community service stint relationship between farming and military service.
to graduate … I’d heard that the local YMCA was
14. The best answer is F. As indicated by the
a good resource … I made my way to the YMCA
passage, aristocratic fear of political reform
looking for easy credits.” These statements best
was the reasoning behind the brothers’ murders.
support answer choice A.
Tiberius Gracchus was murdered over his determi-
8. The best answer is J. The passage states that, nation to rid the Roman Empire of the military
“his eyes were alert. And they were glued on me.” enrollment land requirement, while his brother,
These statements indicate that Jaime was alert, not Gaius Gracchus, also caused trouble due to his
sleepy or bored, so eliminate answer choices F rebellious ideas. “When the younger brother, Gaius
and G. Because Jaime didn’t speak to the narrator Gracchus, began his reforms 10 years later, he was
674 PRACTICE TEST 8 ANSWERS AND EXPLANATIONS

able to extend political rights to the lower classes Julius Caesar’s assassination; “The resulting polit-
and reduce opportunities for bribery and corruption ical chaos ultimately led to the rise of Augustus
among the upper classes. He, too, was murdered for Caesar as emperor, effectively ending the Roman
pushing the system too far.” This information best Republic forever.” The other answer choices are
supports answer choice F. not supported by the passage.

15. The best answer is B. As stated in the passage,


“Marius also challenged the traditional structure PASSAGE III
of the army where nobility were regularly given
21. The best answer is C. The passage states in the
authority over lower-class officers with more expe-
last paragraph that The Táin is an epic of great
rience and ability.” This indicates that Marius saw
value. “If the images meet a cultural need, they can
more value in ability than social class.
come back to life as living artistic works. During
16. The best answer is J. The passage states, “Sulla the English occupation of Ireland up through the
was allied with the Roman Senate, who feared Irish Revolution, many artists plumbed the depths
Marius’ ambition and influence with the masses. of Irish mythology to create what they saw as an
Sulla believed in strengthening the power of the image of Ireland free of English cultural repres-
Senate against the popular Assembly. To this end, sion. Writers still call on The Táin for inspiration,
he marched his armies against Rome, defeating just as ancient Irish bards once called on the ghost
Marius and establishing himself as Dictator. While of Fergus to tell them the true story of the Cattle
Sulla eventually resigned the dictatorship peace- Raid of Cooley.” The passage makes it clear that
fully, he had exiled or killed thousands of political the epic is of significant value to Irish history.
opponents during his reign.” Answers F, G, and H
are all found within the passage. Answer choice J 22. The best answer is J. The passage answers the
is the only answer not found within the passage. questions presented in answer choices F, G, and H.
The question regarding vernacular literature is
17. The best answer is A. The last paragraph of the answered with the following: “Ireland has the old-
passage opens by contrasting Julius Caesar’s ties to est vernacular literature in Europe. Where other
aristocracy to his actions as a reformer. As indicated early European authors wrote their literary works
by the passages, Julius Caesar supported many in Latin, the Irish began writing down their stories
ideas that were exactly the opposite of what his in their own language starting at least as early as
family ties desired. This contrast helps to illustrate the 6th century A.D. and continuing to the mod-
that Julius Caesar was a true political reformer. ern day.” Vernacular language is defined by the
passage as stories written down in one’s native lan-
18. The best answer is F. Throughout the passage the guage. In regard to Queen Medb’s desire to acquire
author cites the actions of several different reform- a magic bull, her “insistence on equaling the prop-
ers, including Julius Caesar as well as the Gracchi erty of her husband was, therefore, for reasons far
brothers. Every reformer detailed by the author more serious than vanity… Wives were considered
was assassinated. Because the author includes the legal equals to men if they came into their marriage
stories and outcomes of these reformers, one can with as much or more property than their husbands.
infer that many Roman reformers were eventually If she had less, the wife would be a legal dependent
assassinated. of her husband and, like a child, would have limited
19. The best answer is D. The passage states, “Like the rights of her own.” Finally, the passage explains
Gracchi brothers, Julius Caesar supported the redis- that The Táin Bó Culainge means “The Cattle Raid
tribution of public lands to the poor and protected of Cooley.” Answer choice J is the only question
the grain supply (a large part of the unemploy- not answered by information in the passage.
ment dole). In his armies, he promoted ability
23. The best answer is A. In regard to why the epic
before social rank. He was also widely seen to
plays an important role in Irish literature, the pas-
support the middle and lower classes against the
sage states, “During the English occupation of
privileges of the aristocracy, namely the Senate.”
Ireland up through the Irish Revolution, many
Answer choices A, B, and C are all political reforms
artists plumbed the depths of Irish mythology to
mentioned by the passage.
create what they saw as an image of Ireland free of
20. The best answer is H. The passage closes with the English cultural repression.” This information best
following statement in regard to the aftermath of supports answer choice A.
PRACTICE TEST 8 ANSWERS AND EXPLANATIONS 675

24. The best answer is F. The second paragraph serves Queen Medb takes up with the warrior Fergus, with
to describe The Cattle Raid of Cooley. In its nar- the approval of her husband, in order to guarantee
rowest sense, the raid refers to a series of battles Fergus’ allegiance during the war.”
fought by the northern Irish province of Connacht
to steal a magic bull from the neighboring province 29. The best answer is A. As indicated by the pas-
of Ulster. However, the cycle includes many other sage, there are strong thematic similarities between
legends that together tell the national story of the the King Arthur story and The Táin. Specifically,
people of Ulster, especially during the reign of the the passage states, “Still, even Medb’s daughter,
great Ulster king, Conchobor mac Nessa. Finnabair, is tied linguistically to the Arthurian
legend: Finnabair and Guinevere are different
25. The best answer is D. Of the four answer choices,
spellings of the same name.” This statement is used
A, B, and C can all be found in the passage. “The
to further support the passages earlier tie between
remscéla tell how the bulls were originally two pig-
the two stories; “Thematically, the work explores
keepers who knew magic; the stories explain how
several great issues that would occupy medieval
the Ulstermen came to be cursed with debilitating
authors for over six hundred years. One of the
pain whenever their country was in danger. They
most important was the lovers’ triangle between
explain who Cú Chulainn was and how he got his
the king, the queen, and the warrior hero. Think
name. And, most poignantly, the remscéla tell how
of the romances of King Arthur, Queen Guinevere,
Fergus lost his crown and why he agreed to fight
and Sir Lancelot, or those of Tristran, Isolde, and
against his countrymen. This last story, told in the
her husband, King Mark.” These statements best
tragic legend of Deirdre and the Sons of Usnech,
support answer choice A.
is one of the most striking of all the Irish myths.”
Answer choice D is the only choice not mentioned 30. The best answer is H. The passage explains that
by the passage. Medb’s dedication to acquiring one of the two
26. The best answer is G. As stated in the last para- magic bulls of Ulster was because not only did
graph of the passage, “Over twelve hundred years her husband have the other magic bull, but if she
old, The Táin is certainly an epic work—but epic acquired a bull as well, she would attain the same
doesn’t necessarily mean “dead.” If the images level of political power as her husband. Otherwise,
meet a cultural need, they can come back to life she would be at his mercy. Due to these circum-
as living artistic works … Writers still call on stances, Medb was very ambitious in her pursuits.
The Táin for inspiration, just as ancient Irish bards The other answer choices are not supported by the
once called on the ghost of Fergus to tell them the passage.
true story of the Cattle Raid of Cooley.” Not only
does this segment of the passage establish answer
PASSAGE IV
choices F, H, and J as elements of The Táin, but it
also states that an epic isn’t dead, as long as it still 31. The best answer is D. Of the four answer choices,
has cultural significance. choices A, B, and C can all be found in the
first paragraph of the passage. This passage reads,
27. The best answer is D. In an effort to explain why
“Homeopathy is a system for treating physical dis-
Queen Medb wanted a magic bull, the passage
ease and other ailments using the theory of treating
states the following: “Wives were considered legal
“like with like.” In practice, homeopathic medicine
equals to men if they came into their marriage with
seeks substances that mimic an ailment’s symptoms;
as much or more property than their husbands. If
this sameness is considered “likeness.” The sub-
she had less, the wife would be a legal dependent
stance is then diluted to infinitesimal amounts and
of her husband and, like a child, would have lim-
administered to the patient in order to cure the prob-
ited rights of her own. Irish queens were used to
lem. Homeopathic treatment is currently in use for
having their own political autonomy and making
everything from cancer to colds and flu, though
their own political deals.” This information best
many scientists remain heavily skeptical about its
supports answer choice D.
efficacy.” Not only is answer choice D not men-
28. The best answer is J. When speaking of love trian- tioned in this paragraph, but the passage indicates
gles, the passage gives the following as examples, that, “Modern scientists have been unable to find
“Think of the romances of King Arthur, Queen any evidence to support the theory of molecular
Guinevere, and Sir Lancelot, or those of Tristran, memory,” which is the opposite of the statement
Isolde, and her husband, King Mark. In The Táin, asserted by answer choice D.
676 PRACTICE TEST 8 ANSWERS AND EXPLANATIONS

32. The best answer is F. The passage states that, then fill the prescriptions.” Based on this informa-
“Hahnemann had hypothesized that shaking the tion, one can conclude that, although there is a lack
solution after each dilution would imprint the of clinical data, homeopathy remains a popular form
molecular “memory” of the original substance into of treatment, even among doctors and pharmacists.
the solution, which would allow the diluted dose to
36. The best answer is H. As is explained by the pas-
be effective without the possibility of overdose or
sage, “When a person became ill, doctors believed
adverse side effects.” This information best supports
it was because one or more of the humors had
answer choice F.
come out of balance. Some of the best treatments
33. The best answer is C. As stated in the passage, were thought to be bloodletting and purgation—
“Hahnemann first stumbled upon his theory when he with the assumption being that these treatments
was investigating a common treatment for malaria, would effectively drain off the excess humors.” This
cinchona bark. Modern scientists now know that information best supports answer choice F.
cinchona bark contains quinine—a substance still
37. The best answer is B. As described in the sec-
used to treat malaria—but at the time, no one knew
ond paragraph of the passage, “… Sanguine, or
why the bark was effective.” This information best
passionate, persons were hot and wet.”
supports answer choice C.
38. The best answer is J. The fourth paragraph of
34. The best answer is J. The passage states, “Modern
the passage states that, “Perhaps not surprisingly,
scientists have been unable to find any evidence to
Hahnemann’s new field of homeopathy (i.e. “simi-
support the theory of molecular memory. In fact, the
lar suffering”) was met with considerable resistance
idea that diluting a substance makes it stronger runs
from doctors comfortable with their usual prac-
against the principles of chemistry and physics.”
tices.” As it is used in this sentence, one can
Based on this information, one can infer that modern
infer that practices most closely means the var-
doctors are unable to find evidence to support home-
ious treatments employed by doctors of Hahne-
opathic treatment because the theory of molecular
mann’s era. This information best supports answer
memory is contrary to the principles of chemistry
choice J.
and physics.
39. The best answer is B. The fourth paragraph of
35. The best answer is C. The last paragraph of the
the passage explains, “Hahnemann further hypoth-
passage states, “Today, homeopathy is generally
esized that, while undiluted substances would only
disregarded by many conventional medical prac-
worsen symptoms in the sick, heavily diluted
titioners. Homeopathic practitioners are frequently
substances could be effective for a cure.” This
termed quacks by conventional scientists. Neverthe-
information best supports answer choice B.
less, homeopathy remains extremely popular both in
the United States and abroad. In European countries 40. The best answer is H. The last paragraph of
such as France and England, conventional doc- the passage explains that homeopathy is a contro-
tors frequently prescribe homeopathic treatments versial form of medical treatment. “Nevertheless,
for common illnesses, such as colds and flu. Phar- homeopathy remains extremely popular both in the
macists who are trained to answer questions about United States and abroad.” This information best
the homeopathic treatments’ use and desired effects supports answer choice H.
PRACTICE TEST 9 ANSWER KEY 739

ANSWER KEY

English Test
1. A 21. D 41. A 61. A
2. F 22. F 42. H 62. G
3. C 23. B 43. D 63. D
4. H 24. H 44. F 64. H
5. A 25. D 45. D 65. B
6. G 26. H 46. J 66. H
7. C 27. A 47. A 67. C
8. F 28. J 48. G 68. J
9. D 29. D 49. D 69. A
10. G 30. H 50. H 70. G
11. D 31. B 51. B 71. C
12. G 32. F 52. J 72. J
13. B 33. A 53. C 73. A
14. F 34. J 54. F 74. J
15. C 35. A 55. C 75. A
16. G 36. J 56. F
17. A 37. B 57. B
18. F 38. H 58. G
19. B 39. A 59. C
20. F 40. G 60. J
PRACTICE TEST 9 ANSWER KEY 741

Reading Test Science Reasoning Test


1. B 21. A 1. D 21. C
2. J 22. F 2. G 22. J
3. A 23. D 3. B 23. B
4. H 24. G 4. J 24. F
5. A 25. D 5. B 25. B
6. F 26. H 6. F 26. J
7. C 27. B 7. A 27. A
8. G 28. F 8. F 28. J
9. C 29. C 9. C 29. A
10. J 30. J 10. G 30. H
11. A 31. A 11. D 31. D
12. H 32. G 12. H 32. G
13. D 33. C 13. B 33. A
14. F 34. F 14. H 34. H
15. C 35. D 15. C 35. C
16. G 36. H 16. F 36. G
17. B 37. A 17. B 37. A
18. H 38. G 18. G 38. H
19. D 39. D 19. C 39. B
20. F 40. J 20. J 40. H
PRACTICE TEST 9 ANSWERS AND EXPLANATIONS 747

ANSWERS AND EXPLANATIONS also not supported. In fact, no connective is needed,


and answer choice D is best.
English Test Explanations 10. The best answer is G. The underlined phrase
is a list with only two items, and no commas
PASSAGE I are necessary to separate them from each other.
Likewise, because it’s a restrictive clause, neces-
1. The best answer is A. The pronoun this tells the
sary for the sentence to make sense, it doesn’t
reader that a description is coming further in the
need commas to separate it from the rest of the
sentence. The sentence as it is written uses the
sentence.
correct verb tense and keeps the entire thought
in one sentence; therefore, it’s the best answer 11. The best answer is D. This pronoun properly refers
choice. to the writer’s grandfather. Therefore answer choice
D, his, is best.
2. The best answer is F. The paragraph is most
concerned with the writer’s Grandma and her emo- 12. The best answer is G. The topic of this paragraph
tional relationship to the chest. Answer choices G, is dreams—more specifically, the contrast between
H, and J all shift attention away from the grand- the dreams of the writer’s grandfather and grand-
mother’s personal response and towards superficial, mother. The grandfather’s dreams were to be a
physical descriptions. heroic bomber pilot, while the grandmother wanted
him to do something that would keep him safe for
3. The best answer is C. The best answer choice cor-
his family. Answer choice G best introduces the
rectly uses commas to separate the two clauses.
tension between the two.
Treasure is an adjective modifying trove, so there
is no need for a comma between them (answer 13. The best answer is B. The writer makes it clear
choice A). Answer choices B and D include unnec- that her grandfather took pride not only in being a
essary commas after and. pilot, but also in his wife. It would make sense that
her grandfather would have a picture of the plane
4. The best answer is H. All of the answer choices
he named after his wife, and that the writer would
have acceptable replacements for the idiomatic say-
refer to it as her grandfather’s pride and joy.
ing truth be told. However, answer choice H has
the phrase kind of type of, which is redundant. 14. The best answer is F. Use the possessive relative
Therefore, it is not an acceptable replacement. pronoun to refer to the writer’s grandmother.
5. The best answer is A. The first sentence sets the 15. The best answer is C. This passage is intensely
stage for the action, but does not give any new personal. It doesn’t try to place the actions into
information. It could easily be deleted without a larger historical context, and gives very little
losing the sense of the story. information about the use of airplanes in World
War II. Therefore, answer choices A and B are inap-
6. The best answer is G. The object of this sen-
propriate. The passage indicates that the writer’s
tence is the interior of the wooden box. Because
grandparents had a loving relationship, so answer
wooden box is singular and possessive, it needs an
choice D is not best.
apostrophe and an s.
7. The best answer is C. The story is being told using
the simple past tense. Answer choice A creates an
incomplete sentence. Answer choice D indicates PASSAGE II
action over time, which is also not appropriate in
16. The best answer is G. Only answer choice G has a
this context.
subject (tomatoes), verb (have become), and object
8. The best answer is F. The phrase that begins with (a staple) without being in a dependent clause. The
flying is a nonrestrictive clause. That means it could other answer choices lack a main verb and create
be removed without destroying the meaning of the incomplete sentences.
sentence. Nonrestrictive clauses need to be set off
17. The best answer is A. This phrase is a three-item
by commas, making answer choice F best.
list; each item must be separated by a comma.
9. The best answer is D. Answer choices A and C Answer choices C and D are incorrect because
imply a contrast that is not supported by the sen- the comma needs to be placed after stands, not
tence. Answer choice B implies a conclusion that is after and.
748 PRACTICE TEST 9 ANSWERS AND EXPLANATIONS

18. The best answer is F. Answer choice G is incorrect creates a run-on sentence. Answer choice J implies
because the phrase doesn’t do anything to explain a contrast that doesn’t make sense in this context.
anything about the writer. The phrase also doesn’t
27. The best answer is A. The adjective bright modi-
mention other summer produce, so answer choice H
fies red, so the words don’t need to be separated
is incorrect. The phrase does, however, explain
by a comma. This eliminates answer choices B
that tomatoes are plentiful during the growing sea-
and D. However, the phrase Once a tomato... is
son, which would logically lead to a need for more
a nonrestrictive relative clause and does need to be
tomato recipes.
separated from the main sentence by a comma.
19. The best answer is B. The correct idiom is plenty
28. The best answer is J. The phrase doesn’t add
of. Omitting the underlined portion, while not
significant details or explain why fresh vegeta-
grammatically incorrect, would reduce the clarity
bles are important. Therefore, answer choices F
of the sentence, which focuses on the number of
and G can be eliminated. Both of the remaining
ways to prepare and eat tomatoes.
phrases are specific, so answer choice H can be
20. The best answer is F. As written, the underlined eliminated. Answer choice J is best because the
portion introduces the idea of a contrast between proposed replacement is much wordier than the
recipes that are universally delicious, but have original without adding anything significant.
individual levels of difficulty in preparation. The
29. The best answer is D. The pronoun here refers
remaining answer choices are either too vague or
back to the noun Italians. Therefore, the plural
awkward.
pronoun, them, is best.
21. The best answer is D. The writer wants to give
30. The best answer is H. The key phrase in this ques-
a sequence of events: first peeling and deseeding,
tion is modern cuisine. The essay’s focus is much
then cooking. Only answer choice D tells the reader
more general than how tomatoes are used in a par-
precisely which action comes first.
ticular type of cuisine. Therefore, answer choices F
22. The best answer is F. The verb here is in the pas- and G can be eliminated. The essay does mention
sive voice because the action, peeling, is happening specific recipes, so answer choice J can be elimi-
to the subject. Therefore, the correct form is accom- nated. Answer choice H is best because the writer
plished. Because accomplished is a verb, it needs takes a general approach and includes pieces of
to be modified by the adverb, easily. information like the fact that Italians once thought
that tomatoes were poisonous.
23. The best answer is B. The pronoun whoever is the
subject of this sentence, so who is the correct form.
Answer choices A and C use the object form whom,
and should be eliminated. The passage is written in PASSAGE III
the present tense, so answer choice B is correct.
31. The best answer is B. By definition, a hole is
24. The best answer is H. In this sentence, the phrase empty, so the underlined portion, as it is, is some-
surrounding cool enough to handle doesn’t have what redundant. The adjective that best fits the tone
a main verb, so it can’t stand on its own. This of the paragraph is gaping. Omitting the under-
eliminates all choices except answer choice H. lined portion eliminates the modifier a, creating an
awkward, ungrammatical sentence.
25. The best answer is D. The paragraph explains
in great detail how to peel and deseed tomatoes. 32. The best answer is F. The phrase beginning with
The last sentence tells us this is important, but gumming is a nonrestrictive relative clause; there-
only answer choice D includes information that fore it needs to be set apart from the main sentence
tells us why. Answer choices C and D explain by a comma. The phrase is not a complete sentence,
why it might be important to use a strainer, but so answer choices G and H should be eliminated.
their scope doesn’t cover the entire paragraph.
33. The best answer is A. This paragraph introduces
Answer choice B introduces a new idea, growing
a new idea: the tools and techniques of modern
new plants, that is also outside the scope of the
dentistry. No connective to the previous paragraph
paragraph.
is necessary, so answer choice A is best. Answer
26. The best answer is H. This sentence is actually two choices C and D are incorrect because they imply
complete sentences. Joining them with a comma, continuity with the previous paragraph. Answer
as in answer choice F, is called a “comma splice” choice B implies a level of contrast that isn’t
and is grammatically incorrect. Answer choice G necessary.
PRACTICE TEST 9 ANSWERS AND EXPLANATIONS 749

34. The best answer is J. Answer choices F, G, and H 42. The best answer is H. Answer choices F, G, and
are all redundant. By definition, a disposable needle J are either too wordy or grammatically incorrect.
is one that can be disposed of. Answer choice J has Answer choice H is both succinct and grammati-
the all the information needed; therefore, it’s the cally correct, so it is best.
best answer.
43. The best answer is D. The phrase after 1950
35. The best answer is A. The focus of this paragraph means the same thing as second half of the twen-
is equipment; therefore, answer choice A is best tieth century. So, answer choices A, B, and C are
because it describes the widespread use of auto- all redundant. Answer choice D is best because it
claves in modern dentistry. Answer choice B should has all the information necessary without repeating
be eliminated because the phrase does not further itself.
define the term autoclave. Answer choice C is 44. The best answer is F. The tone of the passage
incorrect because the phrase helps explain how the is scientific and dispassionate. Answer choice G,
offices had become more sterile. Answer choice D therefore, should be eliminated because it has the
can be eliminated because the focus of the para- wrong tone. Answer choice H is too vague, and
graph is not on teeth. answer choice J is off topic. Answer choice F
36. The best answer is J. The sentence already tells explains precise ways in which dentistry has
the reader that the equipment uses heat to sterilize improved over the past 50 years, which relates
the equipment. Therefore, the phrase beginning by directly back to the main topic of the essay.
using the heat is redundant. 45. The best answer is D. Paragraph 3 does not contain
detailed information about dental practices, as do
37. The best answer is B. The verb work has to be in
Paragraphs 2 and 4. Instead, it’s more of a summary
the present tense to match the rest of the paragraph.
of the changes in dentistry over time. Therefore, the
Therefore, answer choices A and C can be elimi-
best place for it is at the end of the essay, which
nated. Answer choice D would need the auxiliary
makes answer choice D best.
verb are to be correct.

38. The best answer is H. The contrast on the other PASSAGE IV


hand needs to be separated from the main sen-
46. The best answer is J. Here, Mr. Peale is a type
tence by a comma. The phrase, constant changes,
of nonrestrictive clause modifying our Advanced
however, is the subject of the main clause and
English teacher, and should be separated by com-
doesn’t need a comma. This phrase is also followed
mas from the rest of the sentence.
by a restrictive relative clause, which requires
no comma. 47. The best answer is A. That we especially liked is
a restrictive relative clause. That means, if it were
39. The best answer is A. Answer choices C and D removed, the basic sense of the sentence would
imply a contrast with the previous sentence, and change. Restrictive relative clauses do not take
so can be eliminated. Answer choice B makes the commas, so answer choices B and D can be elim-
phrase a separate clause. Answer choice A is correct inated. The final clause is not independent, so it
because the phrase is part of a noun phrase that acts does not take a comma. This eliminates choice C.
as the subject of the sentence and doesn’t need a
conjunction. 48. The best answer is G. Because the list of painters
is the object of the verb shared, a colon is incorrect;
40. The best answer is G. All of the answer eliminate answer choice F. Commas are not used to
choices offer some added detail. However, answer begin lists, so eliminate answer choice H. Answer
choices F, H, and J are too vague. Only answer Choice J, share, is in the wrong tense.
choice G gives a detail that improves the reader’s
understanding of the challenges faced by modern 49. The best answer is D. The topic of this paragraph is
dentists as opposed to those of 50 years ago. to introduce a day when the writer had to reproduce
a famous painting in her English class. Answer
41. The best answer is A. This sentence is really two choices A and C provide specific details that are
complete sentences, and therefore needs to be sepa- not followed up in the next paragraph. Therefore,
rated by a period or a semicolon. Answer choice B they are off topic and should be eliminated. Answer
creates a comma splice, answer choice C creates choice B does not have the right tone and is also
a run-on sentence, and answer choice D creates a off topic. The best choice is to delete the sentence
long, incomplete sentence. altogether because it is irrelevant.
750 PRACTICE TEST 9 ANSWERS AND EXPLANATIONS

50. The best answer is H. Answer choices F, G, and J sense. Only answer choice G provides an appropri-
are redundant. Answer choice H, however, contains ate adverb.
all the necessary information without any extra
59. The best answer is C. Answer choices A and B
words.
are wordier versions of answer choice C, so they
51. The best answer is B. The passage has been written can be eliminated. Answer choice D doesn’t make
in the past tense. Therefore, answer choices A and C sense and can be eliminated.
can be eliminated. Answer choice D doesn’t make
60. The best answer is J. Answer choice F is in the
grammatical sense, so it should be eliminated.
future tense, so eliminate it. Answer choices G
52. The best answer is J. The word proclaimed and H are awkward and wordy, so eliminate them.
means to state loudly and with assurance. Answer Answer choice J is concise and grammatically
choices F, G, and H are all close synonyms. Answer correct.
choice J, however, indicates that Mr. Peale put his
thoughts in writing, which is not an acceptable PASSAGE V
alternative.
61. The best answer is A. The conjunction however
53. The best answer is C. The writer’s phrase is not needs to be separated from the rest of the sen-
directed at either her teacher or Impressionist art. tence by commas. Answer choices B and C create
So, answer choices A and B can be eliminated. She incomplete sentences.
is instead commenting on how his response made
her feel—which was ambivalent. 62. The best answer is G. The phrase in Africa is a
restrictive clause that should not be separated from
54. The best answer is F. The subject of Sentence 3 the rest of the sentence by punctuation.
is Mr. Peale’s lack of comment. It would not
make sense to place the sentence before Sentence 2 63. The best answer is D. The use of the modifier
because that’s where the writer gives that informa- other instead of another tells the reader that the
tion. So, eliminate answer choices G and H. By noun countries needs to be plural. Additionally, the
Sentence 4, Mr. Peale has moved on, so eliminate introductory clause must be separated by a comma
answer choice J. Answer choice F makes the most from the main clause. Answer choice C uses a
logical sense. comma correctly, but it incorrectly includes the
plural possessive form of the noun.
55. The best answer is C. Answer choices A and B
are unnecessarily wordy. Answer choice D implies 64. The best answer is H. The conjunction here needs
a relative clause that isn’t there. Answer choice C to connect this sentence to the previous one by
is grammatically correct and concise. suggesting that this sentence offers an example.
Answer choices F and G imply a contrast to the
56. The best answer is F. The proposed sentence needs previous sentence. Answer choice J implies a com-
to link the idea that the writer had captured the parative. Only answer choice H correctly implies
essence of the original painting and that she valued that the tall trees on plantations around the world
her reproduction enough to hang it on her wall, assisted in the growth of coffee plants by simulating
even though it wasn’t terribly dramatic. The pro- their preferred habitats.
posed sentence does this by emphasizing that the
reproduction had captured the parts of the original 65. The best answer is B. The two phrases are each
most valued by the writer: its beauty and serenity. complete sentences and must be joined by either
a semicolon or some form of end punctuation.
57. The best answer is B. The correct sentence needs Answer choice C creates a “comma splice” by using
to be in the past tense, to stay consistent with the a comma to join two independent clauses.
rest of the essay and use of active voice. So, answer
choice A can be eliminated. Answer choice C 66. The best answer is H. This paragraph is written in
doesn’t make grammatical sense. Answer choice D the present tense. Therefore, the simple past tense
has the awkward and unnecessary phrase by me, so is correct. Answer choice H is also in the active
it also can be eliminated. voice, making it the best choice.

58. The best answer is G. Answer choice F implies 67. The best answer is C. The sense of the phrase is
a conditional state not supported by the sentence. that the new plants are sun-tolerant and also able
Answer choice H implies a contrast not supported to thrive in open sunlight. All of the choices have
by the sentence. Answer choice J does not make this sense of continuity except answer choice C.
PRACTICE TEST 9 ANSWERS AND EXPLANATIONS 751

68. The best answer is J. The phrase amount of is be acceptable alternatives. Answer choice J, there-
idiomatic and should be used here. The remaining fore, implies a conclusion or direct result. It is not
answer choices are not idiomatic. an acceptable alternative to as.
69. The best answer is A. The conjunction here needs 73. The best answer is A. The phrase as well as many
to give the sense of result or conclusion. Answer other fauna and flora is a nonrestrictive clause
choices B and C imply contrast. Answer choice D because removing it does not significantly alter the
implies continuity, but not result. meaning of the sentence. A nonrestrictive clause is
70. The best answer is G. The key to this question separated from the main sentence by commas.
is effective visual detail. Answer choice G offers 74. The best answer is J. Sentence 3 tells the reader
the most precise detail because it specifies that the about recorded decreases in the songbird popu-
ground protection is from trees and that the rainfall lation of the northern United States. However,
is pounding. the effect of coffee production on birds is not
71. The best answer is C. Because ornithologists is introduced until Sentence 5. Therefore, Sentence
the subject of the sentence and discover is the verb, 3 should logically come after Sentence 5, after
the relative pronoun who is unnecessary. Therefore, Sentence 6 most specifically. That makes answer
answer choices A and B can be eliminated. Answer choice J, before Sentence 7, the best answer.
choice D is in the past tense, while the rest of the
75. The best answer is A. Answer choice A clearly and
paragraph is in the present tense.
concisely states the idea of many people coming to
72. The best answer is J. The conjunction as implies a realization. The other choices state the same idea,
a reason or explanation, or even action over time. but they are grammatically awkward and too wordy
Consequently, answer choices F, G, and H would all or redundant.
760 PRACTICE TEST 9 ANSWERS AND EXPLANATIONS

Reading Test Explanations any hardships she and John may have faced to
support answer choice F as an accurate description
PASSAGE I of the passage.
1. The best answer is B. The tone of the first seven 7. The best answer is C. The question asks about
paragraphs suggests that Libby is not as excited John’s perception of his wife. When John says,
about the move as is her husband John. The pas- “Libby, we have an entire car and trailer to
sages states, “… hoping the enthusiasm in my unpack. You might as well forget about looking
voice would match the sparkle in his eyes” to indi- glamorous, at least for now. Besides, it’s not like
cate that Libby was attempting to appear excited we’re back in D.C. No C-Span cameras here!”
for John’s sake. The other answer choices are he is referring to her attempt to fix her hair after
clearly indicated in the first seven paragraphs. the long trip. This suggests that he thought Libby
2. The best answer is J. The passage states that, was somewhat vain and too concerned about her
“The fall semester was just a few short weeks appearance. The other answer choices are not
away.” This statement and the mention elsewhere supported by the passage.
in the passage of the hot weather indicates that
8. The best answer is G. Prior to the lines men-
it was most likely late summer when the couple
tioned in the question, the passage states that
moved into their new apartment.
Libby “knew it [the future] could be anything we
3. The best answer is A. According to the pas- wanted it to be.” The passage goes on to state that
sage, Libby initially was less enthusiastic than Libby, “felt a real thrill of excitement” after she
John about the move, but eventually changed looked around the apartment. These statements
her attitude. The passages states, “… hoping the best support answer choice G.
enthusiasm in my voice would match the sparkle
in his eyes” to indicate that Libby was initially 9. The best answer is C. The first two sentences of
attempting to appear excited for John’s sake. This the first paragraph set the tone for the paragraph;
information best supports answer choice A. as far as Libby, the main character, was concerned
they had made it “just in time.” Additionally, after
4. The best answer is H. According to the passage, she got out of the car, her mood began to improve,
Libby initially was less enthusiastic than John which suggests that being in the car had made her
about the move, but eventually changed her atti- somewhat unhappy. Answer choices A and D are
tude. The passage states that, “There’s something too general and broad, while answer choice B is
monumental about your first step into your new too specific.
home … Now, suddenly, I knew the future had
begun … I knew it could be anything we wanted 10. The best answer is J. The primary focus of the
it to be.” These statements best support an atti- passage is the young couple’s move from Wash-
tude of cautious optimism. While Libby was not ington, D.C. to Ohio, where their lives would be
very excited when she first arrived, nothing in the quite different. They packed up the life they had
passage suggests that she dreaded the move, so in D.C. and moved it halfway across the country
eliminate answer choice G. Likewise, the other to the Midwest. The statement “unload our life”
answer choices are not supported by details in the refers to this significant change.
passage.
5. The best answer is A. The passage states that, PASSAGE II
“We each grabbed an armful and headed towards
our new apartment, just a few blocks away 11. The best answer is A. According to the passage,
from the squat, industrial-looking building where there is a “specific measure, the ‘poverty line,”’
John would start teaching.” This statement best that can be used to determine poverty. Because the
supports answer choice A. While Libby was passage refers to this line as being a specific mea-
accustomed to university buildings that were ivy- sure, answer choice A is best. The other answer
covered, John’s new workplace was different. choices are not supported by the passage.

6. The best answer is F. The passage does not 12. The best answer is H. As stated in the passage,
describe the hardships faced by a young couple in “The question of poverty is extremely complex.”
their first home. While there is some mention of Therefore, one can infer that measuring poverty is
Libby’s initial reservations about moving into the a difficult task. The other answer choices are not
new apartment, there is not enough detail about supported by the passage.
PRACTICE TEST 9 ANSWERS AND EXPLANATIONS 761

13. The best answer is D. According to the pas- what poverty is, and when someone has been
sage, the average American family in the 1950s lifted out of it.” Based on this information, one
“spent approximately one-third of its household can infer that the paragraph aims to detail the
income on food.” When one subtracts 1/3 from the importance of measuring poverty in relation to aid
whole, 2/3 remains. Therefore, the average Amer- disbursement.
ican family spent 2/3 of their income on non-food
18. The best answer is H. The passage states the fol-
items in the 1950s.
lowing information: “While families today spend
14. The best answer is F. As stated in the passage, about 12 percent of their income on food—
“While families today spend about 12 percent of nowhere near the 33 percent assumed in the
their income on food—nowhere near the 33 per- 1950s …” This statement best supports answer
cent assumed in the 1950s—the cost of important choice H.
budget items, such as housing, transportation, and
19. The best answer is D. According to the passage,
health care, has increased dramatically. Orshan-
“Any change in the measured poverty level of a
sky’s poverty measure, which only takes into
society is an indicator of economic health within
account the ability of a household to provide itself
that society, and no president has been willing
with food, is missing several essential components
to increase the perceived amount of poverty for
to be accurate in modern society. With over $60
a statistical recalculation, no matter how justi-
billion in federal aid tied each year to this guide-
fied. Indeed, some economists say that updating
line, not to mention an additional $260 billion
the poverty measure would increase the number
in Medicaid spending, the fact is many Ameri-
of those considered poor, and therefore eligible
cans are falling deeper into poverty and failing
for government aid, by as much as 2 percent-
to receive the aid they so desperately need and
age points.” Based on this information, answer
deserve.” This information suggests that Ameri-
choices A, B, and C are all reasons why no
cans are much poorer than they were in the past,
president is willing to make a poverty level
which best supports answer choice F.
change.
15. The best answer is C. The passage states that,
20. The best answer is F. As stated in the pas-
“Orshansky’s poverty measure, which only takes
sage, “Orshansky’s statistical measurement was
into account the ability of a household to provide
one small part of the federal government’s plan
itself with food, is missing several essential com-
to attack the difficult national economic condi-
ponents to be accurate in modern society. With
tions that were hurting millions of Americans in
over $60 billion in federal aid tied each year to
the early 1960s.” Specifically, Orshansky worked
this guideline, not to mention an additional $260
to formulate a way to define poverty so that the
billion in Medicaid spending, the fact is many
government could help those who met this def-
Americans are falling deeper into poverty and fail-
inition. This information best supports answer
ing to receive the aid they so desperately need and
choice F.
deserve.” These statements best support answer
choice C.
PASSAGE III
16. The best answer is G. As indicated in the pas-
sage, “Orshansky’s statistical measurement was 21. The best answer is A. This passage is written
one small part of the federal government’s plan to by someone reflecting on her past. Specifically,
attack the difficult national economic conditions the author reflects on both her childhood and
that were hurting millions of Americans in the her college years. According to this information,
early 1960s. President Lyndon Johnson labeled answer choices B and C can be eliminated. The
the plan the government’s “War on Poverty,” and author mentions Sr. Benedicta, but the author is
it led to such national programs as Head Start, not Sr. Benedicta. Therefore, answer choice D can
VISTA, and the Jobs Corps.” These statements be eliminated, making answer choice A best.
best support answer choice G.
22. The best answer is F. Initially, the author
17. The best answer is B. The first paragraph of discusses her childhood yearning to meet
the passage states that, “Poverty is an enduring J.R.R. Tolkien. Paragraph 3 details this longing.
problem that must be addressed by all modern However, after studying at Oxford as a young
societies.” It also states that, “projects have to adult and experiencing Tolkien’s life second-hand,
be evaluated to see if they are effective. No one the author realizes that she does not have to meet
can measure the benefits of aid without defining him to enjoy his writing.
762 PRACTICE TEST 9 ANSWERS AND EXPLANATIONS

23. The best answer is D. The author begins the published.” Based on this information, one can
passage detailing one of J.R.R. Tolkien’s great- infer that the author very much enjoyed Tolkien’s
est works. “Of course, today, most people think writings and she continued reading his works.
of him as the author of The Lord of the Rings— This best supports answer choice B.
a monumental work that became an epic film.”
28. The best answer is F. The author says that she
This statement indicates the author’s acknowl-
loved “the stories our housekeeper would tell of
edgement of Tolkien’s talent. Later in the passage,
Lawrence of Arabia’s ghost who, apparently, lived
when speaking of Tolkien’s personality, the author
in our own quarters. When inexplicable drafts
says, “Apparently, most people have this impres-
would sweep my room, our housekeeper swore
sion of Tolkien’s as a gentle, grandfatherly sort
it was Lawrence.” One can infer from this state-
of man, but, unless you were his grandchild, that
ment that the housekeeper thought Lawrence of
wasn’t actually the case. In person, he was fre-
Arabia’s ghost haunted their quarters.
quently severe and not terribly friendly.” This
statement describes Tolkien’s intimidating per- 29. The best answer is C. In the passage, the author
sonality. Together, this information best supports says that Sr. Benedicta “was no slouch as an
answer choice D. academic,” followed by the statement that the
professor had “published several highly-regarded
24. The best answer is G. As stated by the author, “I
books in her field.” Based on this information,
loved Oxford.” Throughout the seventh paragraph
one can infer that Sr. Benedicta was an excellent
of the passage, the author lists several reasons
researcher.
to support this love for the university. The other
answer choices are not supported by the tone or 30. The best answer is J. The passage states, “Oxford
details of the passage. isn’t set up like most American universities. It’s
not a single uniform entity. Instead, it’s a collec-
25. The best answer is D. In the fourth paragraph tion of 39 independent colleges, each with its own
of the passage, the author says, “But, growing internal structure and activities, with an overly-
up in the Midwest, the possibility of traveling to ing administration that conducts examinations and
England seemed very remote. Then I discovered confers degrees.” As it is used in this statement,
that Tolkien had died years before I’d even started the word uniform means consistent.
The Hobbit. I forgot about my dream and got down
to the business of school and sports and college
PASSAGE IV
applications.” Based on this information, one can
infer that the author came to an understanding of 31. The best answer is A. By reading the passage, one
her situation. learns about El Niño and its effects. For exam-
ple, the first paragraph ends with the following
26. The best answer is H. As stated by the author,
statement: “Of these, probably the best-known
“Sr. Benedicta was a very smart but very kindly,
is the El Niño-Southern Oscillation, popularly
elderly nun. She was no slouch as an academic and
called ‘El Niño.’” Following this, the article offers
had published several highly-regarded books in
details about the oscillation, such as, “Along
her field. As a colleague, she had spent time with
this area of South America, El Niños reduce the
Tolkien when she was newly hired at St. Stephen’s
upwelling of cold, nutrient-rich water that sustains
College. One day, when she had asked me how
large fish populations. Predators such as larger
I liked studying at Oxford, I decided to tell her
fish and sea birds depend on these populations
about my dream.” The author’s decision to share
for survival, as do local fisheries.” The passage
this dream was an effort to connect to Tolkien
also details the various political ramifications of
through Benedicta, which best supports answer
El Niño. This information best supports answer
choice H.
choice A.
27. The best answer is B. The author begins the
32. The best answer is G. According to the passage,
passage with the following, “I was introduced to
“The term El Niño was first reported in scien-
Tolkien’s writings by a friend when I was 10 years
tific circles in 1892.” Because 1892 is within the
old. Aileen gave me a copy of The Hobbit, and told
1890s, answer choice G is best.
me her father was reading The Lord of the Rings
to her and her brother at the dinner table every 33. The best answer is C. The paragraph starts
night after the family had finished eating. I read with the following: “Sometimes, pressure caused
The Hobbit and was hooked. By the time I was by intense weather can have unexpected polit-
14, I had read every piece of fiction Tolkien had ical effects.” The paragraph then goes on to
PRACTICE TEST 9 ANSWERS AND EXPLANATIONS 763

give examples of how the weather may have 38. The best answer is G. According to the pas-
caused certain political events, such as the French sage, “Some scientists argue that unusually cold
Revolution and Germany’s failure to capture weather brought by a strong El Niño phenomenon
Moscow during WWII. This information best caused significant crop damage in 1788–89, which
supports answer choice C. many say contributed to the French Revolution.
Other climate researchers claim that strong oscil-
34. The best answer is F. Paragraph 2 of the passage lation coupling, combined with strong El Niños in
puts the following information in parentheses, the late 1930s and early 1940s, led to a profound
“The reverse phenomenon, a cold ocean current, cold snap in Northern Europe in the middle of the
is known by a corresponding term, La Niña, Second World War. The scientists argue that this
Spanish for ‘little girl.’” While this information unexpected cold snap significantly contributed to
is interesting and related to the topic, it is not the failure of Germany to capture Moscow, which
immediately relevant to its surrounding context. changed the course of World War II.” This infor-
mation taken in its entirety best supports answer
choice G. Answer choices F and H are not sup-
35. The best answer is D. The paragraph states,
ported by the passage, and, although the passage
“Sometimes, however, the various oscillations
indicates that some scientists argue that El Niño
“beat” together at the same frequency, causing
changed the course of the Second World War,
the fluctuations to be synchronized. When this
nothing in the passage suggests that Germany
happens, scientists say the resulting weather can
would have won had it not been for El Niño.
be intensified.” These details best support answer
choice D. 39. The best answer is D. According to the passage,
“Weather effects can be damaging. The warming
36. The best answer is H. The third paragraph of patterns of El Niño are one of the leading causes
the passage begins with the following: “As cli- of natural damage to coral reefs, while wider
matology developed as a discipline, scientists ENSO fluctuations may cause flooding or drought
discovered that both trends in the current were part to occur on land. In these cases, extreme shifts
of a larger phenomenon affecting global climate can cause economic pressure by disrupting entire
patterns, the Southern Oscillation. The definition fishing industries or damaging crops.” Based on
of El Niño has therefore expanded and continues this information, damage to coral reefs, flooding,
to change as climate researchers compile more drought, and the disruption of fishing industries
data.” The paragraph then goes on to detail the and crops are all possible effects of El Niño. Only
process by which this definition is determined, answer choice D is NOT mentioned specifically
which best supports answer choice H. in the passage as a negative consequence.
40. The best answer is J. The last paragraph of the
37. The best answer is A. When detailing El Niño, passage states that, “ENSO phenomena, along
the passage states, “Most of the time, strong with the other three oscillations, are separate from
El Niños bring wet winters to the Southwestern those attributed to global warming. The causes are
United States and milder winters to the Midwest. completely independent.” The passage also indi-
They tend to bring dry conditions to Indone- cates that “because El Niño and global warming
sia and northern Australia.” Of the four given both can result in strong temperature variability,
answers, answer choice A is most closely related disruptive rain distribution, and extreme damage
to these weather effects. Because California is in to a variety of ecosystems, any synchronicity will
the Southwestern United States, unusually wet be closely observed by scientists seeking to doc-
weather indicates an El Niño weather pattern. ument the total effects of each.” Based on this
Furthermore, because droughts are reported in information, once can conclude that, while they
Australia, it seems likely that an El Niño oscil- are independent of one another, El Niño patterns
lation is present. The other answer choices are and global warming have similar effects of global
not supported by the passage. weather, making answer choice J best.
822 PRACTICE TEST 10 ANSWER KEY

ANSWER KEY

English Test
1. D 21. A 41. C 61. D
2. F 22. H 42. J 62. G
3. A 23. D 43. A 63. A
4. H 24. G 44. F 64. H
5. C 25. C 45. C 65. A
6. G 26. J 46. H 66. H
7. D 27. B 47. D 67. D
8. G 28. H 48. F 68. G
9. C 29. A 49. C 69. C
10. J 30. G 50. J 70. J
11. B 31. A 51. A 71. B
12. F 32. G 52. G 72. J
13. B 33. C 53. D 73. B
14. F 34. H 54. H 74. F
15. B 35. B 55. C 75. C
16. J 36. F 56. G
17. A 37. C 57. B
18. H 38. H 58. F
19. B 39. A 59. D
20. J 40. F 60. J
824 PRACTICE TEST 10 ANSWER KEY

Reading Test Science Reasoning Test


1. C 21. C 1. B 21. D
2. J 22. J 2. H 22. H
3. B 23. C 3. C 23. B
4. H 24. F 4. F 24. F
5. D 25. D 5. A 25. B
6. G 26. G 6. H 26. H
7. D 27. B 7. B 27. C
8. J 28. J 8. G 28. G
9. A 29. A 9. D 29. D
10. H 30. F 10. J 30. G
11. C 31. B 11. A 31. D
12. F 32. F 12. J 32. F
13. D 33. D 13. D 33. B
14. G 34. G 14. F 34. H
15. A 35. C 15. C 35. A
16. H 36. H 16. G 36. J
17. A 37. B 17. B 37. A
18. J 38. F 18. J 38. G
19. D 39. D 19. C 39. D
20. G 40. J 20. G 40. G
PRACTICE TEST 10 ANSWERS AND EXPLANATIONS 829

ANSWERS AND EXPLANATIONS

English Test Explanations

PASSAGE I
1. The best answer is D. The question requires you to shows that yellow fever is not spread by direct
determine the correct punctuation. A good rule of contact.
thumb when it comes to commas is to use them
7. The best answer is D. This question requires you to
where you would naturally pause when reading
determine whether to use the plural or possessive
a sentence. A comma is not necessary anywhere
form of a word. An apostrophe is used to show
within the underlined portion of the sentence, so
possession. Since the “theories” do not possess
answer choice D is correct.
anything, eliminate answer choices A and C. “The-
2. The best answer is F. The best way to answer orize” is a verb that means to “formulate a theory.”
this question is to go through each of the answer Eliminate answer choice B. The sentence requires
choices and determine which of the phrases works the plural form of “theory,” which is “theories,” so
best when substituted for the underlined portion of answer choice D is correct.
the sentence. Answer choice G can be eliminated
because it is awkward and does not make sense. 8. The best answer is G. This question requires you
Answer choice H can also be eliminated because to use specific language. It does not make sense
it is too wordy. Yellow fever was a serious dis- that either the doctors or the Army was infect-
ease, not just “annoying,” so answer choice J can ing soldiers with yellow fever, so eliminate answer
be eliminated because it is not supported by the choices H and J. Likewise, eliminate answer choice
context of the passage. The sentence is best as it is F, because it implies that the doctors were infecting
written. the soldiers.

3. The best answer is A. “In spite of” and “regard- 9. The best answer is C. The question requires you
less of” are prepositional phrases that are not to determine which descriptive word best empha-
supported by the rest of the sentence. Eliminate sizes how fast yellow fever was infecting soldiers.
answer choices B and D. The sentence no longer “Great,” “normal,” and “discouraging” do not
makes sense when “it was” is substituted for effectively describe the rate of infection, so answer
the underlined portion, so answer choice C can choices A, B, and D can eliminated. “An alarm-
eliminated. ing” rate best emphasizes the rapid spread of yellow
fever, and it is idiomatic.
4. The best answer is H. The question tests your
ability to express the idea clearly and simply. The 10. The best answer is J. The question tests your
underlined portion lacks a verb that is parallel to ability to express the idea clearly and simply,
the other verbs used in the sentence. Eliminate and to identify irrelevant information. “So help-
answer choice F. Answer choices G and J are wordy ing his cause,” and “which helped his cause”
and slightly awkward. The phrase “commonly are both unnecessary and awkward within this
accepted” is clear and concise. sentence. The most clear and simple way to
express the sentence is to end it after the word
5. The best answer is C. The question requires you “experiments.”
to correctly punctuate the underlined portion of the
sentence. A semicolon should be followed by an 11. The best answer is B. The question requires you
independent clause. Since “such as the clothing or to determine the correct form of the underlined
blankets …” could not stand alone, answer choice portion of the sentence. “Materials” is plural, so
A should be eliminated. Answer choice B creates answer choices C and D can be eliminated because
an incomplete sentence, and answer choice D is both are in singular form. Since the paragraph dis-
awkward and ungrammatical. cusses events that occurred in the past, answer
choice A can also be eliminated, as “have” is
6. The best answer is G. To answer this question, present tense.
look for an answer choice that proves that the
spread of yellow fever is different from any other 12. The best answer is F. The first two sentences of the
disease that is spread by contact with an infected paragraph discuss the first part of the experiment.
person. Answer choice G is correct because it “A second building …” begins the discussion of a
830 PRACTICE TEST 10 ANSWERS AND EXPLANATIONS

different part of the experiment. The sentence as 19. The best answer is B. The phrase “by nature”
written makes it clear that the paragraph discusses should be treated as extra information in the sen-
separate experiments. tence. This is called a parenthetical expression, and
it should be set off by commas. In other words,
13. The best answer is B. The question requires you
there should be a comma before “by” and a comma
to determine the form of “repeat” that best fits in
after “nature.”
the context of the sentence. To maintain parallel
construction, the past-tense verb “repeated” should 20. The best answer is J. The question requires you
be used. This clearly indicates that the experiments to determine the correct way to express that the
were performed “many times.” The other answer dense habitat belongs to the Giant Pandas. “There”
choices do not make sense when used within the refers to a location, not possession, so answer
sentence. choice F can be eliminated. Eliminate answer
choice G because it does not make it clear to whom
14. The best answer is F. Paragraph 3 discusses the habitat belongs. “They’re” is a contraction of
the experiments that were run to test the source “they are,” which does not express possession, so
of yellow fever. Paragraph 4 explains that the eliminate answer choice H.
results of the experiments saved many people
from yellow fever. Answer choice F is the only 21. The best answer is A. The question tests your
selection that successfully links Paragraph 3 to ability to express the idea clearly and simply. The
Paragraph 4. sentence is best as it is written. The other answer
choices are awkward, and they do not effectively
15. The best answer is B. This question requires you express the idea that the pandas are able to stay
to express the idea clearly and simply. Only answer away from one another if they so choose.
choice B clearly identifies the effect of the vaccines
on the incidence of yellow fever outbreaks. Answer 22. The best answer is H. The question requires you
choice A is awkward, and answer choices C and D to correctly punctuate the underlined portion of the
are irrelevant. sentence. The sentence as written makes the second
sentence incomplete. Eliminate answer choice F.
A semicolon should be followed by an independent
clause, so answer choice G can be eliminated. Since
PASSAGE II
answer choice J would create a run-on sentence,
16. The best answer is J. The question requires you it should be eliminated as well.
to determine which form you should use to sug-
gest that the Giant Panda is native to China. “It’s” 23. The best answer is D. The sentence as it is written
is a contraction for “it is,” so answer choice F is redundant, so answer choice A can be eliminated.
can be eliminated. The sentence refers to “it,” Answer choices C and B can also be eliminated
which is singular, so answer choice G can be elim- because both are awkward. The only selection that
inated. “There” indicates location, not possession, is clear and simple is answer choice D.
so answer choice H can be eliminated. “Its” is sin- 24. The best answer is G. A comma is used to separate
gular and correctly implies that China is the Great the items in a list, but only if the list includes three
Panda’s native country. or more items. No commas are necessary within the
underlined portion, so answer choice G is correct.
17. The best answer is A. The sentence implies that
the Giant Pandas have been pushed from low to 25. The best answer is C. A comma is used to sepa-
high altitudes. In other words, the underlined por- rate the items in a list, but only if the list includes
tion should suggest that the pandas lived at lower three or more items. No commas are necessary
altitudes at sometime in the past. The remaining within the underlined portion, so answer choice C is
answer choices are not supported by the context correct.
the sentence.
26. The best answer is J. This question tests your abil-
18. The best answer is H. The sentence requires that ity to express the idea clearly and simply. The first
the underlined portion refer to an event that has thing you should do is eliminate answer choices F
already occurred. “Will push,” “will have pushed,” and G because both are awkward. It does not make
and “will be pushing,” refer to events that may sense to say “The female perching,” so eliminate
happen in the future. Answer choice H indicates answer choice H. Only answer choice J effectively
that the pandas have already been pushed into indicates what the female is doing and where she
higher altitudes, which is correct. is doing it.
PRACTICE TEST 10 ANSWERS AND EXPLANATIONS 831

27. The best answer is B. The question tests your abil- the footsteps followed Alfred, so eliminate answer
ity to express the idea clearly and simply. The first choice H.
step is to recognize that you should use the noun
33. The best answer is C. Since the sentence is refer-
“rivals,” not “rivalry.” Eliminate answer choice D.
ring to events that occurred in the past, the verb
Next, the word “potential” is used as an adjective to
tense must also be in the past, so eliminate answer
describe “rivals,” it is not used as an adverb (poten-
choice A. “Devoted” is the correct form of the verb,
tially) to describe the action of “fending off,” so
so eliminate answer choice B. Alfred devoted “his
eliminate answer choices A and C.
energies,” not “of his energies,” so answer choice C
28. The best answer is H. The preceding sentence is correct.
refers to the female pandas only, so the “their” in the
34. The best answer is H. The question requires
next sentence still refers to multiple female pandas.
you to determine which of the answer choices
Answer choice F suggest that the cubs dig the dens,
is an example of the “dangers of dynamite during
which does not make sense. Answer choice G does
Nobel’s time.” Answer choice F refers to “today,”
not refer to multiple females and can be eliminated.
not Nobel’s time, so it can be eliminated. Answer
“She digs” refers to only one female panda, so
choices G and J can be eliminated because they do
answer choice J can also be eliminated.
not include any examples of the dangers of using
29. The best answer is A. The first sentence of dynamite.
Paragraph 4 begins with “Therefore” and then sug-
gests that a Giant Panda is a popular display at 35. The best answer is B. This question tests your abil-
any zoo. A logical introduction should explain why ity to recognize redundancy. Since the word “also”
the Giant Panda is a favorite at any zoo. Answer is used earlier in the sentence, it is not necessary
choice A is correct because it says that most people to include it again. Eliminate answer choice C.
have never seen the Giant Panda, which is a log- “In addition” and “too” are synonyms of “also,”
ical reason as to why the Giant Panda is popular so eliminate answer choices A and D.
at the zoo. 36. The best answer is F. This question tests your abil-
30. The best answer is G. The question requires you ity to express the idea clearly and simply. Eliminate
to determine the proper form of the verb “to keep.” answer choices G and H, because they are wordy
Since the rest of the sentence is in present tense, and redundant. It is not necessary to include the
the verb should also be in present tense. Therefore, word “from” in the sentence, so eliminate answer
“that keeps” is the best choice. Answer choices H choice J. The sentence as it is written clearly indi-
and J are awkward and should be eliminated. cates where the lab was moved from (out of the
city) and to (onto a barge).
37. The best answer is C. To answer this question cor-
PASSAGE III rectly, look for the most precise answer choice.
Eliminate answer choices B and D, because they
31. The best answer is A. The question tests your
do not make sense when inserted into the sentence.
ability to properly determine the verb tense of
The word “some” is too general; based on the con-
the sentence. Although the naming of the Nobel
text of the sentence, Nobel refined one way, or “a
Peace Prize occurred in the past, it is still named
way” to control the detonation of dynamite.
after Alfred Nobel, so the verb should be in
the present tense. Eliminate answer choice C 38. The best answer is H. The dynamite belongs
because it refers to the future, and eliminate answer to Nobel, so answer choice F can be eliminated
choice D because it refers to the past. Answer because it does not show possession. Now you
choice B does not make any sense because it must decide between “Nobels”’ and “Nobel’s.”
does not include “after,” which is essential to the Nobel is a single person, so the proper way to
sentence. The Nobel Peace Prize is not named show possession is with an apostrophe s. The sen-
Alfred Nobel. tence would no longer makes sense if you omitted,
or removed, the underlined portion, so eliminate
32. The best answer is G. Since the sentence is refer-
answer choice J.
ring to events that occurred in the past, the verb
tense must also be in the past. “Followed” is the 39. The best answer is A. The best way to answer this
correct form of the verb, and Alfred followed “in” question is to determine which sentence belongs in
his father’s footsteps, not “by” them, so answer the first position. Sentence 1 effectively introduces
choice G is correct. It does not make sense that the paragraph, and serves as a good transition
832 PRACTICE TEST 10 ANSWERS AND EXPLANATIONS

from the preceding paragraph. Eliminate answer PASSAGE IV


choices C and D, which do not place Sentence 1 in
46. The best answer is H. This question tests your
the first position. Sentence 1 shows where Nobel
ability to recognize redundancy. The word “many”
moved his lab. Sentence 2 states that “there,” in that
indicates both a “series” or a “variety,” so elimi-
new location, Nobel refined the way in which dyna-
nate answer choices F, G, and J because they are
mite is controlled. Therefore, Sentences 2 should
redundant. Inserting the word “many” clearly and
follow Sentence 1. Only answer choice A has all of
simply expresses the idea.
the sentences in the correct order.
47. The best answer is D. This question tests your
40. The best answer is F. “Who” is a pronoun that ability to express the idea clearly and simply. The
refers to “Nobel.” “That” and “which” do not refer passage discusses the “construction,” or “build-
to a person, so both answer choices G and H can ing,” of homes. Answer choices A and B are not
be eliminated. Answer choice J creates a run-on idiomatic. Answer choice C implies that the sub-
sentence, so it should be eliminated. division “built” the plot of land, so it should be
eliminated.
41. The best answer is C. The best way to answer
this question is to go through each of the answer 48. The best answer is F. The choice with the most
choices and determine which of the phrases works descriptive language will provide the most detailed
best when substituted for the underlined portion image. Answer choices G and H are too broad,
of the sentence. All of the answer choices make and they do not include details about how many
sense and convey the same meaning, except answer trees there are on the plot of land. While “tree-
choice C. It does not make sense to say that Nobel filled” is more descriptive, it does not evoke the
“had interesting literature, poetry, and social image of thick woods that the writer is trying to
issues.” convey.

42. The best answer is J. “In his will” is an intro- 49. The best answer is C. This question tests your abil-
ductory phrase that must be followed by a comma ity to express the idea clearly and simply, as well as
separating it from the rest of the sentence. Answer your ability to select the correct verb form. Since
choices F, G, and H can all be eliminated because “builders” is plural, you should use the plural verb
they do not have a comma after “will.” Also, form “take.” Eliminate answer choice D. The word
only answer choice J makes it clear that, in his “care” as it is used in the sentence should be singu-
will, Nobel instructed his estate to establish the lar, not plural, so eliminate answer choice B. Since
prizes. the paragraph is written in the simple present tense,
answer choice C most effectively and accurately
43. The best answer is A. The sentence makes it clear expresses the idea.
that more than one prize should be given, so elim-
50. The best answer is J. This question tests your abil-
inate answer choice B. Nobel’s wish was to set
ity to recognize redundancy. The writer includes
up “prizes,” not an “establishment,” so eliminate
the words “as possible” earlier in the sentence, so
answer choice C. Answer choice D does not make
it is not necessary to use them again at the end of
sense when inserted into the sentence, so it should
the sentence. Omitting the underlined portion gets
also be eliminated.
rid of the redundancy created by the other answer
44. The best answer is F. Because the sentence is choices.
discussing Nobel’s date of death, the singular pos- 51. The best answer is A. This question requires you
sessive is correct. Nobel only has one date of death, to punctuate the underlined portion correctly. The
so eliminate answer choice J. words “large” and “heavy” are coordinate adjec-
tives describing the “equipment.” It is necessary to
45. The best answer is C. The phrase “all over the separate coordinate adjectives with a comma if you
world” is idiomatic. This question requires you to do not use a coordinate conjunction such as “and.”
express the idea clearly and simply. The sentence Answer choice D uses a comma in the right place,
as it is written is awkward and does not make sense, but the word “largely” is not appropriate.
so eliminate answer choice A. Answer choice B is
redundant and should be eliminated. Omitting, or 52. The best answer is G. This question tests your
removing the underlined portion gets rid of infor- ability to express the idea clearly. The sentence as it
mation that is essential to the sentence. Eliminate is written includes the ambiguous pronoun “they.”
answer choice D. It is unclear to whom or to what the pronoun “they”
PRACTICE TEST 10 ANSWERS AND EXPLANATIONS 833

is referring. Eliminate answer choice F. Likewise, explanation of the importance of a tree’s root
answer choices H and J are ambiguous and unclear, system.
and should be eliminated. Only answer choice G
59. The best answer is D. Since Sentence 4 explains
clearly indicates who chooses to flatten the land.
why it is necessary to keep roots from becoming
53. The best answer is D. This question tests your buried, it makes sense that Sentence 5 should be
ability to express the idea clearly. The sentence as placed immediately before Sentence 4. Placing it
it is written includes the ambiguous pronoun “it.” elsewhere in the paragraph would not make sense.
It is unclear to whom or to what the pronoun “it”
60. The best answer is J. This question requires you
is referring. Eliminate answer choice A. Answer
to determine the main idea of the essay. Since the
choice C includes the possessive form “its,”
focus is not on ways to prevent damage during con-
which is not correct here. Only answer choice D
struction, eliminate answer choices F and G. The
clearly indicates what is not detected until much
essay focuses primarily on how trees and their root
later.
systems can become damaged during construction.
54. The best answer is H. This question tests your Answer choices J best supports this idea.
ability to express the idea clearly and simply. The
context of the sentence suggests that a tree will
probably die if 40 percent or more of its root system PASSAGE V
“is lost.” Since the paragraph is written in simple
61. The best answer is D. This question tests your
present tense, the best selection is answer choice H.
ability to express the idea clearly and simply. It
55. The best answer is C. To maintain parallel con- does not make sense that an officer of the law
struction within the sentence, the subject and verb or an armed military person would be wearing
must agree. Since the subject is plural (“tree roots”) both a suit and a uniform, so eliminate answer
you must use the plural form of the verb (“extend”). choice A. Likewise, eliminate answer choices B
Eliminate answer choices A and B, which include and C because they do not effectively convey the
singular verb forms. The paragraph is written connection between an officer of the law or an
in simple present tense, so eliminate answer armed military person, and what those people are
choice D which includes the future tense of the likely to be wearing.
verb form.
62. The best answer is G. In this sentence, the word
56. The best answer is G. To maintain parallel con- “most” is used to indicate the number of people
struction within the sentence, the subject and verb who might opt for a bullet-proof vest. “Most”
must agree. Since the subject is plural (“bulldozers, should always be used with a plural noun, so elim-
dump trucks, and cement trucks”) you must use the inate answer choice F. Answer choices H and J are
plural form of the verb (“drive”). Eliminate answer awkward, and do not clearly and simply express
choice F. The paragraph is written in the simple the idea, so they should be eliminated as well.
present tense, so eliminate answer choice H, which
63. The best answer is A. This question tests your abil-
includes the past tense form of the verb, and answer
ity to express the idea clearly and simply. Answer
choice J, which includes the past participle form of
choice B is unclear—prevents bullets from doing
the verb.
what? Therefore, it should be eliminated. Answer
57. The best answer is B. This question requires you to choice C creates a double negative, so it should
punctuate the underlined portion correctly. A good be eliminated. Answer choice D is too wordy and
rule of thumb when it comes to commas is to use contains irrelevant information, so it should be
them where you would naturally pause when read- eliminated as well. The sentence as it is written
ing a sentence. There is no natural pause after the best conveys the idea that bullet-proof vests aren’t
word “equipment” or the word “destroy.” Since the actually bullet-proof.
idea is that “compacting” the soil “may destroy”
64. The best answer is H. This question requires
the tree, you do not need to separate the clauses
you to punctuate the underlined portion correctly.
with a comma.
A good rule of thumb when it comes to com-
58. The best answer is F. A detailed explanation of mas is to use them where you would naturally
why a tree’s root system is so important would pause when reading a sentence. A semicolon should
best accomplish the goal presented in the question. be followed by an independent clause, so elimi-
Answer choices G, H, and J, while correct, are nate answer choice F. Answer choice G creates an
too broad and do not effectively continue the incomplete sentence, and answer choice J creates
834 PRACTICE TEST 10 ANSWERS AND EXPLANATIONS

a run-on sentence. Both of these choices should be see if it makes sense and effectively conveys the
eliminated. Only answer choice H clearly indicates idea. Answer choices F, G, and H are very wordy
that weaponry, ammunition, and materials have all and awkward, and they do not express the idea as
changed through the centuries. clearly and simply as does answer choice J.
65. The best answer is A. This question tests your 71. The best answer is B. This question requires you
ability to express the idea clearly and simply. to correctly punctuate the underlined portion. The
Since “silk” is the material being discussed, it sentence as it is written is a run-on sentence, so
makes sense to place the underlined portion directly eliminate answer choice A. Answer choice C cre-
after the word “silk,” as it is written. Placing the ates an incomplete sentence, so it should also be
underlined portion elsewhere makes the sentence eliminated. A semicolon should be followed by an
awkward and unclear. independent clause, so eliminate answer choice D.
66. The best answer is H. This question requires you 72. The best answer is J. This question tests your abil-
to select the best transition between sentences. The ity to express the idea clearly and simply, which,
word “soon” implies a sense of time that does in this case, means eliminating answer choices that
not fit within the context of the paragraph. Elimi- include irrelevant information. Since the sentence
nate answer choice F. The paragraph discusses the already indicates that the impact is spread around
importance of protecting the wearer from bullets. the body, any information about how this is in con-
It makes sense that, because the flak-jacket did trast to the impact being sustained in one area is
not protect the wearer from bullets, that the best redundant and irrelevant. Omitting, or removing,
transition word to use is “unfortunately.” the underlined portion keeps the sentence simple,
while still effectively expressing the idea.
67. The best answer is D. This question tests your
ability to recognize redundancy. The word “today” 73. The best answer is B. The question asks you to
suggests “recent” and “modern,” so it is not neces- indicate that body armor does not always prevent
sary to include all of those words in the sentence. injury or death. Answer choices A, C, and D sug-
The most simple way to express the idea is with gest that the material in a bullet-proof vest will
the word “today’s.” often or always prevent injury or death. Answer
choice B effectively shows that, only under some
68. The best answer is G. The best way to answer this circumstances, but certainly not always, will body
question correctly is to insert the answer choices armor prevent injury or death.
in the sentence and select the one that makes
sense. You must answer the question, “A variety 74. The best answer is F. The best way to answer
of synthetic materials, some of which are” what? this question correctly, is to determine what, if
The word “effective” best answers that question. anything, the underlined portion adds to the sen-
Answer choices F, H, and J are very awkward and tence. The underlined portion includes informa-
do not successfully convey the idea being presented tion that indicates that the bullet-proof-vest wearer
in the sentence. could sustain serious injury while wearing the vest.
If this information is removed, the paragraph would
69. The best answer is C. The preceding sentence lose a statement that suggests protective gear does
discusses the “production” of synthetic materials, not guarantee or ensure a lack of serious injury.
so you should look for an answer choice that further The other answer choices are not supported by the
explains the production of these materials. Elim- context of the paragraph.
inate answer choices A and D because they do
not discuss production of material. You must also 75. The best answer is C. This question requires you
tie the idea presented in the preceding sentence to to select the correct verb form. The verb “entrust”
the concept of cost, since that is mentioned in the means to “place trust in,” which makes sense based
sentence about which you are being asked. Answer on the context of the paragraph. The idea being
choice C does this best. conveyed is that police officers and military per-
sonal know that their safety cannot be “entrusted”
70. The best answer is J. This question tests your abil- completely to the protective gear that they wear;
ity to express the idea clearly and simply. In this they must take additional precautions to ensure
case, you should try the shortest answer choice to their safety.
842 PRACTICE TEST 10 ANSWERS AND EXPLANATIONS

Reading Test Explanations impossible that Mr. Rochester should have a sin-
cere affection for me?” Miss Eyre is indicating that
PASSAGE I she believes Mr. Rochester is truly fond of her and
she is hurt that Mrs. Fairfax has not yet demon-
1. The best answer is C. The sentence before the
strated that she believes or thinks this. The other
quote states, “but, believe me, you cannot be too
answer choices are not supported by the details in
careful. Try and keep Mr. Rochester at a distance:
the passage.
distrust yourself as well as him.” Mrs. Fairfax is
suggesting that Mr. Rochester’s feelings should not 8. The best answer is J. Throughout the pas-
be trusted because they may not be genuine. This sage, Miss Eyre expresses her confidence that
best supports answer choice C. Mr. Rochester is not too old for her. She refutes
Mrs. Fairfax’s claim that people might mistake
2. The best answer is J. Mrs. Fairfax states, “It is an
Mr. Rochester for Miss Eyre’s father. Therefore,
old saying that ‘all is not gold that glitters’; and in
she is self-assured in her feelings. The other answer
this case I do fear there will be something found to
choices are not supported by the passage.
be different to what either you or I expect.” This
shows that Mrs. Fairfax believes Miss Eyre will 9. The best answer is A. Mrs. Fairfax implies that
discover that things may not turn out as she hoped Mr. Rochester, in deciding to marry Miss Eyre,
or expected and may regret her decision. The other is not being his normally careful and proud self.
answer choices are not supported by the passage. Mrs. Fairfax is shocked and surprised by the
marriage proposal.
3. The best answer is B. Mrs. Fairfax says, “Gentle-
men in his station are not accustomed to marry their 10. The best answer is H. Mrs. Fairfax states, “I am
governesses.” She is pointing out a difference in sorry to grieve you . . . but you are so young, and so
Miss Eyre and Mr. Rochester’s position and fortune little acquainted with men, I wished to put you on
and hinting that this difference is not a good thing. your guard. It is an old saying that ‘all is not gold
Mrs. Fairfax also clearly indicates that she is uncer- that glitters’; and in this case I do fear there will be
tain about Miss Eyre’s future with Mr. Rochester. something found to be different to what either you
This information best supports answer choice B. or I expect.” Mrs. Fairfax is expressing her fear
that things may go wrong or not be as they appear
4. The best answer is H. Mrs. Fairfax is explaining
and that Miss Eyre will end up getting hurt. The
that she would have cautioned Miss Eyre against
other answer choices are not supported by details
forming a relationship with Mr. Rochester, but
in the passage.
Miss Eyre had seemed mature and wise enough
to conclude on her own that forming an intimate
relationship with Mr. Rochester would be unwise. PASSAGE II
This best supports answer choice H.
11. The best answer is C. The first paragraph lists
5. The best answer is D. Mrs. Fairfax states, the main concerns of Abraham Lincoln that were
“Gentlemen in his station are not accustomed to addressed during the debates. The rest of the passage
marry their governesses.” The words, “not accus- then provides details, definitions, and explanations
tomed to” imply that this is not a common occur- about the concepts introduced in the first paragraph.
rence, and that their relationship is not typical.
12. The best answer is F. In the second paragraph,
The other answer choices are not supported by the
Lincoln refers to the actions of the Founding
passage.
Fathers. This provided historical evidence for his
6. The best answer is G. Mrs. Fairfax reminds argument against the current state of slavery in
Miss Eyre that, “all is not gold that glitters.” There- America during the debates. The other answer
fore, she is using the analogy of an object that choices are not supported by the passage.
appears to be gold, may not really be gold. In this
13. The best answer is D. The passage states, “The
way, Miss Eyre’s relationship with Mr. Rochester
Missouri Compromise and the Compromise of
may appear to be genuine and good, but it might
1850 were at odds with the new Dred Scott deci-
not be. This best supports answer choice G.
sion, which denied that Congress had a right to
7. The best answer is D. Mrs. Fairfax says, exclude slavery in the territories.” This suggests
“Mr. Rochester, I dare say, is fond of you. I have that Congress may have initially had the right to
always noticed that you were a sort of pet of his.” exclude slavery. The other answer choices are not
Miss Eyre asks, “Why?—am I a monster? Is it supported by the passage.
PRACTICE TEST 10 ANSWERS AND EXPLANATIONS 843

14. The best answer is G. The emphasis of these lines for, and her last performances. This best supports
on the physical violence that was occurring over answer choice C.
slavery in Kansas supports the idea that slavery was
22. The best answer is J. The author mentions that
an issue that was dividing citizens and states.
she appeared in A Trip to Scarborough, which was
15. The best answer is A. The passage states that, “the altered, from Vanbrugh’s Relapse. The passage
‘Spirit of ’76’. . . was the hope of the Founding does not say that the author actually appeared in
Fathers that slavery would be strangled within Vanbrugh’s Relapse.
the original southern states over time.” This best
23. The best answer is C. According to the passage,
supports answer choice A.
the author was quite nervous and fearful about
16. The best answer is H. The passage states, going on stage: “I then began to fear that my resolu-
“Lincoln’s ‘House Divided’ speech and his tion would fail . . .” This information best supports
arguments in the Lincoln-Douglas debates show the selection of the word “courage.”
that he believed that slavery was threatening to
24. The best answer is F. The author states that she
become a national institution. He saw the American
played the character of Sir Henry Revel in The
public become increasingly indifferent to slavery.”
Miniature Picture. Therefore, she did not always
This best supports answer choice H.
play female characters. The other answer choices
17. The best answer is A. The passage states, “A key are not supported by the details in the passage.
piece of evidence is that the Dred Scott decision
25. The best answer is D. The author states, “The
was pushed back until after the election of 1856.”
second character which I played was Amanda, in
The other answer choices are not supported by the
A Trip to Scarborough. The play was altered from
passage.
Vanbrugh’s Relapse.” This suggests that the author
18. The best answer is J. The Declaration of Indepen- acted in at least one, un-original play. The other
dence is not specifically mentioned in this passage. answer choices are not supported by the passage.
The passage states, “He mentioned the unani-
26. The best answer is G. The passage states, “The
mous abolition of the African slave trade as well
second character which I played was Amanda, in
as, the Northwest Ordinance and the lack of the
A Trip to Scarborough.” Then the author writes,
word slave in the Constitution, to show that the
“The third character I played was Statira, in
Founding Fathers intended slavery to be strangled
Alexander the Great.” Finally, the author writes,
in the original Southern States.”
“The last character that I played was Sir Harry
19. The best answer is D. The passage states, “The Revel, in Lady Craven’s comedy of The Minia-
Missouri Compromise and the Compromise of ture Picture; and the epilogue song in The Irish
1850 were at odds with the new Dred Scott deci- Widow was my last farewell to the labor of my
sion, which denied that Congress had a right to profession.” Since you were able to determine that
exclude slavery in the territories.” The passage she first appeared in A Trip to Scarborough, Roman
reports that the Missouri Compromise was used to Numeral III, you would eliminate all of the answer
regulate slavery in the territories. The passage also choices that did not place Roman Numeral III in
states that the supporters of slavery now favored the first position.
the Dred Scott decision that effectively overruled
27. The best answer is B. The narrator writes, “I stood
the Missouri Compromise.
mute and bending with alarm, which did not sub-
20. The best answer is G. The passage states, “In side till I had feebly articulated the few sentences
addition, the Democrats had made previous leg- of the first short scene, during the whole of which
islation in 1850 and 1854 that had language that I had never once ventured to look at the audi-
seemed to predict that Congress would not be ence.” Later in the night, the narrator says, “As
able to exclude slavery in the territories because I acquired courage, I found the applause aug-
of Constitutional constraints.” The other answer ment; and the night was concluded with peals of
choices are not supported by the passage. loud approbation. I was complimented on all sides.
I then experienced, for the first time in my life,
a pleasure that language could not explain.” This
PASSAGE III best supports answer choice B.
21. The best answer is C. The author recounts her first 28. The best answer is J. Quotes from the pas-
three performances, the roles she was most famous sage include: “My dress was a pale pink satin,
844 PRACTICE TEST 10 ANSWERS AND EXPLANATIONS

trimmed with crêpe, and richly spangled with also result in cankers and dieback similar to ash
silver”, “I leaned upon the Nurse’s arm, almost yellows.” The passage indicates that the trees may
fainting. Mr. Sheridan and several other friends decline slowly, not rapidly.
encouraged me to proceed”, and “The second scene
34. The best answer is G. The passage states, “Ash
being the masquerade, I had time to collect myself.”
yellows is caused by a phytoplasma: virus-like
Nothing in the passage suggests that the narra-
pathogens that are spread by insects.”
tor received criticism for delivering her opening
lines feebly. 35. The best answer is C. The passage states, “The
decline of these trees may have a severe impact on
29. The best answer is A. The crowd’s applause
the health of other plant and animal communities.
stunned her and she momentarily lost her ability to
Green ash provides nesting sites for several species
think or reason the noise and crowd overwhelmed
of birds and other wild creatures. Insects and fish
her hearing and other senses, answer choice A. The
flourish in the cool waters made possible by the
other answer choices do not make sense based on
shade of ash trees.” This best supports answer
the context.
choice C.
30. The best answer is F. Although a gentleman did
36. The best answer is H. The passage states, “A sur-
comment on the narrator’s performance, this was
vey of several Midwestern states found that some
not the primary focus of these lines. The narra-
ash decline was found to be independent of ash
tor speaks more about being filled with pleasure
yellows. The droughts in the 1980s may have
from hearing the applause and completing her first
caused the decline of ash trees in the Midwest.”
performance. These lines do not state that the
The other answer choices are not supported by the
narrator believed she was famous, nor do these
passage.
lines tell the reader how the narrator believed she
performed. 37. The best answer is B. The passage states that,
“Insects and fish flourish in the cool waters made
possible by the shade of ash trees.” The paragraph
PASSAGE IV discusses the importance of ash trees “in the ecol-
ogy of North American forests.” It makes the most
31. The best answer is B. The passage states, “No
sense, based on the context of the paragraph, that
single factor has been proven to cause ash decline.
“flourish” means “prosper.”
Ash yellows and environmental factors may work
together to create ash decline.” This best supports 38. The best answer is F. Throughout the passage the
answer choice B. author states that ash trees are important and nec-
essary. It makes sense that the author would agree
32. The best answer is F. The passage states, “Verti-
with a statement suggesting that a cure for a disease
cillium wilt on ash can also result in cankers and
that kills ash trees be researched. The other answer
dieback similar to ash yellows.” The other answer
choices are not supported by the passage.
choices are not supported by the passage.
39. The best answer is D. The passage states that ash
33. The best answer is D. The passage gives the
yellows turns the leaves of ash trees “pale green
following reasons why it is hard to diagnose as
and pale yellow,” not bright yellow.
ash tree: “Trying to diagnose a tree is difficult
because symptoms could be caused by a variety 40. The best answer is J. According to the passage,
of problems.” “Ash yellows and environmental there is no known cure for ash yellows. Elimi-
factors may work together to create ash decline.” nate answer choices F and H because they include
“A survey of several Midwestern states found that Roman Numeral I. The remaining Roman Numer-
some ash decline was found to be independent als, II, III, and IV, are supported by details in the
of ash yellows.” “Verticillium wilt on ash can passage, so answer choice J must be correct.

You might also like